Treating pulmonary embolism at home?

Article Type
Changed
Mon, 01/14/2019 - 11:34
Display Headline
Treating pulmonary embolism at home?
PRACTICE CHANGER

Treat low-risk patients with pulmonary embolism (PE) with low-molecular-weight heparin (LMWH) in an outpatient setting.1

Aujesky D, Roy PM, Verschuren F, et al. Outpatient versus inpatient treatment for patients with acute pulmonary embolism: an international, open-label, randomised, non-inferiority trial. Lancet. 2011; 378:41-48.

STRENGTH OF RECOMMENDATION

B: Based on one good quality randomized controlled trial (RCT).

 

ILLUSTRATIVE CASE

Three months after undergoing surgical repair of an ankle fracture, a 50-year-old woman presents with acute onset dyspnea at rest and pleuritic chest pain. Her left calf is tender and swollen. The patient has a history of hypertension and smokes about 10 cigarettes per day. Her temperature is 37°C (99°F); pulse rate, 98; blood pressure, 135/85 mm Hg; respiratory rate, 25; and pulse oximetry, 92%.

You order a spiral CT, which reveals a contrast filling defect indicative of a PE. Her score on the Pulmonary Embolism Severity Index (PESI) is 50, an indication of low risk. She wants to know if she can be treated at home. What should you tell her?

In the past, intravenous unfractionated heparin, administered in an inpatient setting, was the recommended initial anticoagulation therapy for patients with venous thromboembolism (VTE). LMWH, which can be administered subcutaneously and does not require laboratory monitoring, has made it possible to treat VTE without hospitalization.

Outpatient PE care hindered by lack of evidence
Guidelines from the American College of Physicians, American Academy of Family Physicians, and British Thoracic Society recommend outpatient treatment of deep vein thrombosis with LMWH, which they find to be safe and cost effective for select patients.2,3 Until recently, the safety and efficacy of out-patient management of PE has been less clear.

The lack of an accurate prediction tool to identify patients who could be treated safely outside of the hospital was one barrier to the development of evidence-based recommendations for outpatient PE treatment. In 2005, the PESI,4 a validated tool that identifies patients with low risk of death from PE, was developed. Until recently, the absence of an RCT comparing inpatient and outpatient treatment for acute PE was another barrier.

STUDY SUMMARY: Outpatient treatment measures up

The Outpatient Treatment of Pulmonary Embolism (OTPE) study was a multinational, randomized, noninferiority trial comparing outpatient vs inpatient treatment of low-risk patients with acute PE. Participants had to be ≥18 years old, have acute symptomatic and objectively verified PE, and be at low risk of death based on the PESI score.4 In addition to excluding patients at moderate or high risk, the researchers identified 14 other exclusion criteria, including hypoxia, chest pain requiring opiates, and high risk for bleeding.

Patients were randomly assigned to the outpatient (n=171) or inpatient (n=168) group. Both groups received subcutaneous LMWH (enoxaparin, 1 mg/kg twice a day) for ≥5 days, followed by oral anticoagulation with a vitamin K antagonist for ≥90 days. Patients in the outpatient group were discharged from the emergency department (ED) within 24 hours of randomization, after being trained by a nurse to self-inject. Therapy after discharge was managed either by the patient’s primary care physician or the hospital’s anticoagulation staff.

The LMWH was discontinued in patients with an INR ≥2.0 for 2 consecutive days. All patients were followed for 90 days, and contacted by the study team daily for the first week and then at 14, 30, 60, and 90 days. On each occasion, participants were asked about symptoms of recurrent VTE, bleeding, and the use of health care resources.

The primary outcome was the recurrence of symptomatic, objectively confirmed VTE within the study period. Secondary outcomes were major bleeding and all-cause mortality. Outcomes were confirmed by clinicians who were unaware of treatment assignments.

Patients were also asked to rate both their overall satisfaction with their care and their treatment preference 14 days after randomization, using a 5-point Likert questionnaire. Prior to the trial, the investigators decided that outpatient treatment would be considered noninferior to inpatient care if the difference between rates of recurrent VTE did not exceed 4%, a measure used in previous studies comparing treatment regimens for VTE and outpatient vs inpatient treatment of DVT.5,6

Little difference in readmission rates, ED or office visits
One in 171 outpatients (0.6%) and none of the inpatients had recurrent VTE. Two out-patients (1.2%)—and no inpatients—developed major bleeding within 14 days, the result of intramuscular hematomas that occurred on Days 3 and 13. There was one additional bleeding event (menometrorrhagia) in the outpatient group on Day 50, but it was believed to be unrelated to the PE treatment. Per-protocol analysis, a more conservative measure used in noninferiority studies, found a difference in major bleeding rates of 3.8%. One person in each group died from non-VTE and nontreatment-related causes.

 

 

 

Almost all participants (99%) completed the satisfaction survey, which indicated that 92% of outpatients and 95% of inpatients were satisfied or very satisfied with their care. Hospital readmission rates, ED visits, and visits to primary care physicians were similar, with no significant differences between the 2 groups. The mean time spent in the hospital was 0.5 days (standard deviation [SD], 1.0) for outpatients and 3.9 days (SD, 3.1) for in-patients. Fourteen percent of outpatients and 6% of inpatients received home nursing visits for enoxaparin injection. The total number of home visits was higher among outpatients (348 vs 105). Because both groups had extreme outliers, however, this difference was not statistically significant.

WHAT’S NEW: It’s safe to keep low-risk patients at home

This is the first RCT comparing the safety and effectiveness of outpatient and inpatient treatment of acute, symptomatic PE. Results were statistically comparable, and patients were satisfied being treated at home. Outpatient treatment was less expensive because of the shorter length of stay (0.5 vs 3.9 days) and was associated with the same rates of hospital readmission, ED visits, and visits to primary care physicians. There were more home nursing visits in the outpatient treatment group. But even if you assume a cost of $200 per home visit, the additional cost would be about $282 per individual in the outpatient group—significantly less than the cost of the additional 3.4 days in the hospital for each individual in the inpatient group.

The study also confirmed that the PESI accurately identifies low-risk patients with PE who can be treated in an outpatient setting. Thirty percent of patients who were screened for the OTPE trial met the low-risk eligibility requirement.

CAVEATS: Use of risk assessment tool is essential

The average age of patients in this study was 47 years in the outpatient group and 49 years in the inpatient group. In addition, only 1% to 3% of the patients were diagnosed with cancer. Older patients who have both cancer and PE would be unlikely to qualify for outpatient care.

Physicians applying this practice changer should use the PESI to ensure that outpatient treatment for PE is used only for individuals at low risk.

CHALLENGES TO IMPLEMENTATION: ED coordination, training, and home care won’t be easy

This practice changer may be difficult for family physicians, who might not be included in emergency physicians’ decisions regarding the appropriate treatment for acute PE. In this study, primary care physicians were notified of the randomized treatment plan for their patients, and 17 potential participants were excluded from the trial because of their doctors’ opposition.

Outpatient management should be considered only if arrangements for adequate home nursing care can be made, if needed— and only for patients who are able to follow instructions and self-inject LMWH. Newer anticoagulation medications that are either injected once a day or taken orally might decrease the need for home nursing visits.

Acknowledgement

The PURLs Surveillance System is supported in part by Grant Number UL1RR024999 from the National Center for Research Resources, a Clinical Translational Science award to the University of Chicago. The content is solely the responsibility of the authors and does not necessarily represent the official views of the National Center for Research Resources or the National Institutes of Health.

Click here to view PURL METHODOLOGY

References

1. Aujesky D, Roy PM, Verschuren F, et al. Outpatient versus inpatient treatment for patients with acute pulmonary embolism: an international, open-label, randomised, non-inferiority trial. Lancet. 2011;378:41-48.

2. Snow V, Qaseem A, Barry P, et al. Management of venous thromboembolism: a clinical practice guideline from the American College of Physicians and the American Academy of Family Physicians. Ann Intern Med. 2007;146:204-210.

3. British Thoracic Society Standards of Care Committee Pulmonary Embolism Guideline Development Group. British Thoracic Society guidelines for the management of suspected acute pulmonary embolism. Thorax. 2003;58:470-483.

4. Aujesky D, Obrosky DS, Stone RA, et al. Derivation and validation of a prognostic model for pulmonary embolism. Am J Respir Crit Care Med. 2005;172:1041-1046.

5. Koopman MM, Prandoni P, Piovella F, et al. Treatment of venous thrombosis with intravenous unfractionated heparin administered in the hospital as compared with subcutaneous low-molecular-weight heparin administered at home. The Tasman Study Group. N Engl J Med. 1996;334:682-687.

6. Schulman S, Kearon C, Kakkar AK, et al. Dabigatran versus warfarin in the treatment of acute venous thromboembolism. N Engl J Med. 2009;361:2342-2352.

Article PDF
Author and Disclosure Information
Outpatient care of PE: tips from an emergency physician
Donald M. Yealy, MD

Mari Egan, MD
University of Chicago

Kate Rowland, MD
University of Chicago

PURLs EDITOR
John Hickner, MD, MSc
Cleveland Clinic

Issue
The Journal of Family Practice - 61(6)
Publications
Topics
Page Number
349-352
Legacy Keywords
Mari Egan ;MD; Kate Rowland ;MD; acute PE; low-molecular-weight heparin; LMWH; outpatient PE care; venous thromboembolism; OTPE study; treatment preference
Sections
Author and Disclosure Information
Outpatient care of PE: tips from an emergency physician
Donald M. Yealy, MD

Mari Egan, MD
University of Chicago

Kate Rowland, MD
University of Chicago

PURLs EDITOR
John Hickner, MD, MSc
Cleveland Clinic

Author and Disclosure Information
Outpatient care of PE: tips from an emergency physician
Donald M. Yealy, MD

Mari Egan, MD
University of Chicago

Kate Rowland, MD
University of Chicago

PURLs EDITOR
John Hickner, MD, MSc
Cleveland Clinic

Article PDF
Article PDF
PRACTICE CHANGER

Treat low-risk patients with pulmonary embolism (PE) with low-molecular-weight heparin (LMWH) in an outpatient setting.1

Aujesky D, Roy PM, Verschuren F, et al. Outpatient versus inpatient treatment for patients with acute pulmonary embolism: an international, open-label, randomised, non-inferiority trial. Lancet. 2011; 378:41-48.

STRENGTH OF RECOMMENDATION

B: Based on one good quality randomized controlled trial (RCT).

 

ILLUSTRATIVE CASE

Three months after undergoing surgical repair of an ankle fracture, a 50-year-old woman presents with acute onset dyspnea at rest and pleuritic chest pain. Her left calf is tender and swollen. The patient has a history of hypertension and smokes about 10 cigarettes per day. Her temperature is 37°C (99°F); pulse rate, 98; blood pressure, 135/85 mm Hg; respiratory rate, 25; and pulse oximetry, 92%.

You order a spiral CT, which reveals a contrast filling defect indicative of a PE. Her score on the Pulmonary Embolism Severity Index (PESI) is 50, an indication of low risk. She wants to know if she can be treated at home. What should you tell her?

In the past, intravenous unfractionated heparin, administered in an inpatient setting, was the recommended initial anticoagulation therapy for patients with venous thromboembolism (VTE). LMWH, which can be administered subcutaneously and does not require laboratory monitoring, has made it possible to treat VTE without hospitalization.

Outpatient PE care hindered by lack of evidence
Guidelines from the American College of Physicians, American Academy of Family Physicians, and British Thoracic Society recommend outpatient treatment of deep vein thrombosis with LMWH, which they find to be safe and cost effective for select patients.2,3 Until recently, the safety and efficacy of out-patient management of PE has been less clear.

The lack of an accurate prediction tool to identify patients who could be treated safely outside of the hospital was one barrier to the development of evidence-based recommendations for outpatient PE treatment. In 2005, the PESI,4 a validated tool that identifies patients with low risk of death from PE, was developed. Until recently, the absence of an RCT comparing inpatient and outpatient treatment for acute PE was another barrier.

STUDY SUMMARY: Outpatient treatment measures up

The Outpatient Treatment of Pulmonary Embolism (OTPE) study was a multinational, randomized, noninferiority trial comparing outpatient vs inpatient treatment of low-risk patients with acute PE. Participants had to be ≥18 years old, have acute symptomatic and objectively verified PE, and be at low risk of death based on the PESI score.4 In addition to excluding patients at moderate or high risk, the researchers identified 14 other exclusion criteria, including hypoxia, chest pain requiring opiates, and high risk for bleeding.

Patients were randomly assigned to the outpatient (n=171) or inpatient (n=168) group. Both groups received subcutaneous LMWH (enoxaparin, 1 mg/kg twice a day) for ≥5 days, followed by oral anticoagulation with a vitamin K antagonist for ≥90 days. Patients in the outpatient group were discharged from the emergency department (ED) within 24 hours of randomization, after being trained by a nurse to self-inject. Therapy after discharge was managed either by the patient’s primary care physician or the hospital’s anticoagulation staff.

The LMWH was discontinued in patients with an INR ≥2.0 for 2 consecutive days. All patients were followed for 90 days, and contacted by the study team daily for the first week and then at 14, 30, 60, and 90 days. On each occasion, participants were asked about symptoms of recurrent VTE, bleeding, and the use of health care resources.

The primary outcome was the recurrence of symptomatic, objectively confirmed VTE within the study period. Secondary outcomes were major bleeding and all-cause mortality. Outcomes were confirmed by clinicians who were unaware of treatment assignments.

Patients were also asked to rate both their overall satisfaction with their care and their treatment preference 14 days after randomization, using a 5-point Likert questionnaire. Prior to the trial, the investigators decided that outpatient treatment would be considered noninferior to inpatient care if the difference between rates of recurrent VTE did not exceed 4%, a measure used in previous studies comparing treatment regimens for VTE and outpatient vs inpatient treatment of DVT.5,6

Little difference in readmission rates, ED or office visits
One in 171 outpatients (0.6%) and none of the inpatients had recurrent VTE. Two out-patients (1.2%)—and no inpatients—developed major bleeding within 14 days, the result of intramuscular hematomas that occurred on Days 3 and 13. There was one additional bleeding event (menometrorrhagia) in the outpatient group on Day 50, but it was believed to be unrelated to the PE treatment. Per-protocol analysis, a more conservative measure used in noninferiority studies, found a difference in major bleeding rates of 3.8%. One person in each group died from non-VTE and nontreatment-related causes.

 

 

 

Almost all participants (99%) completed the satisfaction survey, which indicated that 92% of outpatients and 95% of inpatients were satisfied or very satisfied with their care. Hospital readmission rates, ED visits, and visits to primary care physicians were similar, with no significant differences between the 2 groups. The mean time spent in the hospital was 0.5 days (standard deviation [SD], 1.0) for outpatients and 3.9 days (SD, 3.1) for in-patients. Fourteen percent of outpatients and 6% of inpatients received home nursing visits for enoxaparin injection. The total number of home visits was higher among outpatients (348 vs 105). Because both groups had extreme outliers, however, this difference was not statistically significant.

WHAT’S NEW: It’s safe to keep low-risk patients at home

This is the first RCT comparing the safety and effectiveness of outpatient and inpatient treatment of acute, symptomatic PE. Results were statistically comparable, and patients were satisfied being treated at home. Outpatient treatment was less expensive because of the shorter length of stay (0.5 vs 3.9 days) and was associated with the same rates of hospital readmission, ED visits, and visits to primary care physicians. There were more home nursing visits in the outpatient treatment group. But even if you assume a cost of $200 per home visit, the additional cost would be about $282 per individual in the outpatient group—significantly less than the cost of the additional 3.4 days in the hospital for each individual in the inpatient group.

The study also confirmed that the PESI accurately identifies low-risk patients with PE who can be treated in an outpatient setting. Thirty percent of patients who were screened for the OTPE trial met the low-risk eligibility requirement.

CAVEATS: Use of risk assessment tool is essential

The average age of patients in this study was 47 years in the outpatient group and 49 years in the inpatient group. In addition, only 1% to 3% of the patients were diagnosed with cancer. Older patients who have both cancer and PE would be unlikely to qualify for outpatient care.

Physicians applying this practice changer should use the PESI to ensure that outpatient treatment for PE is used only for individuals at low risk.

CHALLENGES TO IMPLEMENTATION: ED coordination, training, and home care won’t be easy

This practice changer may be difficult for family physicians, who might not be included in emergency physicians’ decisions regarding the appropriate treatment for acute PE. In this study, primary care physicians were notified of the randomized treatment plan for their patients, and 17 potential participants were excluded from the trial because of their doctors’ opposition.

Outpatient management should be considered only if arrangements for adequate home nursing care can be made, if needed— and only for patients who are able to follow instructions and self-inject LMWH. Newer anticoagulation medications that are either injected once a day or taken orally might decrease the need for home nursing visits.

Acknowledgement

The PURLs Surveillance System is supported in part by Grant Number UL1RR024999 from the National Center for Research Resources, a Clinical Translational Science award to the University of Chicago. The content is solely the responsibility of the authors and does not necessarily represent the official views of the National Center for Research Resources or the National Institutes of Health.

Click here to view PURL METHODOLOGY

PRACTICE CHANGER

Treat low-risk patients with pulmonary embolism (PE) with low-molecular-weight heparin (LMWH) in an outpatient setting.1

Aujesky D, Roy PM, Verschuren F, et al. Outpatient versus inpatient treatment for patients with acute pulmonary embolism: an international, open-label, randomised, non-inferiority trial. Lancet. 2011; 378:41-48.

STRENGTH OF RECOMMENDATION

B: Based on one good quality randomized controlled trial (RCT).

 

ILLUSTRATIVE CASE

Three months after undergoing surgical repair of an ankle fracture, a 50-year-old woman presents with acute onset dyspnea at rest and pleuritic chest pain. Her left calf is tender and swollen. The patient has a history of hypertension and smokes about 10 cigarettes per day. Her temperature is 37°C (99°F); pulse rate, 98; blood pressure, 135/85 mm Hg; respiratory rate, 25; and pulse oximetry, 92%.

You order a spiral CT, which reveals a contrast filling defect indicative of a PE. Her score on the Pulmonary Embolism Severity Index (PESI) is 50, an indication of low risk. She wants to know if she can be treated at home. What should you tell her?

In the past, intravenous unfractionated heparin, administered in an inpatient setting, was the recommended initial anticoagulation therapy for patients with venous thromboembolism (VTE). LMWH, which can be administered subcutaneously and does not require laboratory monitoring, has made it possible to treat VTE without hospitalization.

Outpatient PE care hindered by lack of evidence
Guidelines from the American College of Physicians, American Academy of Family Physicians, and British Thoracic Society recommend outpatient treatment of deep vein thrombosis with LMWH, which they find to be safe and cost effective for select patients.2,3 Until recently, the safety and efficacy of out-patient management of PE has been less clear.

The lack of an accurate prediction tool to identify patients who could be treated safely outside of the hospital was one barrier to the development of evidence-based recommendations for outpatient PE treatment. In 2005, the PESI,4 a validated tool that identifies patients with low risk of death from PE, was developed. Until recently, the absence of an RCT comparing inpatient and outpatient treatment for acute PE was another barrier.

STUDY SUMMARY: Outpatient treatment measures up

The Outpatient Treatment of Pulmonary Embolism (OTPE) study was a multinational, randomized, noninferiority trial comparing outpatient vs inpatient treatment of low-risk patients with acute PE. Participants had to be ≥18 years old, have acute symptomatic and objectively verified PE, and be at low risk of death based on the PESI score.4 In addition to excluding patients at moderate or high risk, the researchers identified 14 other exclusion criteria, including hypoxia, chest pain requiring opiates, and high risk for bleeding.

Patients were randomly assigned to the outpatient (n=171) or inpatient (n=168) group. Both groups received subcutaneous LMWH (enoxaparin, 1 mg/kg twice a day) for ≥5 days, followed by oral anticoagulation with a vitamin K antagonist for ≥90 days. Patients in the outpatient group were discharged from the emergency department (ED) within 24 hours of randomization, after being trained by a nurse to self-inject. Therapy after discharge was managed either by the patient’s primary care physician or the hospital’s anticoagulation staff.

The LMWH was discontinued in patients with an INR ≥2.0 for 2 consecutive days. All patients were followed for 90 days, and contacted by the study team daily for the first week and then at 14, 30, 60, and 90 days. On each occasion, participants were asked about symptoms of recurrent VTE, bleeding, and the use of health care resources.

The primary outcome was the recurrence of symptomatic, objectively confirmed VTE within the study period. Secondary outcomes were major bleeding and all-cause mortality. Outcomes were confirmed by clinicians who were unaware of treatment assignments.

Patients were also asked to rate both their overall satisfaction with their care and their treatment preference 14 days after randomization, using a 5-point Likert questionnaire. Prior to the trial, the investigators decided that outpatient treatment would be considered noninferior to inpatient care if the difference between rates of recurrent VTE did not exceed 4%, a measure used in previous studies comparing treatment regimens for VTE and outpatient vs inpatient treatment of DVT.5,6

Little difference in readmission rates, ED or office visits
One in 171 outpatients (0.6%) and none of the inpatients had recurrent VTE. Two out-patients (1.2%)—and no inpatients—developed major bleeding within 14 days, the result of intramuscular hematomas that occurred on Days 3 and 13. There was one additional bleeding event (menometrorrhagia) in the outpatient group on Day 50, but it was believed to be unrelated to the PE treatment. Per-protocol analysis, a more conservative measure used in noninferiority studies, found a difference in major bleeding rates of 3.8%. One person in each group died from non-VTE and nontreatment-related causes.

 

 

 

Almost all participants (99%) completed the satisfaction survey, which indicated that 92% of outpatients and 95% of inpatients were satisfied or very satisfied with their care. Hospital readmission rates, ED visits, and visits to primary care physicians were similar, with no significant differences between the 2 groups. The mean time spent in the hospital was 0.5 days (standard deviation [SD], 1.0) for outpatients and 3.9 days (SD, 3.1) for in-patients. Fourteen percent of outpatients and 6% of inpatients received home nursing visits for enoxaparin injection. The total number of home visits was higher among outpatients (348 vs 105). Because both groups had extreme outliers, however, this difference was not statistically significant.

WHAT’S NEW: It’s safe to keep low-risk patients at home

This is the first RCT comparing the safety and effectiveness of outpatient and inpatient treatment of acute, symptomatic PE. Results were statistically comparable, and patients were satisfied being treated at home. Outpatient treatment was less expensive because of the shorter length of stay (0.5 vs 3.9 days) and was associated with the same rates of hospital readmission, ED visits, and visits to primary care physicians. There were more home nursing visits in the outpatient treatment group. But even if you assume a cost of $200 per home visit, the additional cost would be about $282 per individual in the outpatient group—significantly less than the cost of the additional 3.4 days in the hospital for each individual in the inpatient group.

The study also confirmed that the PESI accurately identifies low-risk patients with PE who can be treated in an outpatient setting. Thirty percent of patients who were screened for the OTPE trial met the low-risk eligibility requirement.

CAVEATS: Use of risk assessment tool is essential

The average age of patients in this study was 47 years in the outpatient group and 49 years in the inpatient group. In addition, only 1% to 3% of the patients were diagnosed with cancer. Older patients who have both cancer and PE would be unlikely to qualify for outpatient care.

Physicians applying this practice changer should use the PESI to ensure that outpatient treatment for PE is used only for individuals at low risk.

CHALLENGES TO IMPLEMENTATION: ED coordination, training, and home care won’t be easy

This practice changer may be difficult for family physicians, who might not be included in emergency physicians’ decisions regarding the appropriate treatment for acute PE. In this study, primary care physicians were notified of the randomized treatment plan for their patients, and 17 potential participants were excluded from the trial because of their doctors’ opposition.

Outpatient management should be considered only if arrangements for adequate home nursing care can be made, if needed— and only for patients who are able to follow instructions and self-inject LMWH. Newer anticoagulation medications that are either injected once a day or taken orally might decrease the need for home nursing visits.

Acknowledgement

The PURLs Surveillance System is supported in part by Grant Number UL1RR024999 from the National Center for Research Resources, a Clinical Translational Science award to the University of Chicago. The content is solely the responsibility of the authors and does not necessarily represent the official views of the National Center for Research Resources or the National Institutes of Health.

Click here to view PURL METHODOLOGY

References

1. Aujesky D, Roy PM, Verschuren F, et al. Outpatient versus inpatient treatment for patients with acute pulmonary embolism: an international, open-label, randomised, non-inferiority trial. Lancet. 2011;378:41-48.

2. Snow V, Qaseem A, Barry P, et al. Management of venous thromboembolism: a clinical practice guideline from the American College of Physicians and the American Academy of Family Physicians. Ann Intern Med. 2007;146:204-210.

3. British Thoracic Society Standards of Care Committee Pulmonary Embolism Guideline Development Group. British Thoracic Society guidelines for the management of suspected acute pulmonary embolism. Thorax. 2003;58:470-483.

4. Aujesky D, Obrosky DS, Stone RA, et al. Derivation and validation of a prognostic model for pulmonary embolism. Am J Respir Crit Care Med. 2005;172:1041-1046.

5. Koopman MM, Prandoni P, Piovella F, et al. Treatment of venous thrombosis with intravenous unfractionated heparin administered in the hospital as compared with subcutaneous low-molecular-weight heparin administered at home. The Tasman Study Group. N Engl J Med. 1996;334:682-687.

6. Schulman S, Kearon C, Kakkar AK, et al. Dabigatran versus warfarin in the treatment of acute venous thromboembolism. N Engl J Med. 2009;361:2342-2352.

References

1. Aujesky D, Roy PM, Verschuren F, et al. Outpatient versus inpatient treatment for patients with acute pulmonary embolism: an international, open-label, randomised, non-inferiority trial. Lancet. 2011;378:41-48.

2. Snow V, Qaseem A, Barry P, et al. Management of venous thromboembolism: a clinical practice guideline from the American College of Physicians and the American Academy of Family Physicians. Ann Intern Med. 2007;146:204-210.

3. British Thoracic Society Standards of Care Committee Pulmonary Embolism Guideline Development Group. British Thoracic Society guidelines for the management of suspected acute pulmonary embolism. Thorax. 2003;58:470-483.

4. Aujesky D, Obrosky DS, Stone RA, et al. Derivation and validation of a prognostic model for pulmonary embolism. Am J Respir Crit Care Med. 2005;172:1041-1046.

5. Koopman MM, Prandoni P, Piovella F, et al. Treatment of venous thrombosis with intravenous unfractionated heparin administered in the hospital as compared with subcutaneous low-molecular-weight heparin administered at home. The Tasman Study Group. N Engl J Med. 1996;334:682-687.

6. Schulman S, Kearon C, Kakkar AK, et al. Dabigatran versus warfarin in the treatment of acute venous thromboembolism. N Engl J Med. 2009;361:2342-2352.

Issue
The Journal of Family Practice - 61(6)
Issue
The Journal of Family Practice - 61(6)
Page Number
349-352
Page Number
349-352
Publications
Publications
Topics
Article Type
Display Headline
Treating pulmonary embolism at home?
Display Headline
Treating pulmonary embolism at home?
Legacy Keywords
Mari Egan ;MD; Kate Rowland ;MD; acute PE; low-molecular-weight heparin; LMWH; outpatient PE care; venous thromboembolism; OTPE study; treatment preference
Legacy Keywords
Mari Egan ;MD; Kate Rowland ;MD; acute PE; low-molecular-weight heparin; LMWH; outpatient PE care; venous thromboembolism; OTPE study; treatment preference
Sections
PURLs Copyright

Copyright © 2012 The Family Physicians Inquiries Network. All rights reserved.

Disallow All Ads
Alternative CME
Article PDF Media

BP meds: This simple change improves outcomes

Article Type
Changed
Mon, 01/14/2019 - 11:33
Display Headline
BP meds: This simple change improves outcomes

PRACTICE CHANGER

Advise patients with uncontrolled hypertension to take at least one of their blood pressure (BP) medications at bedtime instead of in the morning. Nighttime dosing leads to better control and lowers the risk of major cardiovascular events.1,2

STRENGTH OF RECOMMENDATION

B: Based on a well-done randomized clinical trial (RCT) and a subgroup analysis.

Hermida RC, Ayala DE, Mojón A, et al. Influence of circadian time of hypertension treatment on cardiovascular risk: results of the MAPEC study. Chronobiol Int. 2010;27:1629-1651.

Hermida RC, Ayala DE, Mojón A, et al. Influence of time of day of blood pressure-lowering treatment on cardiovascular risk in hypertensive patients with type 2 diabetes. Diabetes Care. 2011;34:1270-1276.

ILLUSTRATIVE CASES

  • A 60-year-old man has struggled to get his BP under control despite the use of 3 anti-hypertensives. Is there anything you can recommend to improve his BP control and lower his cardiovascular risk?
  • You prescribe hydrochlorothiazide for a 55-year-old woman with newly diagnosed hypertension. What can you tell her about how to take the medication to maximize its beneficial effects?

Management of hypertension often centers around BP measurements taken in a doctor’s office during the day, although both BP and metabolism fluctuate with circadian rhythms. Most people experience an increase in pressure during the day, with peaks in the morning and evening, followed by a decline in BP while they sleep at night.3

The focus belongs on nighttime BP
Sleeping BP is getting considerable attention, particularly the phenomenon of nondipping. Commonly defined as a <10% decline in systolic pressure during sleep, nondipping is associated with an increased risk of cardiovascular events, such as heart attack and stroke.4 What’s more, mean BP during the night is a better predictor of cardiovascular disease (CVD) risk than BP while the patient is awake.5,6

Evidence suggests that taking an anti-hypertensive medication at night increases its therapeutic effect,7 yet most patients take it in the morning.8 The study detailed in this PURL was designed to investigate whether bedtime dosing significantly affects BP control and CVD risk.

STUDY SUMMARY: Bedtime dosing benefits patients, and there’s no downside

The MAPEC study was an open-label RCT conducted at a single center in Spain.1 Patients were enrolled if they had a diagnosis of either untreated hypertension (based on ambulatory BP monitoring [ABPM] criteria) or resistant hypertension (uncontrolled on ≥3 optimally dosed antihypertensive medications). Exclusion criteria included pregnancy, a history of drug/alcohol abuse, night shift work, acquired immune deficiency syndrome, type 1 diabetes, secondary hypertension, and a previous CVD diagnosis.

Patients were randomly assigned to one of 2 time-of-day dosing groups: morning dosing of all their BP medications (n=1109) or dosing of ≥1 BP medications at bedtime (n=1092). ABPM—in which patients wore a monitor that recorded their BP every 20 minutes during the day and every 30 minutes at night for 48 hours—was conducted once a year, or more frequently when medication adjustments occurred. The use of a specific drug was not required, but physicians were instructed to adjust medications according to a study-specific ABPM protocol.

Patients were followed for a mean of 5.6 years for the endpoints of CVD events and mortality. These endpoints were assessed by researchers blinded to patients’ treatment assignment.

At baseline, the 2 groups were similar in age (mean of 55 years), percentage of men (48%), presence of comorbidities, and baseline clinic and ambulatory BP. Throughout the study, patients in the bedtime dosing group had lower mean asleep systolic and diastolic BP, a lower prevalence of a non-dipping pattern, and a higher prevalence of controlled ambulatory BP. The bedtime group also had a lower risk of total CVD events (relative risk [RR]=0.39; 95% confidence interval [CI], 0.29-0.51; P<.001) and major CVD events (RR=0.33; 95% CI, 0.19-0.55; P<.001), and fewer overall deaths (4.16/1000 vs 2.11/1000 patient-years; P=.008) (TABLE). To prevent one CVD event, 63 patients would need to take their BP medication at bedtime instead of in the morning for one year. To prevent one death, 488 patient would need to adhere to the nighttime schedule for one year.

A subgroup analysis of patients with type 2 diabetes (n=448)2 had similar results: For this population, too, bedtime dosing led to lower asleep BP, a lower prevalence of a non-dipping pattern, and a higher prevalence of controlled ambulatory BP, as well as a lower risk of total CVD events, major CVD events, and CVD-related death. The differences persisted after correction for the use of statins and aspirin. Among those in this subgroup analysis, 29 patients would need to take their BP medications at bedtime for one year to prevent one CVD event, and 263 patients would need to be treated for one year to prevent one death.

 

 

TABLE
Dosing of BP meds: A look at outcomes

Events/1000 patient-yearsMorning dosingBedtime dosingP between groups
overall (n=2201)1
Total events*27.8011.95<.001
CVd death2.080.53.006
Cardiovascular events11.005.27<.001
Cerebrovascular events3.571.23.001
Diabetes subgroup (n=448)2
Total events*54.2419.80<.001
CVd death4.790.86.038
Cardiovascular events15.956.89.008
Cerebrovascular events6.380.86.010
*Includes death from all causes and cardiovascular and cerebrovascular events.
BP, blood pressure; CVD, cardiovascular disease.

WHAT’S NEW: Advantages of preventing nondipping are clearly established

We’ve known that a nondipping pattern is associated with higher cardiovascular risks and that taking antihypertensives at bedtime decreases the prevalence of nondipping patterns. The MAPEC study, however, is the first prospective trial to show that bedtime dosing of BP medications lowers the risk of CVD events and death.

CAVEATS: Methodology, non-US guidelines raise questions about applicability here

MAPEC was an open-label study, meaning that the physicians adjusting BP medications were aware of the treatment groups to which their patients were allocated. Physicians were given guidelines for the titration of medications, but it is unclear whether they treated patients in both treatment groups identically. Patients were also aware of their treatment group, which creates the potential for bias if one group adhered to their medications more closely than the other.

The study was a single-center trial conducted in Spain, which may limit its generalizability to the United States. Notably, Spain’s medication guidelines differ from ours, with angiotensin-converting enzyme inhibitors, angiotensin receptor blockers, beta-blockers, and calcium channel blockers as first-line medications and hydrochlorothiazide as a second-line option.

While ABPM appears to be a better indicator of CVD risk compared with clinic BP monitoring, most US physicians still rely on readings taken in their office for diagnosing and managing hypertension. How ambulatory BP translates to clinic BP is somewhat unclear.

CHALLENGES TO IMPLEMENTATION: Some patients and providers may resist the switch

We see few challenges to implementing bedtime dosing of BP medications for patients with uncontrolled hypertension. It is possible, however, that patients who have a long-standing routine of taking their medications in the morning may be resistant to change. Also, pharmacists and nurses, as well as some physicians, may continue recommending morning dosing, which could be confusing for patients.

Acknowledgement

The PURLs Surveillance System is supported in part by Grant Number UL1RR024999 from the National Center for Research Resources, a Clinical Translational Science Award to the University of Chicago. The content is solely the responsibility of the authors and does not necessarily represent the official views of the National Center for Research Resources or the National Institutes of Health.

References

1. Hermida RC, Ayala DE, Mojón A, et al. Influence of circadian time of hypertension treatment on cardiovascular risk: results of the MAPEC study. Chronobiol Int. 2010;27:1629-1651.

2. Hermida RC, Ayala DE, Mojón A, et al. Influence of time of day of blood pressure-lowering treatment on cardiovascular risk in hypertensive patients with type 2 diabetes. Diabetes Care. 2011;34:1270-1276.

3. Hermida RC, Ayala DE, Portaluppi F. Circadian variation of blood pressure: the basis for the chronotherapy of hypertension. Adv Drug Deliv Rev. 2007;59:904-922.

4. Verdecchia P, Porcellati C, Schillaci G, et al. Ambulatory blood pressure: an independent predictor of prognosis in essential hypertension. Hypertension. 1994;24:793-801.

5. Dolan E, Stanton A, Thijs L, et al. Superiority of ambulatory over clinic blood pressure measurement in predicting mortality: the Dublin study outcome. Hypertension. 2005;46:156-161.

6. Hermida RC, Ayala DE, Mojón A, et al. Decreasing sleep-time blood pressure determined by ambulatory monitoring reduces cardiovascular risk. J Am Coll Cardiol. 2011;58:1165-1173.

7. De la Sierra A, Redon J, Banegas JR, et al. Prevalence and factors associated with circadian blood pressure patterns in hypertensive patients. Hypertension. 2009;53:466-472.

8. Hermida RC, Ayala DE, Calvo C, et al. Chronotherapy of hypertension: administration-time-dependent effects of treatment on the circadian pattern of blood pressure. Adv Drug Deliv Rev. 2007;59:923-939.

Article PDF
Author and Disclosure Information

Kate Kirley, MD
University of Chicago

Umang Sharma, MD
University of Chicago

Kate Rowland, MD
University of Chicago

PURLs EDITOR
John Hickner, MD, MSc

Cleveland Clinic

Issue
The Journal of Family Practice - 61(3)
Publications
Topics
Page Number
153-155
Legacy Keywords
Kate Kirley;MD; Umang Sharma;MD; Kate Rowland;MD; uncontrolled hypertension; bedtime dosing; blood pressure medications; BP meds; cardiovascular events; open-label RCT; CVD risk; morning dosing
Sections
Author and Disclosure Information

Kate Kirley, MD
University of Chicago

Umang Sharma, MD
University of Chicago

Kate Rowland, MD
University of Chicago

PURLs EDITOR
John Hickner, MD, MSc

Cleveland Clinic

Author and Disclosure Information

Kate Kirley, MD
University of Chicago

Umang Sharma, MD
University of Chicago

Kate Rowland, MD
University of Chicago

PURLs EDITOR
John Hickner, MD, MSc

Cleveland Clinic

Article PDF
Article PDF

PRACTICE CHANGER

Advise patients with uncontrolled hypertension to take at least one of their blood pressure (BP) medications at bedtime instead of in the morning. Nighttime dosing leads to better control and lowers the risk of major cardiovascular events.1,2

STRENGTH OF RECOMMENDATION

B: Based on a well-done randomized clinical trial (RCT) and a subgroup analysis.

Hermida RC, Ayala DE, Mojón A, et al. Influence of circadian time of hypertension treatment on cardiovascular risk: results of the MAPEC study. Chronobiol Int. 2010;27:1629-1651.

Hermida RC, Ayala DE, Mojón A, et al. Influence of time of day of blood pressure-lowering treatment on cardiovascular risk in hypertensive patients with type 2 diabetes. Diabetes Care. 2011;34:1270-1276.

ILLUSTRATIVE CASES

  • A 60-year-old man has struggled to get his BP under control despite the use of 3 anti-hypertensives. Is there anything you can recommend to improve his BP control and lower his cardiovascular risk?
  • You prescribe hydrochlorothiazide for a 55-year-old woman with newly diagnosed hypertension. What can you tell her about how to take the medication to maximize its beneficial effects?

Management of hypertension often centers around BP measurements taken in a doctor’s office during the day, although both BP and metabolism fluctuate with circadian rhythms. Most people experience an increase in pressure during the day, with peaks in the morning and evening, followed by a decline in BP while they sleep at night.3

The focus belongs on nighttime BP
Sleeping BP is getting considerable attention, particularly the phenomenon of nondipping. Commonly defined as a <10% decline in systolic pressure during sleep, nondipping is associated with an increased risk of cardiovascular events, such as heart attack and stroke.4 What’s more, mean BP during the night is a better predictor of cardiovascular disease (CVD) risk than BP while the patient is awake.5,6

Evidence suggests that taking an anti-hypertensive medication at night increases its therapeutic effect,7 yet most patients take it in the morning.8 The study detailed in this PURL was designed to investigate whether bedtime dosing significantly affects BP control and CVD risk.

STUDY SUMMARY: Bedtime dosing benefits patients, and there’s no downside

The MAPEC study was an open-label RCT conducted at a single center in Spain.1 Patients were enrolled if they had a diagnosis of either untreated hypertension (based on ambulatory BP monitoring [ABPM] criteria) or resistant hypertension (uncontrolled on ≥3 optimally dosed antihypertensive medications). Exclusion criteria included pregnancy, a history of drug/alcohol abuse, night shift work, acquired immune deficiency syndrome, type 1 diabetes, secondary hypertension, and a previous CVD diagnosis.

Patients were randomly assigned to one of 2 time-of-day dosing groups: morning dosing of all their BP medications (n=1109) or dosing of ≥1 BP medications at bedtime (n=1092). ABPM—in which patients wore a monitor that recorded their BP every 20 minutes during the day and every 30 minutes at night for 48 hours—was conducted once a year, or more frequently when medication adjustments occurred. The use of a specific drug was not required, but physicians were instructed to adjust medications according to a study-specific ABPM protocol.

Patients were followed for a mean of 5.6 years for the endpoints of CVD events and mortality. These endpoints were assessed by researchers blinded to patients’ treatment assignment.

At baseline, the 2 groups were similar in age (mean of 55 years), percentage of men (48%), presence of comorbidities, and baseline clinic and ambulatory BP. Throughout the study, patients in the bedtime dosing group had lower mean asleep systolic and diastolic BP, a lower prevalence of a non-dipping pattern, and a higher prevalence of controlled ambulatory BP. The bedtime group also had a lower risk of total CVD events (relative risk [RR]=0.39; 95% confidence interval [CI], 0.29-0.51; P<.001) and major CVD events (RR=0.33; 95% CI, 0.19-0.55; P<.001), and fewer overall deaths (4.16/1000 vs 2.11/1000 patient-years; P=.008) (TABLE). To prevent one CVD event, 63 patients would need to take their BP medication at bedtime instead of in the morning for one year. To prevent one death, 488 patient would need to adhere to the nighttime schedule for one year.

A subgroup analysis of patients with type 2 diabetes (n=448)2 had similar results: For this population, too, bedtime dosing led to lower asleep BP, a lower prevalence of a non-dipping pattern, and a higher prevalence of controlled ambulatory BP, as well as a lower risk of total CVD events, major CVD events, and CVD-related death. The differences persisted after correction for the use of statins and aspirin. Among those in this subgroup analysis, 29 patients would need to take their BP medications at bedtime for one year to prevent one CVD event, and 263 patients would need to be treated for one year to prevent one death.

 

 

TABLE
Dosing of BP meds: A look at outcomes

Events/1000 patient-yearsMorning dosingBedtime dosingP between groups
overall (n=2201)1
Total events*27.8011.95<.001
CVd death2.080.53.006
Cardiovascular events11.005.27<.001
Cerebrovascular events3.571.23.001
Diabetes subgroup (n=448)2
Total events*54.2419.80<.001
CVd death4.790.86.038
Cardiovascular events15.956.89.008
Cerebrovascular events6.380.86.010
*Includes death from all causes and cardiovascular and cerebrovascular events.
BP, blood pressure; CVD, cardiovascular disease.

WHAT’S NEW: Advantages of preventing nondipping are clearly established

We’ve known that a nondipping pattern is associated with higher cardiovascular risks and that taking antihypertensives at bedtime decreases the prevalence of nondipping patterns. The MAPEC study, however, is the first prospective trial to show that bedtime dosing of BP medications lowers the risk of CVD events and death.

CAVEATS: Methodology, non-US guidelines raise questions about applicability here

MAPEC was an open-label study, meaning that the physicians adjusting BP medications were aware of the treatment groups to which their patients were allocated. Physicians were given guidelines for the titration of medications, but it is unclear whether they treated patients in both treatment groups identically. Patients were also aware of their treatment group, which creates the potential for bias if one group adhered to their medications more closely than the other.

The study was a single-center trial conducted in Spain, which may limit its generalizability to the United States. Notably, Spain’s medication guidelines differ from ours, with angiotensin-converting enzyme inhibitors, angiotensin receptor blockers, beta-blockers, and calcium channel blockers as first-line medications and hydrochlorothiazide as a second-line option.

While ABPM appears to be a better indicator of CVD risk compared with clinic BP monitoring, most US physicians still rely on readings taken in their office for diagnosing and managing hypertension. How ambulatory BP translates to clinic BP is somewhat unclear.

CHALLENGES TO IMPLEMENTATION: Some patients and providers may resist the switch

We see few challenges to implementing bedtime dosing of BP medications for patients with uncontrolled hypertension. It is possible, however, that patients who have a long-standing routine of taking their medications in the morning may be resistant to change. Also, pharmacists and nurses, as well as some physicians, may continue recommending morning dosing, which could be confusing for patients.

Acknowledgement

The PURLs Surveillance System is supported in part by Grant Number UL1RR024999 from the National Center for Research Resources, a Clinical Translational Science Award to the University of Chicago. The content is solely the responsibility of the authors and does not necessarily represent the official views of the National Center for Research Resources or the National Institutes of Health.

PRACTICE CHANGER

Advise patients with uncontrolled hypertension to take at least one of their blood pressure (BP) medications at bedtime instead of in the morning. Nighttime dosing leads to better control and lowers the risk of major cardiovascular events.1,2

STRENGTH OF RECOMMENDATION

B: Based on a well-done randomized clinical trial (RCT) and a subgroup analysis.

Hermida RC, Ayala DE, Mojón A, et al. Influence of circadian time of hypertension treatment on cardiovascular risk: results of the MAPEC study. Chronobiol Int. 2010;27:1629-1651.

Hermida RC, Ayala DE, Mojón A, et al. Influence of time of day of blood pressure-lowering treatment on cardiovascular risk in hypertensive patients with type 2 diabetes. Diabetes Care. 2011;34:1270-1276.

ILLUSTRATIVE CASES

  • A 60-year-old man has struggled to get his BP under control despite the use of 3 anti-hypertensives. Is there anything you can recommend to improve his BP control and lower his cardiovascular risk?
  • You prescribe hydrochlorothiazide for a 55-year-old woman with newly diagnosed hypertension. What can you tell her about how to take the medication to maximize its beneficial effects?

Management of hypertension often centers around BP measurements taken in a doctor’s office during the day, although both BP and metabolism fluctuate with circadian rhythms. Most people experience an increase in pressure during the day, with peaks in the morning and evening, followed by a decline in BP while they sleep at night.3

The focus belongs on nighttime BP
Sleeping BP is getting considerable attention, particularly the phenomenon of nondipping. Commonly defined as a <10% decline in systolic pressure during sleep, nondipping is associated with an increased risk of cardiovascular events, such as heart attack and stroke.4 What’s more, mean BP during the night is a better predictor of cardiovascular disease (CVD) risk than BP while the patient is awake.5,6

Evidence suggests that taking an anti-hypertensive medication at night increases its therapeutic effect,7 yet most patients take it in the morning.8 The study detailed in this PURL was designed to investigate whether bedtime dosing significantly affects BP control and CVD risk.

STUDY SUMMARY: Bedtime dosing benefits patients, and there’s no downside

The MAPEC study was an open-label RCT conducted at a single center in Spain.1 Patients were enrolled if they had a diagnosis of either untreated hypertension (based on ambulatory BP monitoring [ABPM] criteria) or resistant hypertension (uncontrolled on ≥3 optimally dosed antihypertensive medications). Exclusion criteria included pregnancy, a history of drug/alcohol abuse, night shift work, acquired immune deficiency syndrome, type 1 diabetes, secondary hypertension, and a previous CVD diagnosis.

Patients were randomly assigned to one of 2 time-of-day dosing groups: morning dosing of all their BP medications (n=1109) or dosing of ≥1 BP medications at bedtime (n=1092). ABPM—in which patients wore a monitor that recorded their BP every 20 minutes during the day and every 30 minutes at night for 48 hours—was conducted once a year, or more frequently when medication adjustments occurred. The use of a specific drug was not required, but physicians were instructed to adjust medications according to a study-specific ABPM protocol.

Patients were followed for a mean of 5.6 years for the endpoints of CVD events and mortality. These endpoints were assessed by researchers blinded to patients’ treatment assignment.

At baseline, the 2 groups were similar in age (mean of 55 years), percentage of men (48%), presence of comorbidities, and baseline clinic and ambulatory BP. Throughout the study, patients in the bedtime dosing group had lower mean asleep systolic and diastolic BP, a lower prevalence of a non-dipping pattern, and a higher prevalence of controlled ambulatory BP. The bedtime group also had a lower risk of total CVD events (relative risk [RR]=0.39; 95% confidence interval [CI], 0.29-0.51; P<.001) and major CVD events (RR=0.33; 95% CI, 0.19-0.55; P<.001), and fewer overall deaths (4.16/1000 vs 2.11/1000 patient-years; P=.008) (TABLE). To prevent one CVD event, 63 patients would need to take their BP medication at bedtime instead of in the morning for one year. To prevent one death, 488 patient would need to adhere to the nighttime schedule for one year.

A subgroup analysis of patients with type 2 diabetes (n=448)2 had similar results: For this population, too, bedtime dosing led to lower asleep BP, a lower prevalence of a non-dipping pattern, and a higher prevalence of controlled ambulatory BP, as well as a lower risk of total CVD events, major CVD events, and CVD-related death. The differences persisted after correction for the use of statins and aspirin. Among those in this subgroup analysis, 29 patients would need to take their BP medications at bedtime for one year to prevent one CVD event, and 263 patients would need to be treated for one year to prevent one death.

 

 

TABLE
Dosing of BP meds: A look at outcomes

Events/1000 patient-yearsMorning dosingBedtime dosingP between groups
overall (n=2201)1
Total events*27.8011.95<.001
CVd death2.080.53.006
Cardiovascular events11.005.27<.001
Cerebrovascular events3.571.23.001
Diabetes subgroup (n=448)2
Total events*54.2419.80<.001
CVd death4.790.86.038
Cardiovascular events15.956.89.008
Cerebrovascular events6.380.86.010
*Includes death from all causes and cardiovascular and cerebrovascular events.
BP, blood pressure; CVD, cardiovascular disease.

WHAT’S NEW: Advantages of preventing nondipping are clearly established

We’ve known that a nondipping pattern is associated with higher cardiovascular risks and that taking antihypertensives at bedtime decreases the prevalence of nondipping patterns. The MAPEC study, however, is the first prospective trial to show that bedtime dosing of BP medications lowers the risk of CVD events and death.

CAVEATS: Methodology, non-US guidelines raise questions about applicability here

MAPEC was an open-label study, meaning that the physicians adjusting BP medications were aware of the treatment groups to which their patients were allocated. Physicians were given guidelines for the titration of medications, but it is unclear whether they treated patients in both treatment groups identically. Patients were also aware of their treatment group, which creates the potential for bias if one group adhered to their medications more closely than the other.

The study was a single-center trial conducted in Spain, which may limit its generalizability to the United States. Notably, Spain’s medication guidelines differ from ours, with angiotensin-converting enzyme inhibitors, angiotensin receptor blockers, beta-blockers, and calcium channel blockers as first-line medications and hydrochlorothiazide as a second-line option.

While ABPM appears to be a better indicator of CVD risk compared with clinic BP monitoring, most US physicians still rely on readings taken in their office for diagnosing and managing hypertension. How ambulatory BP translates to clinic BP is somewhat unclear.

CHALLENGES TO IMPLEMENTATION: Some patients and providers may resist the switch

We see few challenges to implementing bedtime dosing of BP medications for patients with uncontrolled hypertension. It is possible, however, that patients who have a long-standing routine of taking their medications in the morning may be resistant to change. Also, pharmacists and nurses, as well as some physicians, may continue recommending morning dosing, which could be confusing for patients.

Acknowledgement

The PURLs Surveillance System is supported in part by Grant Number UL1RR024999 from the National Center for Research Resources, a Clinical Translational Science Award to the University of Chicago. The content is solely the responsibility of the authors and does not necessarily represent the official views of the National Center for Research Resources or the National Institutes of Health.

References

1. Hermida RC, Ayala DE, Mojón A, et al. Influence of circadian time of hypertension treatment on cardiovascular risk: results of the MAPEC study. Chronobiol Int. 2010;27:1629-1651.

2. Hermida RC, Ayala DE, Mojón A, et al. Influence of time of day of blood pressure-lowering treatment on cardiovascular risk in hypertensive patients with type 2 diabetes. Diabetes Care. 2011;34:1270-1276.

3. Hermida RC, Ayala DE, Portaluppi F. Circadian variation of blood pressure: the basis for the chronotherapy of hypertension. Adv Drug Deliv Rev. 2007;59:904-922.

4. Verdecchia P, Porcellati C, Schillaci G, et al. Ambulatory blood pressure: an independent predictor of prognosis in essential hypertension. Hypertension. 1994;24:793-801.

5. Dolan E, Stanton A, Thijs L, et al. Superiority of ambulatory over clinic blood pressure measurement in predicting mortality: the Dublin study outcome. Hypertension. 2005;46:156-161.

6. Hermida RC, Ayala DE, Mojón A, et al. Decreasing sleep-time blood pressure determined by ambulatory monitoring reduces cardiovascular risk. J Am Coll Cardiol. 2011;58:1165-1173.

7. De la Sierra A, Redon J, Banegas JR, et al. Prevalence and factors associated with circadian blood pressure patterns in hypertensive patients. Hypertension. 2009;53:466-472.

8. Hermida RC, Ayala DE, Calvo C, et al. Chronotherapy of hypertension: administration-time-dependent effects of treatment on the circadian pattern of blood pressure. Adv Drug Deliv Rev. 2007;59:923-939.

References

1. Hermida RC, Ayala DE, Mojón A, et al. Influence of circadian time of hypertension treatment on cardiovascular risk: results of the MAPEC study. Chronobiol Int. 2010;27:1629-1651.

2. Hermida RC, Ayala DE, Mojón A, et al. Influence of time of day of blood pressure-lowering treatment on cardiovascular risk in hypertensive patients with type 2 diabetes. Diabetes Care. 2011;34:1270-1276.

3. Hermida RC, Ayala DE, Portaluppi F. Circadian variation of blood pressure: the basis for the chronotherapy of hypertension. Adv Drug Deliv Rev. 2007;59:904-922.

4. Verdecchia P, Porcellati C, Schillaci G, et al. Ambulatory blood pressure: an independent predictor of prognosis in essential hypertension. Hypertension. 1994;24:793-801.

5. Dolan E, Stanton A, Thijs L, et al. Superiority of ambulatory over clinic blood pressure measurement in predicting mortality: the Dublin study outcome. Hypertension. 2005;46:156-161.

6. Hermida RC, Ayala DE, Mojón A, et al. Decreasing sleep-time blood pressure determined by ambulatory monitoring reduces cardiovascular risk. J Am Coll Cardiol. 2011;58:1165-1173.

7. De la Sierra A, Redon J, Banegas JR, et al. Prevalence and factors associated with circadian blood pressure patterns in hypertensive patients. Hypertension. 2009;53:466-472.

8. Hermida RC, Ayala DE, Calvo C, et al. Chronotherapy of hypertension: administration-time-dependent effects of treatment on the circadian pattern of blood pressure. Adv Drug Deliv Rev. 2007;59:923-939.

Issue
The Journal of Family Practice - 61(3)
Issue
The Journal of Family Practice - 61(3)
Page Number
153-155
Page Number
153-155
Publications
Publications
Topics
Article Type
Display Headline
BP meds: This simple change improves outcomes
Display Headline
BP meds: This simple change improves outcomes
Legacy Keywords
Kate Kirley;MD; Umang Sharma;MD; Kate Rowland;MD; uncontrolled hypertension; bedtime dosing; blood pressure medications; BP meds; cardiovascular events; open-label RCT; CVD risk; morning dosing
Legacy Keywords
Kate Kirley;MD; Umang Sharma;MD; Kate Rowland;MD; uncontrolled hypertension; bedtime dosing; blood pressure medications; BP meds; cardiovascular events; open-label RCT; CVD risk; morning dosing
Sections
Article Source

PURLs Copyright

Copyright © 2012 The Family Physicians Inquiries Network.
All rights reserved.

Inside the Article

Article PDF Media

Combatting lice in a single treatment

Article Type
Changed
Mon, 01/14/2019 - 11:33
Display Headline
Combatting lice in a single treatment
PRACTICE CHANGER

Consider prescribing spinosad—a recently approved topical pediculicide that is more effective than permethrin and eliminates the need for nit combingfor children with head lice.1

Stough D, Shellabarger S, Quiring J, et al. Efficacy and safety of spinosad and permethrin crème rinses for pediculosis capitis (head lice). Pediatrics. 2009;124:e389-e395.

STRENGTH OF RECOMMENDATION

B: Based on a single randomized controlled trial (RCT).

 

ILLUSTRATIVE CASE

A 6-year-old girl is brought in by her mother, who has just been notified of an outbreak of head lice in the child’s school. You examine her scalp and observe viable eggs (nits) at the hair shaft, and tell mom that her daughter has head lice. What’s the most effective treatment?

Head lice are common and easily spread among schoolchildren—and a source of frustration for patients, parents, and family physicians alike. Permethrin cream 1% (Nix), one of the most widely used and well-studied pediculicides, is available over the counter.2

Prescription pediculicides, including malathion (Ovide) and lindane (Kwell), are used less frequently due to their side effect profiles: Malathion is highly flammable due to a high alcohol content and lindane can cause neurotoxicity.2 Both are typically reserved for cases resistant to permethrin 1% cream. Local resistance patterns influence physician prescribing, of course, and greater resistance has increased the need for multiple treatments.3

Dual treatment—and combing— are often required
Permethrin is pediculicidal (ie, it kills only live lice); it is not ovicidal (ie, it does not kill nits at every stage).4,5 Thus, a second application is usually needed 7 to 10 days after the first to ensure complete eradication. And, because permethrin is not ovicidal, nit combing—a tedious, time-consuming, and often painful processis required to remove lice and unhatched eggs from the hair shafts. Another downside: Children in districts that do not allow students to return to school until they are completely nit-free often miss school as a result of this lengthy process.

The US Food and Drug Administration (FDA) recently approved spinosad (Natroba topical suspension 0.9% cream rinse) for the treatment of lice in patients ≥4 years old.6 Spinosad, which is available only by prescription, eliminates the need for nit combing. But how well does it work?

STUDY SUMMARY: For most patients, a single application is sufficient

The study by Stough et al included 2 identical multicenter RCTs comparing 0.9% spinosad without nit combing vs 1% permethrin with combing in patients ≥6 months old who had active head lice. There were 446 patients in the primary treatment (spinosad) group and 470 in the control (permethrin) group. All participants within the same household were assigned to the same group.

Participants were evaluated on Days 1, 7, and 14. Those with active lice on Day 7 received another treatment (a second application with the same medication). The primary outcome was the percentage of participants who were lice free on Day 14.

 

 

 

The findings: After 14 days, 85% of the children in the spinosad group vs 44% of those in the permethrin group were lice free. In addition, 75% of those treated with spinosad required only one application for a cure, compared with 37% in the permethrin group.

There were no serious adverse events in either group. Participants in the permethrin group were more likely to experience scalp erythema than those in the spinosad group (6.8% vs 3.1%; P<.007). Complete metabolic panels and hematology studies were performed, and no significant differences pre- or posttreatment were found. The dropout rate was <10%, for similar reasons in both groups.

WHAT’S NEW: Extensive nit combing no longer needed

Spinosad is pediculicidal and ovicidal killing lice and their eggs in all stages—thereby eliminating the need for extensive combing in most cases. It has less resistance than current pediculicides, so a second course of treatment is needed only about one-quarter of the time. Spinosad received FDA approval for use in patients ≥4 years in 2011.

CAVEATS: Funding of study raises question of bias

The study by Stough et al was funded by ParaPRO, LLC, the manufacturer of Natroba. The company had access to the data and had a role in the interpretation of the findings and the decision to publish the study. However, the assessors were blinded to treatment group.

CHALLENGES TO iMPLEMENTATION: Cost is high and may not be covered

Cost will be a major barrier to treatment for many families. The average cost of an application of permethrin 1% is about $20; an application of spinosad costs $270 without insurance. Individual health plans may not cover it or may require prior authorization.

Acknowledgement

The PURLs Surveillance System is supported in part by Grant Number UL1RR024999 from the National Center for Research Resources, a Clinical Translational Science Award to the University of Chicago. The content is solely the responsibility of the authors and does not necessarily represent the official views of the National Center for Research Resources or the National Institutes of Health.

Click here to view PURL METHODOLOGY

References

1. Stough D, Shellabarger S, Quiring J, et al. Efficacy and Safety of spinosad and permethrin creme rinses for pediculosis capitis (head lice). Pediatrics. 2009;124:e389-e395.

2. Frankowski BL, Weiner LB. American Academy of Pediatrics, Committee on School Health Committee on Infectious Diseases. Head lice. Pediatrics. 2010;126:392-403.

3. Jones KN, English JC. 3rd. Review of common therapeutic options in the US for the treatment of pediculosis capitis. Clinl infect Dis. 2003;36:1355-1361.

4. Flinders D, Schweinitz P. Pediculosis and scabies. Am Fam Physician. 2004;69:341-350.

5. Centers for Disease Control and Prevention Parasites–lice. Available at http://www.cdc.gov/parasites/lice/. Accessed December 14, 2011.

6. US Food and Drug Administration Drugs@FDA. Available at: http://www.accessdata.fda.gov/scripts/cder/drugsatfda/index.cfm?fuseaction=Search. Drug Details. Accessed December 14, 2011.

Author and Disclosure Information

Dionna Brown, MD
The University of Chicago

Kate Rowland, MD
The University of Chicago

PURLs EDITOR
John Hickner, MD, MSc
Cleveland Clinic

Issue
The Journal of Family Practice - 61(1)
Publications
Topics
Page Number
41-42
Legacy Keywords
Dionna Brown;MD; Kate Rowland;MD; PURLs editor John Hickner;MD;MSc; combatting lice; spinosad; Natroba; topical pediculicide; permethrin cream; Ovide; Kwell; lindane; nit-picking; combing
Sections
Author and Disclosure Information

Dionna Brown, MD
The University of Chicago

Kate Rowland, MD
The University of Chicago

PURLs EDITOR
John Hickner, MD, MSc
Cleveland Clinic

Author and Disclosure Information

Dionna Brown, MD
The University of Chicago

Kate Rowland, MD
The University of Chicago

PURLs EDITOR
John Hickner, MD, MSc
Cleveland Clinic

PRACTICE CHANGER

Consider prescribing spinosad—a recently approved topical pediculicide that is more effective than permethrin and eliminates the need for nit combingfor children with head lice.1

Stough D, Shellabarger S, Quiring J, et al. Efficacy and safety of spinosad and permethrin crème rinses for pediculosis capitis (head lice). Pediatrics. 2009;124:e389-e395.

STRENGTH OF RECOMMENDATION

B: Based on a single randomized controlled trial (RCT).

 

ILLUSTRATIVE CASE

A 6-year-old girl is brought in by her mother, who has just been notified of an outbreak of head lice in the child’s school. You examine her scalp and observe viable eggs (nits) at the hair shaft, and tell mom that her daughter has head lice. What’s the most effective treatment?

Head lice are common and easily spread among schoolchildren—and a source of frustration for patients, parents, and family physicians alike. Permethrin cream 1% (Nix), one of the most widely used and well-studied pediculicides, is available over the counter.2

Prescription pediculicides, including malathion (Ovide) and lindane (Kwell), are used less frequently due to their side effect profiles: Malathion is highly flammable due to a high alcohol content and lindane can cause neurotoxicity.2 Both are typically reserved for cases resistant to permethrin 1% cream. Local resistance patterns influence physician prescribing, of course, and greater resistance has increased the need for multiple treatments.3

Dual treatment—and combing— are often required
Permethrin is pediculicidal (ie, it kills only live lice); it is not ovicidal (ie, it does not kill nits at every stage).4,5 Thus, a second application is usually needed 7 to 10 days after the first to ensure complete eradication. And, because permethrin is not ovicidal, nit combing—a tedious, time-consuming, and often painful processis required to remove lice and unhatched eggs from the hair shafts. Another downside: Children in districts that do not allow students to return to school until they are completely nit-free often miss school as a result of this lengthy process.

The US Food and Drug Administration (FDA) recently approved spinosad (Natroba topical suspension 0.9% cream rinse) for the treatment of lice in patients ≥4 years old.6 Spinosad, which is available only by prescription, eliminates the need for nit combing. But how well does it work?

STUDY SUMMARY: For most patients, a single application is sufficient

The study by Stough et al included 2 identical multicenter RCTs comparing 0.9% spinosad without nit combing vs 1% permethrin with combing in patients ≥6 months old who had active head lice. There were 446 patients in the primary treatment (spinosad) group and 470 in the control (permethrin) group. All participants within the same household were assigned to the same group.

Participants were evaluated on Days 1, 7, and 14. Those with active lice on Day 7 received another treatment (a second application with the same medication). The primary outcome was the percentage of participants who were lice free on Day 14.

 

 

 

The findings: After 14 days, 85% of the children in the spinosad group vs 44% of those in the permethrin group were lice free. In addition, 75% of those treated with spinosad required only one application for a cure, compared with 37% in the permethrin group.

There were no serious adverse events in either group. Participants in the permethrin group were more likely to experience scalp erythema than those in the spinosad group (6.8% vs 3.1%; P<.007). Complete metabolic panels and hematology studies were performed, and no significant differences pre- or posttreatment were found. The dropout rate was <10%, for similar reasons in both groups.

WHAT’S NEW: Extensive nit combing no longer needed

Spinosad is pediculicidal and ovicidal killing lice and their eggs in all stages—thereby eliminating the need for extensive combing in most cases. It has less resistance than current pediculicides, so a second course of treatment is needed only about one-quarter of the time. Spinosad received FDA approval for use in patients ≥4 years in 2011.

CAVEATS: Funding of study raises question of bias

The study by Stough et al was funded by ParaPRO, LLC, the manufacturer of Natroba. The company had access to the data and had a role in the interpretation of the findings and the decision to publish the study. However, the assessors were blinded to treatment group.

CHALLENGES TO iMPLEMENTATION: Cost is high and may not be covered

Cost will be a major barrier to treatment for many families. The average cost of an application of permethrin 1% is about $20; an application of spinosad costs $270 without insurance. Individual health plans may not cover it or may require prior authorization.

Acknowledgement

The PURLs Surveillance System is supported in part by Grant Number UL1RR024999 from the National Center for Research Resources, a Clinical Translational Science Award to the University of Chicago. The content is solely the responsibility of the authors and does not necessarily represent the official views of the National Center for Research Resources or the National Institutes of Health.

Click here to view PURL METHODOLOGY

PRACTICE CHANGER

Consider prescribing spinosad—a recently approved topical pediculicide that is more effective than permethrin and eliminates the need for nit combingfor children with head lice.1

Stough D, Shellabarger S, Quiring J, et al. Efficacy and safety of spinosad and permethrin crème rinses for pediculosis capitis (head lice). Pediatrics. 2009;124:e389-e395.

STRENGTH OF RECOMMENDATION

B: Based on a single randomized controlled trial (RCT).

 

ILLUSTRATIVE CASE

A 6-year-old girl is brought in by her mother, who has just been notified of an outbreak of head lice in the child’s school. You examine her scalp and observe viable eggs (nits) at the hair shaft, and tell mom that her daughter has head lice. What’s the most effective treatment?

Head lice are common and easily spread among schoolchildren—and a source of frustration for patients, parents, and family physicians alike. Permethrin cream 1% (Nix), one of the most widely used and well-studied pediculicides, is available over the counter.2

Prescription pediculicides, including malathion (Ovide) and lindane (Kwell), are used less frequently due to their side effect profiles: Malathion is highly flammable due to a high alcohol content and lindane can cause neurotoxicity.2 Both are typically reserved for cases resistant to permethrin 1% cream. Local resistance patterns influence physician prescribing, of course, and greater resistance has increased the need for multiple treatments.3

Dual treatment—and combing— are often required
Permethrin is pediculicidal (ie, it kills only live lice); it is not ovicidal (ie, it does not kill nits at every stage).4,5 Thus, a second application is usually needed 7 to 10 days after the first to ensure complete eradication. And, because permethrin is not ovicidal, nit combing—a tedious, time-consuming, and often painful processis required to remove lice and unhatched eggs from the hair shafts. Another downside: Children in districts that do not allow students to return to school until they are completely nit-free often miss school as a result of this lengthy process.

The US Food and Drug Administration (FDA) recently approved spinosad (Natroba topical suspension 0.9% cream rinse) for the treatment of lice in patients ≥4 years old.6 Spinosad, which is available only by prescription, eliminates the need for nit combing. But how well does it work?

STUDY SUMMARY: For most patients, a single application is sufficient

The study by Stough et al included 2 identical multicenter RCTs comparing 0.9% spinosad without nit combing vs 1% permethrin with combing in patients ≥6 months old who had active head lice. There were 446 patients in the primary treatment (spinosad) group and 470 in the control (permethrin) group. All participants within the same household were assigned to the same group.

Participants were evaluated on Days 1, 7, and 14. Those with active lice on Day 7 received another treatment (a second application with the same medication). The primary outcome was the percentage of participants who were lice free on Day 14.

 

 

 

The findings: After 14 days, 85% of the children in the spinosad group vs 44% of those in the permethrin group were lice free. In addition, 75% of those treated with spinosad required only one application for a cure, compared with 37% in the permethrin group.

There were no serious adverse events in either group. Participants in the permethrin group were more likely to experience scalp erythema than those in the spinosad group (6.8% vs 3.1%; P<.007). Complete metabolic panels and hematology studies were performed, and no significant differences pre- or posttreatment were found. The dropout rate was <10%, for similar reasons in both groups.

WHAT’S NEW: Extensive nit combing no longer needed

Spinosad is pediculicidal and ovicidal killing lice and their eggs in all stages—thereby eliminating the need for extensive combing in most cases. It has less resistance than current pediculicides, so a second course of treatment is needed only about one-quarter of the time. Spinosad received FDA approval for use in patients ≥4 years in 2011.

CAVEATS: Funding of study raises question of bias

The study by Stough et al was funded by ParaPRO, LLC, the manufacturer of Natroba. The company had access to the data and had a role in the interpretation of the findings and the decision to publish the study. However, the assessors were blinded to treatment group.

CHALLENGES TO iMPLEMENTATION: Cost is high and may not be covered

Cost will be a major barrier to treatment for many families. The average cost of an application of permethrin 1% is about $20; an application of spinosad costs $270 without insurance. Individual health plans may not cover it or may require prior authorization.

Acknowledgement

The PURLs Surveillance System is supported in part by Grant Number UL1RR024999 from the National Center for Research Resources, a Clinical Translational Science Award to the University of Chicago. The content is solely the responsibility of the authors and does not necessarily represent the official views of the National Center for Research Resources or the National Institutes of Health.

Click here to view PURL METHODOLOGY

References

1. Stough D, Shellabarger S, Quiring J, et al. Efficacy and Safety of spinosad and permethrin creme rinses for pediculosis capitis (head lice). Pediatrics. 2009;124:e389-e395.

2. Frankowski BL, Weiner LB. American Academy of Pediatrics, Committee on School Health Committee on Infectious Diseases. Head lice. Pediatrics. 2010;126:392-403.

3. Jones KN, English JC. 3rd. Review of common therapeutic options in the US for the treatment of pediculosis capitis. Clinl infect Dis. 2003;36:1355-1361.

4. Flinders D, Schweinitz P. Pediculosis and scabies. Am Fam Physician. 2004;69:341-350.

5. Centers for Disease Control and Prevention Parasites–lice. Available at http://www.cdc.gov/parasites/lice/. Accessed December 14, 2011.

6. US Food and Drug Administration Drugs@FDA. Available at: http://www.accessdata.fda.gov/scripts/cder/drugsatfda/index.cfm?fuseaction=Search. Drug Details. Accessed December 14, 2011.

References

1. Stough D, Shellabarger S, Quiring J, et al. Efficacy and Safety of spinosad and permethrin creme rinses for pediculosis capitis (head lice). Pediatrics. 2009;124:e389-e395.

2. Frankowski BL, Weiner LB. American Academy of Pediatrics, Committee on School Health Committee on Infectious Diseases. Head lice. Pediatrics. 2010;126:392-403.

3. Jones KN, English JC. 3rd. Review of common therapeutic options in the US for the treatment of pediculosis capitis. Clinl infect Dis. 2003;36:1355-1361.

4. Flinders D, Schweinitz P. Pediculosis and scabies. Am Fam Physician. 2004;69:341-350.

5. Centers for Disease Control and Prevention Parasites–lice. Available at http://www.cdc.gov/parasites/lice/. Accessed December 14, 2011.

6. US Food and Drug Administration Drugs@FDA. Available at: http://www.accessdata.fda.gov/scripts/cder/drugsatfda/index.cfm?fuseaction=Search. Drug Details. Accessed December 14, 2011.

Issue
The Journal of Family Practice - 61(1)
Issue
The Journal of Family Practice - 61(1)
Page Number
41-42
Page Number
41-42
Publications
Publications
Topics
Article Type
Display Headline
Combatting lice in a single treatment
Display Headline
Combatting lice in a single treatment
Legacy Keywords
Dionna Brown;MD; Kate Rowland;MD; PURLs editor John Hickner;MD;MSc; combatting lice; spinosad; Natroba; topical pediculicide; permethrin cream; Ovide; Kwell; lindane; nit-picking; combing
Legacy Keywords
Dionna Brown;MD; Kate Rowland;MD; PURLs editor John Hickner;MD;MSc; combatting lice; spinosad; Natroba; topical pediculicide; permethrin cream; Ovide; Kwell; lindane; nit-picking; combing
Sections
PURLs Copyright

Copyright © 2012 The Family Physicians Inquiries Network.
All rights reserved.

Disallow All Ads
Alternative CME

Zinc for the common cold—not if, but when

Article Type
Changed
Mon, 01/14/2019 - 11:32
Display Headline
Zinc for the common cold—not if, but when
PRACTICE CHANGER

Advise patients to start taking zinc supplements (available as tablets, syrup, or lozenges) within 24 hours of the onset of a cold.1

Singh M, Das RR. Zinc for the common cold. Cochrane Database Syst Rev. 2011;(2):CD001364.

STRENGTH OF RECOMMENDATION

A: Based on a meta-analysis of 13 randomized placebo-controlled trials.

 

ILLUSTRATIVE CASE

A healthy 31-year-old woman schedules a visit because of congestion and rhinorrhea. She tells you that her cold symptoms developed in the last day or 2 and asks for something to speed her recovery. What can you suggest?

Americans experience roughly 500 million episodes of noninfluenza-related viral respiratory tract infections annually (ie, common colds), or roughly 2.5 episodes per person each year. The total economic burden of the common cold is nearly $40 billion annually.2

Most cold remedies offer little relief
We’ve all cared for patients with cold symptoms. And despite the desire to help, many of us have been frustrated by the fact that there is no cure for the common cold, and most over-the-counter therapies offer little or no relief.

What about zinc? Zinc has been in and out of favor as a treatment for the common cold, based on contradictory findings, since the first randomized trial was published in 1984.3

A 1998 systematic review (which did not include a quantitative synthesis of data) suggested that zinc was beneficial in reducing the duration and severity of cold symptoms.4 A meta-analysis published in 2000 found that zinc was ineffective compared with placebo in reducing the likelihood that cold symptoms were present after 7 days.5 A meta-analysis published in June 2011 concluded that zinc lozenges reduce the duration of cold symptoms by 12% to 48%, but only at daily doses >75 mg.6

These conflicting results have made it difficult to recommend zinc to patients—until now.

STUDY SUMMARY: Zinc for colds? Cochrane review provides convincing evidence

The 2011 Cochrane review that’s the focus of this PURL1 addressed the question: Does zinc reduce the duration and severity of the common cold? The review also assessed zinc’s ability to prevent colds, although only 2 studies focused on prevention.

The researchers included only randomized, double-blind, placebo-controlled trials in which oral zinc supplementation was used for ≥5 consecutive days for treatment or ≥5 months for prevention. Studies included patients of any age, taking any dosage and any formulation of zinc.

Primary outcomes included the duration of symptoms, the severity of symptoms, and the incidence of the common cold in prevention studies. Secondary outcomes included the proportion of patients symptomatic after 3, 5, and 7 days of treatment, the time to resolution of individual symptoms (eg, cough), change in individual symptom scores, school absences, antibiotic use, and adverse effects.

Thirteen therapeutic trials and 2 prevention trials met the inclusion criteria. These studies, all of which were judged to be of high quality with a low risk of bias, had a total of 996 patients in the therapeutic trials and 394 patients in the prevention trials. Participants ranged in age from 1 to 65 years.

Therapeutic trials. In general, therapeutic studies included lozenges containing 10 to 24 mg zinc gluconate, with one lozenge taken every 1 to 4 hours during the day for 3 to 7 days. In one therapeutic trial that enrolled only children ages 1 to 10, the intervention group took zinc sulfate syrup (15 mg bid) for 10 days. In 11 of the 13 therapeutic studies, treatment began in the first 24 hours of symptoms; in the remaining 2, it began within 48 hours of symptoms.

Ten studies reported on duration of symptoms; 6 of them were similar enough to allow for pooling of results. The pooled results showed that patients who took zinc had a shorter duration of cold symptoms (0.97 days; 95% confidence interval [CI], -1.56 to -0.38), compared with those on placebo. Pooled results from 5 trials revealed that zinc significantly reduced the severity of symptoms by a standard effect size of 0.39 (95% CI, -0.77 to -0.02), which is considered a small to moderate effect.

 

 

 

Prevention trials. In one prevention trial, which included only children ages 6½ to 10 years, those in the intervention group took one 10-mg zinc sulfate tablet 6 days a week for 5 months. In the other trial, children in the intervention group took 15 mg zinc sulfate syrup daily for 7 months.

Pooled results from these 2 studies revealed that daily zinc supplementation substantially reduced the incidence of colds. The incident rate ratio (the number of children who developed colds while taking zinc compared with the number who developed colds while on placebo) was 0.64 (95% CI, 0.47-0.88). In the original trials, one study found a difference of 0.5 colds (1.7 in the control group vs 1.2 in the intervention group) per season, and the other found a difference of 1.8 colds per season (3.15 in the control group vs 1.37 in the intervention group).

WHAT’s NEW: Evidence of zinc’s cold relief properties is conclusive

This Cochrane review provides convincing evidence from 13 randomized placebo-controlled trials that taking zinc soon after the onset of symptoms of the common cold significantly reduces both the duration and severity of symptoms. Zinc supplements are widely available over the counter, so you can recommend that patients take zinc the next time they develop a cold.

In addition, 2 prevention trials found that zinc can reduce the incidence of colds in children, whether it is taken as a syrup or in tablet form. There have been few trials of zinc for prophylaxis of the common cold, and no previous meta-analyses included preventive studies.7 This Cochrane review substantiates the effectiveness of zinc for prophylaxis of the common cold in young children.

However, children need to take zinc daily for prophylaxis, which may be inconvenient. Long-term safety information is not yet available. Given these considerations, parents may choose to wait for additional evidence about safety before considering daily prophylaxis.

CAVEATS: Adverse effects, long-term use may create problems

In this meta-analysis, side effects from zinc were common. The 2 most frequently reported were bad taste (pooled odds ratio [OR], 2.64; 95% CI, 1.91-3.64) and nausea (pooled OR, 2.15; 95% CI, 1.44-3.23). When you recommend zinc, warn your patients about these adverse effects. The side effects are not severe, so patients can decide for themselves whether the benefit of a reduction in cold duration is worth the downside of nausea and a bad taste in the mouth.

It is also important to note that the trials included in the meta-analysis enrolled healthy children and adults ≤65 years old. Whether zinc benefits people with chronic illnesses (eg, chronic obstructive pulmonary disease) who develop colds is unknown.

Prolonged elevated serum zinc levels can interfere with copper metabolism, and the adverse effects of long-term use of zinc as prophylaxis are unknown. The trials included in the meta-analysis took place in relatively affluent countries in which zinc deficiency is uncommon. It is not known what impact zinc supplementation would have on people in poor countries.

Of the 15 studies included in the meta-analysis, 10 received support from pharmaceutical companies, 4 received support from foundations, and one received support from both.

CHALLENGES TO IMPLEMENTATION: When to talk to patients about zinc

Most patients do not seek medical care for colds. Those who do typically present only after having symptoms for several days, and it is not clear whether zinc supplementation has the same beneficial effects when started after the first 24 hours.2

Thus, you may have few opportunities in the office to recommend zinc for patients with colds, for whom there is evidence of immediate benefit. More likely, you’ll need to incorporate a zinc recommendation into your overall advice about colds.

Zinc is available over the counter in various forms and dosage. After recommending zinc, you may be confronted with the question of which dose, brand, and formulation is best—a question which, unfortunately, remains unanswered.

Acknowledgement

The PURLs Surveillance System is supported in part by Grant Number UL1RR024999 from the National Center For Research Resources, a Clinical Translational Science Award to the University of Chicago. The content is solely the responsibility of the authors and does not necessarily represent the official views of the National Center for Research Resources or the National Institutes of Health.

Click here to view PURL METHODOLOGY

References

1. Singh M, Das RR. Zinc for the common cold. Cochrane Database Syst Rev. 2011;(2):CD001364.-

2. Fendrick AM, Monto AS, Nightengale B, et al. The economic burden of non-influenza-related viral respiratory tract infection in the United States. Arch Intern Med. 2003;163:487-494.

3. Eby GA, Davis DR, Halcomb WW. Reduction in duration of common colds by zinc gluconate lozenges in a double-blind study. Antimicrob Agents Chemother. 1984;25:20-24.

4. Marshall S. Zinc gluconate and the common cold. Review of randomized controlled trials. Can Fam Physician. 1998;44:1037-1042.

5. Jackson JL, Lesho E, Peterson C. Zinc and the common cold: a meta-analysis revisited. J Nutr. 2000;130 (5 suppl):1512S-1515S.

6. Hemila H. Zinc lozenges may shorten the duration of colds: a systematic review. Open Respir Med J. 2011;5:51-58.

7. Singh M, Das RR. Clinical potential of zinc in prophylaxis of the common cold. Exp Rev Resp Med. 2011;5:301-303.

Article PDF
Author and Disclosure Information

Goutham Rao, MD
Department of Family, Medicine, The University of Chicago

Kate Rowland, MD
Department of Family, Medicine, The University of Chicago

PURLs EDITOR
John Hickner, MD, MSc
Cleveland Clinic

Issue
The Journal of Family Practice - 60(11)
Publications
Topics
Page Number
669-671
Legacy Keywords
Goutham Rao;MD; Kate Rowland;MD; John Hickner;MD;MSc; zinc supplementation; duration and severity; common cold; within 24 hours; zinc for colds; 2011 Cochrane review; oral zinc; zinc lozenges; zinc tablets; zinc syrup; symptom scores; school absences; therapeutic trials; zinc gluconate; zinc sulfate; prevention trials; pooled results;
Sections
Author and Disclosure Information

Goutham Rao, MD
Department of Family, Medicine, The University of Chicago

Kate Rowland, MD
Department of Family, Medicine, The University of Chicago

PURLs EDITOR
John Hickner, MD, MSc
Cleveland Clinic

Author and Disclosure Information

Goutham Rao, MD
Department of Family, Medicine, The University of Chicago

Kate Rowland, MD
Department of Family, Medicine, The University of Chicago

PURLs EDITOR
John Hickner, MD, MSc
Cleveland Clinic

Article PDF
Article PDF
PRACTICE CHANGER

Advise patients to start taking zinc supplements (available as tablets, syrup, or lozenges) within 24 hours of the onset of a cold.1

Singh M, Das RR. Zinc for the common cold. Cochrane Database Syst Rev. 2011;(2):CD001364.

STRENGTH OF RECOMMENDATION

A: Based on a meta-analysis of 13 randomized placebo-controlled trials.

 

ILLUSTRATIVE CASE

A healthy 31-year-old woman schedules a visit because of congestion and rhinorrhea. She tells you that her cold symptoms developed in the last day or 2 and asks for something to speed her recovery. What can you suggest?

Americans experience roughly 500 million episodes of noninfluenza-related viral respiratory tract infections annually (ie, common colds), or roughly 2.5 episodes per person each year. The total economic burden of the common cold is nearly $40 billion annually.2

Most cold remedies offer little relief
We’ve all cared for patients with cold symptoms. And despite the desire to help, many of us have been frustrated by the fact that there is no cure for the common cold, and most over-the-counter therapies offer little or no relief.

What about zinc? Zinc has been in and out of favor as a treatment for the common cold, based on contradictory findings, since the first randomized trial was published in 1984.3

A 1998 systematic review (which did not include a quantitative synthesis of data) suggested that zinc was beneficial in reducing the duration and severity of cold symptoms.4 A meta-analysis published in 2000 found that zinc was ineffective compared with placebo in reducing the likelihood that cold symptoms were present after 7 days.5 A meta-analysis published in June 2011 concluded that zinc lozenges reduce the duration of cold symptoms by 12% to 48%, but only at daily doses >75 mg.6

These conflicting results have made it difficult to recommend zinc to patients—until now.

STUDY SUMMARY: Zinc for colds? Cochrane review provides convincing evidence

The 2011 Cochrane review that’s the focus of this PURL1 addressed the question: Does zinc reduce the duration and severity of the common cold? The review also assessed zinc’s ability to prevent colds, although only 2 studies focused on prevention.

The researchers included only randomized, double-blind, placebo-controlled trials in which oral zinc supplementation was used for ≥5 consecutive days for treatment or ≥5 months for prevention. Studies included patients of any age, taking any dosage and any formulation of zinc.

Primary outcomes included the duration of symptoms, the severity of symptoms, and the incidence of the common cold in prevention studies. Secondary outcomes included the proportion of patients symptomatic after 3, 5, and 7 days of treatment, the time to resolution of individual symptoms (eg, cough), change in individual symptom scores, school absences, antibiotic use, and adverse effects.

Thirteen therapeutic trials and 2 prevention trials met the inclusion criteria. These studies, all of which were judged to be of high quality with a low risk of bias, had a total of 996 patients in the therapeutic trials and 394 patients in the prevention trials. Participants ranged in age from 1 to 65 years.

Therapeutic trials. In general, therapeutic studies included lozenges containing 10 to 24 mg zinc gluconate, with one lozenge taken every 1 to 4 hours during the day for 3 to 7 days. In one therapeutic trial that enrolled only children ages 1 to 10, the intervention group took zinc sulfate syrup (15 mg bid) for 10 days. In 11 of the 13 therapeutic studies, treatment began in the first 24 hours of symptoms; in the remaining 2, it began within 48 hours of symptoms.

Ten studies reported on duration of symptoms; 6 of them were similar enough to allow for pooling of results. The pooled results showed that patients who took zinc had a shorter duration of cold symptoms (0.97 days; 95% confidence interval [CI], -1.56 to -0.38), compared with those on placebo. Pooled results from 5 trials revealed that zinc significantly reduced the severity of symptoms by a standard effect size of 0.39 (95% CI, -0.77 to -0.02), which is considered a small to moderate effect.

 

 

 

Prevention trials. In one prevention trial, which included only children ages 6½ to 10 years, those in the intervention group took one 10-mg zinc sulfate tablet 6 days a week for 5 months. In the other trial, children in the intervention group took 15 mg zinc sulfate syrup daily for 7 months.

Pooled results from these 2 studies revealed that daily zinc supplementation substantially reduced the incidence of colds. The incident rate ratio (the number of children who developed colds while taking zinc compared with the number who developed colds while on placebo) was 0.64 (95% CI, 0.47-0.88). In the original trials, one study found a difference of 0.5 colds (1.7 in the control group vs 1.2 in the intervention group) per season, and the other found a difference of 1.8 colds per season (3.15 in the control group vs 1.37 in the intervention group).

WHAT’s NEW: Evidence of zinc’s cold relief properties is conclusive

This Cochrane review provides convincing evidence from 13 randomized placebo-controlled trials that taking zinc soon after the onset of symptoms of the common cold significantly reduces both the duration and severity of symptoms. Zinc supplements are widely available over the counter, so you can recommend that patients take zinc the next time they develop a cold.

In addition, 2 prevention trials found that zinc can reduce the incidence of colds in children, whether it is taken as a syrup or in tablet form. There have been few trials of zinc for prophylaxis of the common cold, and no previous meta-analyses included preventive studies.7 This Cochrane review substantiates the effectiveness of zinc for prophylaxis of the common cold in young children.

However, children need to take zinc daily for prophylaxis, which may be inconvenient. Long-term safety information is not yet available. Given these considerations, parents may choose to wait for additional evidence about safety before considering daily prophylaxis.

CAVEATS: Adverse effects, long-term use may create problems

In this meta-analysis, side effects from zinc were common. The 2 most frequently reported were bad taste (pooled odds ratio [OR], 2.64; 95% CI, 1.91-3.64) and nausea (pooled OR, 2.15; 95% CI, 1.44-3.23). When you recommend zinc, warn your patients about these adverse effects. The side effects are not severe, so patients can decide for themselves whether the benefit of a reduction in cold duration is worth the downside of nausea and a bad taste in the mouth.

It is also important to note that the trials included in the meta-analysis enrolled healthy children and adults ≤65 years old. Whether zinc benefits people with chronic illnesses (eg, chronic obstructive pulmonary disease) who develop colds is unknown.

Prolonged elevated serum zinc levels can interfere with copper metabolism, and the adverse effects of long-term use of zinc as prophylaxis are unknown. The trials included in the meta-analysis took place in relatively affluent countries in which zinc deficiency is uncommon. It is not known what impact zinc supplementation would have on people in poor countries.

Of the 15 studies included in the meta-analysis, 10 received support from pharmaceutical companies, 4 received support from foundations, and one received support from both.

CHALLENGES TO IMPLEMENTATION: When to talk to patients about zinc

Most patients do not seek medical care for colds. Those who do typically present only after having symptoms for several days, and it is not clear whether zinc supplementation has the same beneficial effects when started after the first 24 hours.2

Thus, you may have few opportunities in the office to recommend zinc for patients with colds, for whom there is evidence of immediate benefit. More likely, you’ll need to incorporate a zinc recommendation into your overall advice about colds.

Zinc is available over the counter in various forms and dosage. After recommending zinc, you may be confronted with the question of which dose, brand, and formulation is best—a question which, unfortunately, remains unanswered.

Acknowledgement

The PURLs Surveillance System is supported in part by Grant Number UL1RR024999 from the National Center For Research Resources, a Clinical Translational Science Award to the University of Chicago. The content is solely the responsibility of the authors and does not necessarily represent the official views of the National Center for Research Resources or the National Institutes of Health.

Click here to view PURL METHODOLOGY

PRACTICE CHANGER

Advise patients to start taking zinc supplements (available as tablets, syrup, or lozenges) within 24 hours of the onset of a cold.1

Singh M, Das RR. Zinc for the common cold. Cochrane Database Syst Rev. 2011;(2):CD001364.

STRENGTH OF RECOMMENDATION

A: Based on a meta-analysis of 13 randomized placebo-controlled trials.

 

ILLUSTRATIVE CASE

A healthy 31-year-old woman schedules a visit because of congestion and rhinorrhea. She tells you that her cold symptoms developed in the last day or 2 and asks for something to speed her recovery. What can you suggest?

Americans experience roughly 500 million episodes of noninfluenza-related viral respiratory tract infections annually (ie, common colds), or roughly 2.5 episodes per person each year. The total economic burden of the common cold is nearly $40 billion annually.2

Most cold remedies offer little relief
We’ve all cared for patients with cold symptoms. And despite the desire to help, many of us have been frustrated by the fact that there is no cure for the common cold, and most over-the-counter therapies offer little or no relief.

What about zinc? Zinc has been in and out of favor as a treatment for the common cold, based on contradictory findings, since the first randomized trial was published in 1984.3

A 1998 systematic review (which did not include a quantitative synthesis of data) suggested that zinc was beneficial in reducing the duration and severity of cold symptoms.4 A meta-analysis published in 2000 found that zinc was ineffective compared with placebo in reducing the likelihood that cold symptoms were present after 7 days.5 A meta-analysis published in June 2011 concluded that zinc lozenges reduce the duration of cold symptoms by 12% to 48%, but only at daily doses >75 mg.6

These conflicting results have made it difficult to recommend zinc to patients—until now.

STUDY SUMMARY: Zinc for colds? Cochrane review provides convincing evidence

The 2011 Cochrane review that’s the focus of this PURL1 addressed the question: Does zinc reduce the duration and severity of the common cold? The review also assessed zinc’s ability to prevent colds, although only 2 studies focused on prevention.

The researchers included only randomized, double-blind, placebo-controlled trials in which oral zinc supplementation was used for ≥5 consecutive days for treatment or ≥5 months for prevention. Studies included patients of any age, taking any dosage and any formulation of zinc.

Primary outcomes included the duration of symptoms, the severity of symptoms, and the incidence of the common cold in prevention studies. Secondary outcomes included the proportion of patients symptomatic after 3, 5, and 7 days of treatment, the time to resolution of individual symptoms (eg, cough), change in individual symptom scores, school absences, antibiotic use, and adverse effects.

Thirteen therapeutic trials and 2 prevention trials met the inclusion criteria. These studies, all of which were judged to be of high quality with a low risk of bias, had a total of 996 patients in the therapeutic trials and 394 patients in the prevention trials. Participants ranged in age from 1 to 65 years.

Therapeutic trials. In general, therapeutic studies included lozenges containing 10 to 24 mg zinc gluconate, with one lozenge taken every 1 to 4 hours during the day for 3 to 7 days. In one therapeutic trial that enrolled only children ages 1 to 10, the intervention group took zinc sulfate syrup (15 mg bid) for 10 days. In 11 of the 13 therapeutic studies, treatment began in the first 24 hours of symptoms; in the remaining 2, it began within 48 hours of symptoms.

Ten studies reported on duration of symptoms; 6 of them were similar enough to allow for pooling of results. The pooled results showed that patients who took zinc had a shorter duration of cold symptoms (0.97 days; 95% confidence interval [CI], -1.56 to -0.38), compared with those on placebo. Pooled results from 5 trials revealed that zinc significantly reduced the severity of symptoms by a standard effect size of 0.39 (95% CI, -0.77 to -0.02), which is considered a small to moderate effect.

 

 

 

Prevention trials. In one prevention trial, which included only children ages 6½ to 10 years, those in the intervention group took one 10-mg zinc sulfate tablet 6 days a week for 5 months. In the other trial, children in the intervention group took 15 mg zinc sulfate syrup daily for 7 months.

Pooled results from these 2 studies revealed that daily zinc supplementation substantially reduced the incidence of colds. The incident rate ratio (the number of children who developed colds while taking zinc compared with the number who developed colds while on placebo) was 0.64 (95% CI, 0.47-0.88). In the original trials, one study found a difference of 0.5 colds (1.7 in the control group vs 1.2 in the intervention group) per season, and the other found a difference of 1.8 colds per season (3.15 in the control group vs 1.37 in the intervention group).

WHAT’s NEW: Evidence of zinc’s cold relief properties is conclusive

This Cochrane review provides convincing evidence from 13 randomized placebo-controlled trials that taking zinc soon after the onset of symptoms of the common cold significantly reduces both the duration and severity of symptoms. Zinc supplements are widely available over the counter, so you can recommend that patients take zinc the next time they develop a cold.

In addition, 2 prevention trials found that zinc can reduce the incidence of colds in children, whether it is taken as a syrup or in tablet form. There have been few trials of zinc for prophylaxis of the common cold, and no previous meta-analyses included preventive studies.7 This Cochrane review substantiates the effectiveness of zinc for prophylaxis of the common cold in young children.

However, children need to take zinc daily for prophylaxis, which may be inconvenient. Long-term safety information is not yet available. Given these considerations, parents may choose to wait for additional evidence about safety before considering daily prophylaxis.

CAVEATS: Adverse effects, long-term use may create problems

In this meta-analysis, side effects from zinc were common. The 2 most frequently reported were bad taste (pooled odds ratio [OR], 2.64; 95% CI, 1.91-3.64) and nausea (pooled OR, 2.15; 95% CI, 1.44-3.23). When you recommend zinc, warn your patients about these adverse effects. The side effects are not severe, so patients can decide for themselves whether the benefit of a reduction in cold duration is worth the downside of nausea and a bad taste in the mouth.

It is also important to note that the trials included in the meta-analysis enrolled healthy children and adults ≤65 years old. Whether zinc benefits people with chronic illnesses (eg, chronic obstructive pulmonary disease) who develop colds is unknown.

Prolonged elevated serum zinc levels can interfere with copper metabolism, and the adverse effects of long-term use of zinc as prophylaxis are unknown. The trials included in the meta-analysis took place in relatively affluent countries in which zinc deficiency is uncommon. It is not known what impact zinc supplementation would have on people in poor countries.

Of the 15 studies included in the meta-analysis, 10 received support from pharmaceutical companies, 4 received support from foundations, and one received support from both.

CHALLENGES TO IMPLEMENTATION: When to talk to patients about zinc

Most patients do not seek medical care for colds. Those who do typically present only after having symptoms for several days, and it is not clear whether zinc supplementation has the same beneficial effects when started after the first 24 hours.2

Thus, you may have few opportunities in the office to recommend zinc for patients with colds, for whom there is evidence of immediate benefit. More likely, you’ll need to incorporate a zinc recommendation into your overall advice about colds.

Zinc is available over the counter in various forms and dosage. After recommending zinc, you may be confronted with the question of which dose, brand, and formulation is best—a question which, unfortunately, remains unanswered.

Acknowledgement

The PURLs Surveillance System is supported in part by Grant Number UL1RR024999 from the National Center For Research Resources, a Clinical Translational Science Award to the University of Chicago. The content is solely the responsibility of the authors and does not necessarily represent the official views of the National Center for Research Resources or the National Institutes of Health.

Click here to view PURL METHODOLOGY

References

1. Singh M, Das RR. Zinc for the common cold. Cochrane Database Syst Rev. 2011;(2):CD001364.-

2. Fendrick AM, Monto AS, Nightengale B, et al. The economic burden of non-influenza-related viral respiratory tract infection in the United States. Arch Intern Med. 2003;163:487-494.

3. Eby GA, Davis DR, Halcomb WW. Reduction in duration of common colds by zinc gluconate lozenges in a double-blind study. Antimicrob Agents Chemother. 1984;25:20-24.

4. Marshall S. Zinc gluconate and the common cold. Review of randomized controlled trials. Can Fam Physician. 1998;44:1037-1042.

5. Jackson JL, Lesho E, Peterson C. Zinc and the common cold: a meta-analysis revisited. J Nutr. 2000;130 (5 suppl):1512S-1515S.

6. Hemila H. Zinc lozenges may shorten the duration of colds: a systematic review. Open Respir Med J. 2011;5:51-58.

7. Singh M, Das RR. Clinical potential of zinc in prophylaxis of the common cold. Exp Rev Resp Med. 2011;5:301-303.

References

1. Singh M, Das RR. Zinc for the common cold. Cochrane Database Syst Rev. 2011;(2):CD001364.-

2. Fendrick AM, Monto AS, Nightengale B, et al. The economic burden of non-influenza-related viral respiratory tract infection in the United States. Arch Intern Med. 2003;163:487-494.

3. Eby GA, Davis DR, Halcomb WW. Reduction in duration of common colds by zinc gluconate lozenges in a double-blind study. Antimicrob Agents Chemother. 1984;25:20-24.

4. Marshall S. Zinc gluconate and the common cold. Review of randomized controlled trials. Can Fam Physician. 1998;44:1037-1042.

5. Jackson JL, Lesho E, Peterson C. Zinc and the common cold: a meta-analysis revisited. J Nutr. 2000;130 (5 suppl):1512S-1515S.

6. Hemila H. Zinc lozenges may shorten the duration of colds: a systematic review. Open Respir Med J. 2011;5:51-58.

7. Singh M, Das RR. Clinical potential of zinc in prophylaxis of the common cold. Exp Rev Resp Med. 2011;5:301-303.

Issue
The Journal of Family Practice - 60(11)
Issue
The Journal of Family Practice - 60(11)
Page Number
669-671
Page Number
669-671
Publications
Publications
Topics
Article Type
Display Headline
Zinc for the common cold—not if, but when
Display Headline
Zinc for the common cold—not if, but when
Legacy Keywords
Goutham Rao;MD; Kate Rowland;MD; John Hickner;MD;MSc; zinc supplementation; duration and severity; common cold; within 24 hours; zinc for colds; 2011 Cochrane review; oral zinc; zinc lozenges; zinc tablets; zinc syrup; symptom scores; school absences; therapeutic trials; zinc gluconate; zinc sulfate; prevention trials; pooled results;
Legacy Keywords
Goutham Rao;MD; Kate Rowland;MD; John Hickner;MD;MSc; zinc supplementation; duration and severity; common cold; within 24 hours; zinc for colds; 2011 Cochrane review; oral zinc; zinc lozenges; zinc tablets; zinc syrup; symptom scores; school absences; therapeutic trials; zinc gluconate; zinc sulfate; prevention trials; pooled results;
Sections
PURLs Copyright

Copyright © 2011 The Family Physicians Inquiries Network.
All rights reserved.

Disallow All Ads
Alternative CME
Article PDF Media

Consider this option for heavy menstrual bleeding

Article Type
Changed
Mon, 01/14/2019 - 11:31
Display Headline
Consider this option for heavy menstrual bleeding
PRACTICE CHANGER

Offer tranexamic acid to patients with heavy menstrual bleeding. The extended-release formulation is effective and well tolerated.1

STRENGTH OF RECOMMENDATION

A: Based on 1 good-quality randomized controlled trial (RCT).

Lukes AS, Moore KA, Muse KN, et al. Tranexamic acid treatment for heavy menstrual bleeding: a randomized controlled trial. Obstet Gynecol. 2010;116:865-875.

 

ILLUSTRATIVE CASE

A 32-year-old woman comes to your office complaining of heavy, but regular, menstrual bleeding, accompanied by clots and heavy cramping that often leave her drained and unable to work. She has taken oral contraceptives in the past, but they caused nausea, and nonsteroidal anti-inflammatory drugs (NSAIDs) did not provide adequate symptom relief.

A pelvic ultrasound shows that the patient has normal endometrial thickness and no fibroids. Aside from surgical intervention or the placement of a hormonal intrauterine device (IUD), what can you offer her?

Heavy menstrual bleeding is a frequent problem, common enough to cause an estimated 10% to 30% of women of reproductive age to seek treatment.2-4 Often the bleeding is severe enough to adversely affect the patient’s social, physical, and emotional well-being.

Adverse effects, variable efficacy limit use of other treatments
Quantitatively defined as blood loss ≥80 mL per cycle, heavy menstrual bleeding can also be diagnosed based on a patient’s perception of menstrual blood loss and its effect on her daily life.5,6 NSAIDs, hormonal medications, the placement of a hormonal IUD, and surgical procedures are all treatment options, but potential adverse effects, contraindications, personal preference, and variable efficacy can limit their use.7

The fibrinolysis-blood loss link
Fibrinolytic activity in menstrual blood, leading to increasing blood loss, has prompted the evaluation of hemostatic agents as potential therapeutic options.8 Oral tranexamic acid decreases fibrinolysis, thereby reducing menstrual blood loss;9 however, gastrointestinal (GI) side effects limit the usefulness of immediate-release tranexamic acid.9,10

This formulation of tranexamic acid has been used in Europe for heavy menstrual bleeding. A Cochrane review published in 2000 included 4 studies that compared immediate-release tranexamic acid therapy with placebo. The meta-analysis found a significant reduction in mean blood loss compared with placebo (weighted mean difference [WMD]=-94.0 mL; 95% confidence interval [CI], -151.4 to -36.5) and a significant change in mean reduction of blood loss (WMD=-110.2 mL; 95% CI, -146.5 to -73.8) compared with baseline in the treatment group. However, only one of the studies measured perceived improvement in monthly menstrual blood loss, and its sample size was inadequate to provide a precise estimate of the effect (relative risk [RR] 2.5; 95% CI, 0.9-7.3).11

An extended-release option. Oral extended-release (ER) tranexamic acid (Lysteda), approved by the US Food and Drug Administration in 2009,12 reduces blood loss with fewer GI effects than immediate-release tranexamic acid. In the RCT detailed below, Lukes et al assessed the efficacy and safety of this new formulation.

STUDY SUMMARY: ER formulation reduces blood loss, boosts quality of life

The researchers conducted a multicenter, randomized, double-blind placebo-controlled study comparing the effect of ER tranexamic acid on reduction of menstrual blood flow compared with placebo.1 Reduction in menstrual blood loss >50 mL and a reduction in menstrual blood loss ≥36 mL (an amount previously established to be perceived as meaningful to women) were related primary outcomes. Improvements in limitations in social or leisure and physical activities and in self-perceived menstrual blood loss were secondary outcomes.

 

 

 

Study participants were women ages 18 to 49 years who had heavy menstrual bleeding, a normal pelvic exam, and a normal transvaginal ultrasound; current use of a nonhormonal birth control method was also required. Women with fibroids were not excluded unless surgery was planned. Exclusion criteria included significant coagulation issues, endocrinopathy, ocular disease, pregnancy or lactation, endometrial abnormalities, cervical cancer, anovulatory dysfunctional uterine bleeding, metrorrhagia, menometrorrhagia, and polymenorrhea.

Participants were randomized to receive either tranexamic acid 1.3 g by mouth 3 times a day for 5 days per menstrual cycle, beginning with the onset of heavy bleeding, or a matched placebo. The use of anticoagulants or NSAIDs during the menstrual period was not permitted.

Heavy bleeding was defined as ≥60 mL of blood loss in one measured cycle and an average ≥80 mL of blood loss over 2 measured cycles.

Mean reduction in blood loss per cycle over 6 cycles was 70 mL (a 40.4% reduction) in the active treatment group vs 13 mL (an 8.2% reduction) in the placebo group (P<.001). The proportion of women with a ≥50% reduction from baseline in blood loss was greater in the tranexamic acid group compared with the placebo group (35% vs 7%; P<.001), yielding a number needed to treat of 4. The mean reduction in perceived blood loss was also greater in the treatment group, but the difference was not statistically significant.

The researchers used a validated menstrual quality-of-life scale that measured social and physical quality of life using a 5-point Likert scale. Women treated with tranexamic acid had a mean reduction of 0.89 points from baseline on the social and leisure activity question, compared with a mean reduction of 0.38 points for those in the placebo group. On the physical activity question, those in the tranexamic acid group had a mean reduction of 0.90 points from baseline, vs a mean decline of 0.35 points in the placebo group.

These findings indicate that the women who received tranexamic acid experienced significantly fewer limitations in social and physical activities. Responses to a question about limitations in work activities showed that the treatment group had significant improvements there, as well. The ER form of tranexamic acid used in the study was well tolerated, with no significant differences in adverse effects between the intervention and control groups.

WHAT’S NEW: Women with heavy menstrual bleeding have a new option

The ER formulation of tranexamic acid used in the study does not appear to have the GI side effects associated with the immediate-release formula.

Tranexamic acid is taken only during the menstrual cycle and does not interfere with ovulation. Thus, it can be used by women who desire fertility but are troubled by heavy bleeding.

CAVEATS: Questions about related conditions, use with hormones remain

The study included women with regular heavy menstrual periods (menorrhagia) and therefore may not be applicable to those with irregular heavy periods or anovulatory, dysfunctional uterine bleeding. In clinical practice, these conditions may overlap, but the safety and efficacy of tranexamic acid in such cases is unclear.

Another caveat, at least theoretically, is that research to date has neither identified nor excluded the possibility that tranexamic acid with concomitant use of hormonal agents might increase the risk of thrombotic events.13 This risk is low based on evidence to date, but the theoretical uncertainty leads us to be cautious about the combination of tranexamic acid and hormonal therapy for long-term use.

Xanodyne Pharmaceuticals (which manufactured Lysteda) and Ferring Pharmaceuticals (its current owner) were major sponsors of this study. While we cannot recognize any source of bias as a result of sponsorship, the independence of the investigators in publishing the findings was not clearly stated, so it is possible that future independent studies would contradict these findings.

CHALLENGES TO IMPLEMENTATION: The treatment is costly

Lysteda is expensive, costing about $170 for 30 tablets of 650 mg each.14 Cost aside, ER tranexamic acid appears to be safe, with no major barriers to its use.

Acknowledgement

The PURLs Surveillance System is supported in part by Grant Number UL1RR024999 from the National Center for Research Resources, a Clinical Translational Science Award to the University of Chicago. The content is solely the responsibility of the authors and does not necessarily represent the official views of the National Center for Research Resources or the National Institutes of Health.

Click here to view PURL METHODOLOGY

References

1. Lukes AS, Moore KA, Muse KN, et al. Tranexamic acid treatment for heavy menstrual bleeding: a randomized controlled trial. Obstet Gynecol. 2010;116:865-875.

2. Coulter A, Peto V, Jenkinson C. Quality of life and patient satisfaction following treatment for menorrhagia. Fam Pract. 1994;11:394-401.

3. Liu Z, Doan QV, Blumenthal P, et al. A systematic review evaluating health-related quality of life, work impairment, and health-care costs and utilization in abnormal uterine bleeding. Value Health. 2007;10:183-194.

4. Shapley M, Jordan K, Croft PR. Increased vaginal bleeding: the reasons women give for consulting primary care. J Obstet Gynaecol. 2003;23:48-50.

5. Rees M. Menorrhagia. BMJ Clin Res Ed. 1987;294:759-762.

6. Wyatt KM, Dimmock PW, Walker TJ, et al. Determination of total menstrual blood loss. Fertil Steril. 2001;76:125-131.

7. Hurskainen R, Grenman S, Komi I, et al. Diagnosis and treatment of menorrhagia. Acta Obstet Gynecol Scand. 2007;86:749-757.

8. Dockeray CJ, Sheppard BL, Daly L, et al. The fibrinolytic enzyme system in normal menstruation and excessive uterine bleeding and the effect of tranexamic acid. Eur J Obstet Gynecol Reprod Biol. 1987;24:309-318.

9. Wellington K, Wagstaff AJ. Tranexamic acid: a review of its use in the management of menorrhagia. Drugs. 2003;63:1417-1433.

10. Winkler UH. The effect of tranexamic acid on the quality of life of women with heavy menstrual bleeding. Eur J Obstet Gynecol Reprod Biol. 2001;99:238-243.

11. Lethaby A, Farquhar C, Cooke I. Antifibrinolytics for heavy menstrual bleeding. Cochrane Database Syst Rev. 2000;(4):CD000249.-

12. US Food and Drug Administration. FDA approves Lysteda to treat heavy menstrual bleeding. November 13, 2009. Available at: http://www.fda.gov/NewsEvents/Newsroom/Press Announcements/2009/ucm190551.htm. Accessed June 14, 2011.

13. Berntorp E, Floorud C, Lethagen S. No increased risk of venous thrombosis in women taking tranexamic acid. Thromb Haemost. 2001;86:714-715.

14. Lysteda Drugstore.com. Available at: http://www.drugstore.com/lysteda/650mg-tablets/qxn66479065001. Accessed June 17, 2011.

Article PDF
Author and Disclosure Information

Robert Levy, MD
North Memorial Family Medicine Residency, University of Minnesota, Minneapolis

Shailendra Prasad, MBBS, MPH
North Memorial Family Medicine Residency, University of Minnesota, Minneapolis

Kate Rowland, MD
Department of Family Medicine, University of Chicago

PURLs EDITOR
Bernard Ewigman, MD, MSPH
Department of Family Medicine, University of Chicago

Issue
The Journal of Family Practice - 60(7)
Publications
Topics
Page Number
410-412
Legacy Keywords
Robert Levy;MD; heavy menstrual bleeding; tranexamic acid; extended-release; fibrinolysis; fibrinolytic;
Sections
Author and Disclosure Information

Robert Levy, MD
North Memorial Family Medicine Residency, University of Minnesota, Minneapolis

Shailendra Prasad, MBBS, MPH
North Memorial Family Medicine Residency, University of Minnesota, Minneapolis

Kate Rowland, MD
Department of Family Medicine, University of Chicago

PURLs EDITOR
Bernard Ewigman, MD, MSPH
Department of Family Medicine, University of Chicago

Author and Disclosure Information

Robert Levy, MD
North Memorial Family Medicine Residency, University of Minnesota, Minneapolis

Shailendra Prasad, MBBS, MPH
North Memorial Family Medicine Residency, University of Minnesota, Minneapolis

Kate Rowland, MD
Department of Family Medicine, University of Chicago

PURLs EDITOR
Bernard Ewigman, MD, MSPH
Department of Family Medicine, University of Chicago

Article PDF
Article PDF
PRACTICE CHANGER

Offer tranexamic acid to patients with heavy menstrual bleeding. The extended-release formulation is effective and well tolerated.1

STRENGTH OF RECOMMENDATION

A: Based on 1 good-quality randomized controlled trial (RCT).

Lukes AS, Moore KA, Muse KN, et al. Tranexamic acid treatment for heavy menstrual bleeding: a randomized controlled trial. Obstet Gynecol. 2010;116:865-875.

 

ILLUSTRATIVE CASE

A 32-year-old woman comes to your office complaining of heavy, but regular, menstrual bleeding, accompanied by clots and heavy cramping that often leave her drained and unable to work. She has taken oral contraceptives in the past, but they caused nausea, and nonsteroidal anti-inflammatory drugs (NSAIDs) did not provide adequate symptom relief.

A pelvic ultrasound shows that the patient has normal endometrial thickness and no fibroids. Aside from surgical intervention or the placement of a hormonal intrauterine device (IUD), what can you offer her?

Heavy menstrual bleeding is a frequent problem, common enough to cause an estimated 10% to 30% of women of reproductive age to seek treatment.2-4 Often the bleeding is severe enough to adversely affect the patient’s social, physical, and emotional well-being.

Adverse effects, variable efficacy limit use of other treatments
Quantitatively defined as blood loss ≥80 mL per cycle, heavy menstrual bleeding can also be diagnosed based on a patient’s perception of menstrual blood loss and its effect on her daily life.5,6 NSAIDs, hormonal medications, the placement of a hormonal IUD, and surgical procedures are all treatment options, but potential adverse effects, contraindications, personal preference, and variable efficacy can limit their use.7

The fibrinolysis-blood loss link
Fibrinolytic activity in menstrual blood, leading to increasing blood loss, has prompted the evaluation of hemostatic agents as potential therapeutic options.8 Oral tranexamic acid decreases fibrinolysis, thereby reducing menstrual blood loss;9 however, gastrointestinal (GI) side effects limit the usefulness of immediate-release tranexamic acid.9,10

This formulation of tranexamic acid has been used in Europe for heavy menstrual bleeding. A Cochrane review published in 2000 included 4 studies that compared immediate-release tranexamic acid therapy with placebo. The meta-analysis found a significant reduction in mean blood loss compared with placebo (weighted mean difference [WMD]=-94.0 mL; 95% confidence interval [CI], -151.4 to -36.5) and a significant change in mean reduction of blood loss (WMD=-110.2 mL; 95% CI, -146.5 to -73.8) compared with baseline in the treatment group. However, only one of the studies measured perceived improvement in monthly menstrual blood loss, and its sample size was inadequate to provide a precise estimate of the effect (relative risk [RR] 2.5; 95% CI, 0.9-7.3).11

An extended-release option. Oral extended-release (ER) tranexamic acid (Lysteda), approved by the US Food and Drug Administration in 2009,12 reduces blood loss with fewer GI effects than immediate-release tranexamic acid. In the RCT detailed below, Lukes et al assessed the efficacy and safety of this new formulation.

STUDY SUMMARY: ER formulation reduces blood loss, boosts quality of life

The researchers conducted a multicenter, randomized, double-blind placebo-controlled study comparing the effect of ER tranexamic acid on reduction of menstrual blood flow compared with placebo.1 Reduction in menstrual blood loss >50 mL and a reduction in menstrual blood loss ≥36 mL (an amount previously established to be perceived as meaningful to women) were related primary outcomes. Improvements in limitations in social or leisure and physical activities and in self-perceived menstrual blood loss were secondary outcomes.

 

 

 

Study participants were women ages 18 to 49 years who had heavy menstrual bleeding, a normal pelvic exam, and a normal transvaginal ultrasound; current use of a nonhormonal birth control method was also required. Women with fibroids were not excluded unless surgery was planned. Exclusion criteria included significant coagulation issues, endocrinopathy, ocular disease, pregnancy or lactation, endometrial abnormalities, cervical cancer, anovulatory dysfunctional uterine bleeding, metrorrhagia, menometrorrhagia, and polymenorrhea.

Participants were randomized to receive either tranexamic acid 1.3 g by mouth 3 times a day for 5 days per menstrual cycle, beginning with the onset of heavy bleeding, or a matched placebo. The use of anticoagulants or NSAIDs during the menstrual period was not permitted.

Heavy bleeding was defined as ≥60 mL of blood loss in one measured cycle and an average ≥80 mL of blood loss over 2 measured cycles.

Mean reduction in blood loss per cycle over 6 cycles was 70 mL (a 40.4% reduction) in the active treatment group vs 13 mL (an 8.2% reduction) in the placebo group (P<.001). The proportion of women with a ≥50% reduction from baseline in blood loss was greater in the tranexamic acid group compared with the placebo group (35% vs 7%; P<.001), yielding a number needed to treat of 4. The mean reduction in perceived blood loss was also greater in the treatment group, but the difference was not statistically significant.

The researchers used a validated menstrual quality-of-life scale that measured social and physical quality of life using a 5-point Likert scale. Women treated with tranexamic acid had a mean reduction of 0.89 points from baseline on the social and leisure activity question, compared with a mean reduction of 0.38 points for those in the placebo group. On the physical activity question, those in the tranexamic acid group had a mean reduction of 0.90 points from baseline, vs a mean decline of 0.35 points in the placebo group.

These findings indicate that the women who received tranexamic acid experienced significantly fewer limitations in social and physical activities. Responses to a question about limitations in work activities showed that the treatment group had significant improvements there, as well. The ER form of tranexamic acid used in the study was well tolerated, with no significant differences in adverse effects between the intervention and control groups.

WHAT’S NEW: Women with heavy menstrual bleeding have a new option

The ER formulation of tranexamic acid used in the study does not appear to have the GI side effects associated with the immediate-release formula.

Tranexamic acid is taken only during the menstrual cycle and does not interfere with ovulation. Thus, it can be used by women who desire fertility but are troubled by heavy bleeding.

CAVEATS: Questions about related conditions, use with hormones remain

The study included women with regular heavy menstrual periods (menorrhagia) and therefore may not be applicable to those with irregular heavy periods or anovulatory, dysfunctional uterine bleeding. In clinical practice, these conditions may overlap, but the safety and efficacy of tranexamic acid in such cases is unclear.

Another caveat, at least theoretically, is that research to date has neither identified nor excluded the possibility that tranexamic acid with concomitant use of hormonal agents might increase the risk of thrombotic events.13 This risk is low based on evidence to date, but the theoretical uncertainty leads us to be cautious about the combination of tranexamic acid and hormonal therapy for long-term use.

Xanodyne Pharmaceuticals (which manufactured Lysteda) and Ferring Pharmaceuticals (its current owner) were major sponsors of this study. While we cannot recognize any source of bias as a result of sponsorship, the independence of the investigators in publishing the findings was not clearly stated, so it is possible that future independent studies would contradict these findings.

CHALLENGES TO IMPLEMENTATION: The treatment is costly

Lysteda is expensive, costing about $170 for 30 tablets of 650 mg each.14 Cost aside, ER tranexamic acid appears to be safe, with no major barriers to its use.

Acknowledgement

The PURLs Surveillance System is supported in part by Grant Number UL1RR024999 from the National Center for Research Resources, a Clinical Translational Science Award to the University of Chicago. The content is solely the responsibility of the authors and does not necessarily represent the official views of the National Center for Research Resources or the National Institutes of Health.

Click here to view PURL METHODOLOGY

PRACTICE CHANGER

Offer tranexamic acid to patients with heavy menstrual bleeding. The extended-release formulation is effective and well tolerated.1

STRENGTH OF RECOMMENDATION

A: Based on 1 good-quality randomized controlled trial (RCT).

Lukes AS, Moore KA, Muse KN, et al. Tranexamic acid treatment for heavy menstrual bleeding: a randomized controlled trial. Obstet Gynecol. 2010;116:865-875.

 

ILLUSTRATIVE CASE

A 32-year-old woman comes to your office complaining of heavy, but regular, menstrual bleeding, accompanied by clots and heavy cramping that often leave her drained and unable to work. She has taken oral contraceptives in the past, but they caused nausea, and nonsteroidal anti-inflammatory drugs (NSAIDs) did not provide adequate symptom relief.

A pelvic ultrasound shows that the patient has normal endometrial thickness and no fibroids. Aside from surgical intervention or the placement of a hormonal intrauterine device (IUD), what can you offer her?

Heavy menstrual bleeding is a frequent problem, common enough to cause an estimated 10% to 30% of women of reproductive age to seek treatment.2-4 Often the bleeding is severe enough to adversely affect the patient’s social, physical, and emotional well-being.

Adverse effects, variable efficacy limit use of other treatments
Quantitatively defined as blood loss ≥80 mL per cycle, heavy menstrual bleeding can also be diagnosed based on a patient’s perception of menstrual blood loss and its effect on her daily life.5,6 NSAIDs, hormonal medications, the placement of a hormonal IUD, and surgical procedures are all treatment options, but potential adverse effects, contraindications, personal preference, and variable efficacy can limit their use.7

The fibrinolysis-blood loss link
Fibrinolytic activity in menstrual blood, leading to increasing blood loss, has prompted the evaluation of hemostatic agents as potential therapeutic options.8 Oral tranexamic acid decreases fibrinolysis, thereby reducing menstrual blood loss;9 however, gastrointestinal (GI) side effects limit the usefulness of immediate-release tranexamic acid.9,10

This formulation of tranexamic acid has been used in Europe for heavy menstrual bleeding. A Cochrane review published in 2000 included 4 studies that compared immediate-release tranexamic acid therapy with placebo. The meta-analysis found a significant reduction in mean blood loss compared with placebo (weighted mean difference [WMD]=-94.0 mL; 95% confidence interval [CI], -151.4 to -36.5) and a significant change in mean reduction of blood loss (WMD=-110.2 mL; 95% CI, -146.5 to -73.8) compared with baseline in the treatment group. However, only one of the studies measured perceived improvement in monthly menstrual blood loss, and its sample size was inadequate to provide a precise estimate of the effect (relative risk [RR] 2.5; 95% CI, 0.9-7.3).11

An extended-release option. Oral extended-release (ER) tranexamic acid (Lysteda), approved by the US Food and Drug Administration in 2009,12 reduces blood loss with fewer GI effects than immediate-release tranexamic acid. In the RCT detailed below, Lukes et al assessed the efficacy and safety of this new formulation.

STUDY SUMMARY: ER formulation reduces blood loss, boosts quality of life

The researchers conducted a multicenter, randomized, double-blind placebo-controlled study comparing the effect of ER tranexamic acid on reduction of menstrual blood flow compared with placebo.1 Reduction in menstrual blood loss >50 mL and a reduction in menstrual blood loss ≥36 mL (an amount previously established to be perceived as meaningful to women) were related primary outcomes. Improvements in limitations in social or leisure and physical activities and in self-perceived menstrual blood loss were secondary outcomes.

 

 

 

Study participants were women ages 18 to 49 years who had heavy menstrual bleeding, a normal pelvic exam, and a normal transvaginal ultrasound; current use of a nonhormonal birth control method was also required. Women with fibroids were not excluded unless surgery was planned. Exclusion criteria included significant coagulation issues, endocrinopathy, ocular disease, pregnancy or lactation, endometrial abnormalities, cervical cancer, anovulatory dysfunctional uterine bleeding, metrorrhagia, menometrorrhagia, and polymenorrhea.

Participants were randomized to receive either tranexamic acid 1.3 g by mouth 3 times a day for 5 days per menstrual cycle, beginning with the onset of heavy bleeding, or a matched placebo. The use of anticoagulants or NSAIDs during the menstrual period was not permitted.

Heavy bleeding was defined as ≥60 mL of blood loss in one measured cycle and an average ≥80 mL of blood loss over 2 measured cycles.

Mean reduction in blood loss per cycle over 6 cycles was 70 mL (a 40.4% reduction) in the active treatment group vs 13 mL (an 8.2% reduction) in the placebo group (P<.001). The proportion of women with a ≥50% reduction from baseline in blood loss was greater in the tranexamic acid group compared with the placebo group (35% vs 7%; P<.001), yielding a number needed to treat of 4. The mean reduction in perceived blood loss was also greater in the treatment group, but the difference was not statistically significant.

The researchers used a validated menstrual quality-of-life scale that measured social and physical quality of life using a 5-point Likert scale. Women treated with tranexamic acid had a mean reduction of 0.89 points from baseline on the social and leisure activity question, compared with a mean reduction of 0.38 points for those in the placebo group. On the physical activity question, those in the tranexamic acid group had a mean reduction of 0.90 points from baseline, vs a mean decline of 0.35 points in the placebo group.

These findings indicate that the women who received tranexamic acid experienced significantly fewer limitations in social and physical activities. Responses to a question about limitations in work activities showed that the treatment group had significant improvements there, as well. The ER form of tranexamic acid used in the study was well tolerated, with no significant differences in adverse effects between the intervention and control groups.

WHAT’S NEW: Women with heavy menstrual bleeding have a new option

The ER formulation of tranexamic acid used in the study does not appear to have the GI side effects associated with the immediate-release formula.

Tranexamic acid is taken only during the menstrual cycle and does not interfere with ovulation. Thus, it can be used by women who desire fertility but are troubled by heavy bleeding.

CAVEATS: Questions about related conditions, use with hormones remain

The study included women with regular heavy menstrual periods (menorrhagia) and therefore may not be applicable to those with irregular heavy periods or anovulatory, dysfunctional uterine bleeding. In clinical practice, these conditions may overlap, but the safety and efficacy of tranexamic acid in such cases is unclear.

Another caveat, at least theoretically, is that research to date has neither identified nor excluded the possibility that tranexamic acid with concomitant use of hormonal agents might increase the risk of thrombotic events.13 This risk is low based on evidence to date, but the theoretical uncertainty leads us to be cautious about the combination of tranexamic acid and hormonal therapy for long-term use.

Xanodyne Pharmaceuticals (which manufactured Lysteda) and Ferring Pharmaceuticals (its current owner) were major sponsors of this study. While we cannot recognize any source of bias as a result of sponsorship, the independence of the investigators in publishing the findings was not clearly stated, so it is possible that future independent studies would contradict these findings.

CHALLENGES TO IMPLEMENTATION: The treatment is costly

Lysteda is expensive, costing about $170 for 30 tablets of 650 mg each.14 Cost aside, ER tranexamic acid appears to be safe, with no major barriers to its use.

Acknowledgement

The PURLs Surveillance System is supported in part by Grant Number UL1RR024999 from the National Center for Research Resources, a Clinical Translational Science Award to the University of Chicago. The content is solely the responsibility of the authors and does not necessarily represent the official views of the National Center for Research Resources or the National Institutes of Health.

Click here to view PURL METHODOLOGY

References

1. Lukes AS, Moore KA, Muse KN, et al. Tranexamic acid treatment for heavy menstrual bleeding: a randomized controlled trial. Obstet Gynecol. 2010;116:865-875.

2. Coulter A, Peto V, Jenkinson C. Quality of life and patient satisfaction following treatment for menorrhagia. Fam Pract. 1994;11:394-401.

3. Liu Z, Doan QV, Blumenthal P, et al. A systematic review evaluating health-related quality of life, work impairment, and health-care costs and utilization in abnormal uterine bleeding. Value Health. 2007;10:183-194.

4. Shapley M, Jordan K, Croft PR. Increased vaginal bleeding: the reasons women give for consulting primary care. J Obstet Gynaecol. 2003;23:48-50.

5. Rees M. Menorrhagia. BMJ Clin Res Ed. 1987;294:759-762.

6. Wyatt KM, Dimmock PW, Walker TJ, et al. Determination of total menstrual blood loss. Fertil Steril. 2001;76:125-131.

7. Hurskainen R, Grenman S, Komi I, et al. Diagnosis and treatment of menorrhagia. Acta Obstet Gynecol Scand. 2007;86:749-757.

8. Dockeray CJ, Sheppard BL, Daly L, et al. The fibrinolytic enzyme system in normal menstruation and excessive uterine bleeding and the effect of tranexamic acid. Eur J Obstet Gynecol Reprod Biol. 1987;24:309-318.

9. Wellington K, Wagstaff AJ. Tranexamic acid: a review of its use in the management of menorrhagia. Drugs. 2003;63:1417-1433.

10. Winkler UH. The effect of tranexamic acid on the quality of life of women with heavy menstrual bleeding. Eur J Obstet Gynecol Reprod Biol. 2001;99:238-243.

11. Lethaby A, Farquhar C, Cooke I. Antifibrinolytics for heavy menstrual bleeding. Cochrane Database Syst Rev. 2000;(4):CD000249.-

12. US Food and Drug Administration. FDA approves Lysteda to treat heavy menstrual bleeding. November 13, 2009. Available at: http://www.fda.gov/NewsEvents/Newsroom/Press Announcements/2009/ucm190551.htm. Accessed June 14, 2011.

13. Berntorp E, Floorud C, Lethagen S. No increased risk of venous thrombosis in women taking tranexamic acid. Thromb Haemost. 2001;86:714-715.

14. Lysteda Drugstore.com. Available at: http://www.drugstore.com/lysteda/650mg-tablets/qxn66479065001. Accessed June 17, 2011.

References

1. Lukes AS, Moore KA, Muse KN, et al. Tranexamic acid treatment for heavy menstrual bleeding: a randomized controlled trial. Obstet Gynecol. 2010;116:865-875.

2. Coulter A, Peto V, Jenkinson C. Quality of life and patient satisfaction following treatment for menorrhagia. Fam Pract. 1994;11:394-401.

3. Liu Z, Doan QV, Blumenthal P, et al. A systematic review evaluating health-related quality of life, work impairment, and health-care costs and utilization in abnormal uterine bleeding. Value Health. 2007;10:183-194.

4. Shapley M, Jordan K, Croft PR. Increased vaginal bleeding: the reasons women give for consulting primary care. J Obstet Gynaecol. 2003;23:48-50.

5. Rees M. Menorrhagia. BMJ Clin Res Ed. 1987;294:759-762.

6. Wyatt KM, Dimmock PW, Walker TJ, et al. Determination of total menstrual blood loss. Fertil Steril. 2001;76:125-131.

7. Hurskainen R, Grenman S, Komi I, et al. Diagnosis and treatment of menorrhagia. Acta Obstet Gynecol Scand. 2007;86:749-757.

8. Dockeray CJ, Sheppard BL, Daly L, et al. The fibrinolytic enzyme system in normal menstruation and excessive uterine bleeding and the effect of tranexamic acid. Eur J Obstet Gynecol Reprod Biol. 1987;24:309-318.

9. Wellington K, Wagstaff AJ. Tranexamic acid: a review of its use in the management of menorrhagia. Drugs. 2003;63:1417-1433.

10. Winkler UH. The effect of tranexamic acid on the quality of life of women with heavy menstrual bleeding. Eur J Obstet Gynecol Reprod Biol. 2001;99:238-243.

11. Lethaby A, Farquhar C, Cooke I. Antifibrinolytics for heavy menstrual bleeding. Cochrane Database Syst Rev. 2000;(4):CD000249.-

12. US Food and Drug Administration. FDA approves Lysteda to treat heavy menstrual bleeding. November 13, 2009. Available at: http://www.fda.gov/NewsEvents/Newsroom/Press Announcements/2009/ucm190551.htm. Accessed June 14, 2011.

13. Berntorp E, Floorud C, Lethagen S. No increased risk of venous thrombosis in women taking tranexamic acid. Thromb Haemost. 2001;86:714-715.

14. Lysteda Drugstore.com. Available at: http://www.drugstore.com/lysteda/650mg-tablets/qxn66479065001. Accessed June 17, 2011.

Issue
The Journal of Family Practice - 60(7)
Issue
The Journal of Family Practice - 60(7)
Page Number
410-412
Page Number
410-412
Publications
Publications
Topics
Article Type
Display Headline
Consider this option for heavy menstrual bleeding
Display Headline
Consider this option for heavy menstrual bleeding
Legacy Keywords
Robert Levy;MD; heavy menstrual bleeding; tranexamic acid; extended-release; fibrinolysis; fibrinolytic;
Legacy Keywords
Robert Levy;MD; heavy menstrual bleeding; tranexamic acid; extended-release; fibrinolysis; fibrinolytic;
Sections
PURLs Copyright

Copyright © 2011 The Family Physicians Inquiries Network.
All rights reserved.

Disallow All Ads
Alternative CME
Article PDF Media

Colicky baby? Here’s a surprising remedy

Article Type
Changed
Mon, 01/14/2019 - 11:38
Display Headline
Colicky baby? Here’s a surprising remedy
PRACTICE CHANGER

Suggest that parents of colicky breastfed infants try probiotics (Lactobacillus reuteri), which can significantly reduce daily crying time with no adverse effects.1

STRENGTH OF RECOMMENDATION:

A: A good-quality randomized controlled trial (RCT).

Savino F, Cordisco L, Tarasco V, et al. Lactobacillus reuteri DSM 17938 in infantile colic: a randomized, double-blind, placebo-controlled trial. Pediatrics. 2010;126:e526-e533.

 

ILLUSTRATIVE CASE

The parents of an otherwise healthy 10-week-old breastfed baby girl bring her in for the second time in 2 weeks because of her in-consolable crying. Physical examination and work-up remain normal, and you again diagnose colic. What can you suggest to help decrease the baby’s crying, other than the usual dietary advice?

Colic affects up to 28% of infants, causing considerable stress for parents and for their health care providers.2 Indeed, in the first 3 months of a baby’s life, crying is the No. 1 reason for pediatric visits.3 Parents often perceive—incorrectly—that the inconsolable crying is either a sign of serious illness or a result of poor parenting skills.4

A distressing problem, with few remedies
Despite the ubiquity and frustration that accompany colic, its exact etiology remains unclear and effective treatments remain elusive. With very little quality evidence to support interventions for colicky infants, we often have nothing more than grandmotherly advice to offer parents of babies with this vexing condition.

Current guidelines recommend only one strategy for breastfeeding mothers: a lowallergen diet.5 However, recent studies suggest that low counts of intestinal lactobacilli may play a role in colic and have documented improved symptoms after treatment with lactobacilli compared with treatment with simethicone.6-8 Infant formulas that contain probiotics are now available, as a result.

Although the results of the recent studies have been promising, they were not double-blinded or placebo-controlled. The study detailed here is the first to provide compelling evidence for a safe intervention for colicky breastfed infants.

STUDY SUMMARY: Lactobacilli cut crying time

In a randomized, double-blinded, placebo-controlled trial, Savino et al randomly assigned 50 exclusively breastfed colicky infants ages 2 to 16 weeks to receive either L reuteri DSM 17 938 (108 colony-forming units) or placebo daily for 21 days. Diagnosis of colic was made according to the modified Wessel’s criteria—fussy crying for ≥3 hours per day for ≥3 days per week in the week before enrollment. Their mothers were told to avoid ingesting cow’s milk during the course of the study.

Term infants adequate for gestational age were eligible for inclusion in the study. Exclusion criteria included evidence of chronic illness or gastrointestinal disorders, any intake of probiotics and/or antibiotics in the week preceding recruitment, and any formula feeding.

Parents and providers were blinded during the study, and they reported daily crying time, stool characteristics, adverse events, and growth patterns. An identical looking and tasting triglyceride oil without live bacteria was used in the placebo group. Each infant received 5 drops of L reuteri or placebo each morning 30 minutes before the morning feeding.

The primary outcome was a reduction of average crying time to <3 hours a day by Day 21. A secondary outcome was the number of infants in each group who experienced a 50% decrease in the daily average crying time from baseline on Days 7, 14, and 21 of the study.

Initially, the babies were divided equally between the control and intervention groups, but 4 participants in the control group were later excluded from analysis (1 had fever, 1 had reflux, and the parents of 2 infants did not complete the analysis).

At the time of enrollment, no significant differences were noted between the intervention and control groups regarding type of delivery, sex, age, family history of gastrointestinal disorders, growth parameters, and median daily crying time (370 minutes for the probiotics group vs 300 minutes for the placebo group).

 

 

 

By Day 21, the number of infants with crying times >3 hours was significantly lower in the treatment group compared with the placebo group (4 vs 12, respectively; P=.009). At all stages in the study, crying time for those in the treatment group was less than in the placebo group; median crying times for the intervention group were 95, 60, and 35 minutes per day, vs 185, 150, and 90 minutes for the controls, at 7, 14, and 21 days, respectively.

The number of infants with a 50% reduction in crying time was significantly greater in the treatment group than in the placebo group on Days 7, 14, and 21 (TABLE), although both groups saw an increase in the number of children whose average crying time had dropped by 50% as time went by. The number needed to treat to reduce crying time by 50% on Day 21 was 4.

There were no differences between the groups in growth, weight gain, frequency of stools, or incidence of regurgitation or constipation. No adverse events related to the treatment were reported.

TABLE
Babies respond* to probiotics

Day of studyL reuteri n=25 (%)Placebo n=21 (%)P value
720 (80)8 (36).006
1424 (96)13 (62).007
2124 (96)15 (71).036
*An infant with a decrease in daily average crying time of 50% from baseline was defined as a responder.

WHAT’S NEW: We have an evidence-based remedy that’s safe and effective

This study represents the first randomized, double-blinded, placebo-controlled investigation of probiotics to reduce infant colic. The researchers’ focus on patient-oriented outcomes and their solid study design move the notion of probiotics’ efficacy from conjecture to evidence. Furthermore, the study documents the safety of the intervention in the treatment group. This study increases our evidence-based armamentarium for treating colic, and family physicians should consider prescribing probiotics for healthy breastfed infants with colic.

CAVEATS: Will it work for bottle-fed infants?

This study was conducted in exclusively breast-fed, healthy infants whose mothers avoided dietary cow’s milk, which limits its applicability to a more general infant population. The study was funded by the makers of the probiotic, but the rigorous study design with random allocation, double-blind design, and intention-to-treat analysis makes bias unlikely. Although no adverse effects were reported during this study, there is little available evidence about the long-term effects of probiotics in infants. As L reuteri are naturally occurring gut bacteria, however, it seems unlikely that it would be harmful in the long term.

CHALLENGES TO IMPLEMENTATION: Parents will need to purchase the probiotics

As with any non–FDA-regulated product, it will be important to guide patients toward reputable manufacturers to ensure homogeneity of dosing. A 29-day supply of BioGaia probiotic drops (100 million units once a day), which costs $37 according to the manufacturer’s Web site, http://www.biogaia.com/consumer/biogaia-probiotic-products/probiotic-drops, should be affordable for most parents. Otherwise, little stands in the way of using this therapy to reduce the crying and subsequent stress associated with infant colic.

Acknowledgement

The PURLs Surveillance System is supported in part by Grant Number UL1RR024999 from the National Center for Research Resources, a Clinical Translational Science Award to the University of Chicago. The content is solely the responsibility of the authors and does not necessarily represent the official views of the National Center for Research Resources or the National Institutes of Health.

Click here to view PURL METHODOLOGY

References

1. Savino F, Cordisco L, Tarasco V, et al. Lactobacillus reuteri DSM 17938 in infantile colic: a randomized, double-blind, placebo-controlled trial. Pediatrics. 2010;126:e526-e533.

2. Keefe MR, Kajrlsen KA, Lobo ML, et al. Reducing parenting stress in families with irritable infants. Nurs Res. 2006;55:198-205

3. Forsyth BWC, McCarthy PL, Leventhal JM. Problems of early infancy, formula changes, and mothers’ beliefs about their infants. J Pediatr. 1985;106:1012-1017.

4. Lehtonen LA, Rautava PT. Infantile colic: natural history and treatment. Curr Probl Pediatr. 1996;26:79.-

5. Cincinnati Children’s Hospital Medical Center. Best evidence statement (BESt). Maternal dietary antigen avoidance in lactation. Available at: http://www.cincinnatichildrens.org/svc/alpha/h/health-policy/best.htm. Accessed December 14, 2010.

6. Savino F, Cresi F, Pautasso S, et al. Intestinal microflora in breast-fed colicky and non-colicky infants. Acta Paediatr. 2004;93:825-829.

7. Savino F, Bailo E, Oggero R, et al. Bacterial counts of intestinal Lactobacillus species in infants with colic. Pediatr Allergy Immunol. 2005;16:72-75.

8. Savino F, Pelle E, Palumeri E, Oggero R, et al. Lactobacillus reuteri (American type culture collection strain 55 730) versus simethicone in the treatment of infantile colic: a prospective randomized study. Pediatrics. 2007;119:e124-e130.

Article PDF
Author and Disclosure Information

Thomas Koonce, MD, MPH
Department of Family Medicine, The University of North Carolina at Chapel Hill

Anne Mounsey, MD
Department of Family Medicine, The University of North Carolina at Chapel Hill

Kate Rowland, MD
Department of Family Medicine, University of Chicago

PURLs EDITOR
John Hickner, MD, MSc
Cleveland Clinic

Issue
The Journal of Family Practice - 60(1)
Publications
Topics
Page Number
34-36
Legacy Keywords
Thomas Koonce; PURLs; colicky baby; probiotics; breastfed infant; lactobacilli
Sections
Author and Disclosure Information

Thomas Koonce, MD, MPH
Department of Family Medicine, The University of North Carolina at Chapel Hill

Anne Mounsey, MD
Department of Family Medicine, The University of North Carolina at Chapel Hill

Kate Rowland, MD
Department of Family Medicine, University of Chicago

PURLs EDITOR
John Hickner, MD, MSc
Cleveland Clinic

Author and Disclosure Information

Thomas Koonce, MD, MPH
Department of Family Medicine, The University of North Carolina at Chapel Hill

Anne Mounsey, MD
Department of Family Medicine, The University of North Carolina at Chapel Hill

Kate Rowland, MD
Department of Family Medicine, University of Chicago

PURLs EDITOR
John Hickner, MD, MSc
Cleveland Clinic

Article PDF
Article PDF
PRACTICE CHANGER

Suggest that parents of colicky breastfed infants try probiotics (Lactobacillus reuteri), which can significantly reduce daily crying time with no adverse effects.1

STRENGTH OF RECOMMENDATION:

A: A good-quality randomized controlled trial (RCT).

Savino F, Cordisco L, Tarasco V, et al. Lactobacillus reuteri DSM 17938 in infantile colic: a randomized, double-blind, placebo-controlled trial. Pediatrics. 2010;126:e526-e533.

 

ILLUSTRATIVE CASE

The parents of an otherwise healthy 10-week-old breastfed baby girl bring her in for the second time in 2 weeks because of her in-consolable crying. Physical examination and work-up remain normal, and you again diagnose colic. What can you suggest to help decrease the baby’s crying, other than the usual dietary advice?

Colic affects up to 28% of infants, causing considerable stress for parents and for their health care providers.2 Indeed, in the first 3 months of a baby’s life, crying is the No. 1 reason for pediatric visits.3 Parents often perceive—incorrectly—that the inconsolable crying is either a sign of serious illness or a result of poor parenting skills.4

A distressing problem, with few remedies
Despite the ubiquity and frustration that accompany colic, its exact etiology remains unclear and effective treatments remain elusive. With very little quality evidence to support interventions for colicky infants, we often have nothing more than grandmotherly advice to offer parents of babies with this vexing condition.

Current guidelines recommend only one strategy for breastfeeding mothers: a lowallergen diet.5 However, recent studies suggest that low counts of intestinal lactobacilli may play a role in colic and have documented improved symptoms after treatment with lactobacilli compared with treatment with simethicone.6-8 Infant formulas that contain probiotics are now available, as a result.

Although the results of the recent studies have been promising, they were not double-blinded or placebo-controlled. The study detailed here is the first to provide compelling evidence for a safe intervention for colicky breastfed infants.

STUDY SUMMARY: Lactobacilli cut crying time

In a randomized, double-blinded, placebo-controlled trial, Savino et al randomly assigned 50 exclusively breastfed colicky infants ages 2 to 16 weeks to receive either L reuteri DSM 17 938 (108 colony-forming units) or placebo daily for 21 days. Diagnosis of colic was made according to the modified Wessel’s criteria—fussy crying for ≥3 hours per day for ≥3 days per week in the week before enrollment. Their mothers were told to avoid ingesting cow’s milk during the course of the study.

Term infants adequate for gestational age were eligible for inclusion in the study. Exclusion criteria included evidence of chronic illness or gastrointestinal disorders, any intake of probiotics and/or antibiotics in the week preceding recruitment, and any formula feeding.

Parents and providers were blinded during the study, and they reported daily crying time, stool characteristics, adverse events, and growth patterns. An identical looking and tasting triglyceride oil without live bacteria was used in the placebo group. Each infant received 5 drops of L reuteri or placebo each morning 30 minutes before the morning feeding.

The primary outcome was a reduction of average crying time to <3 hours a day by Day 21. A secondary outcome was the number of infants in each group who experienced a 50% decrease in the daily average crying time from baseline on Days 7, 14, and 21 of the study.

Initially, the babies were divided equally between the control and intervention groups, but 4 participants in the control group were later excluded from analysis (1 had fever, 1 had reflux, and the parents of 2 infants did not complete the analysis).

At the time of enrollment, no significant differences were noted between the intervention and control groups regarding type of delivery, sex, age, family history of gastrointestinal disorders, growth parameters, and median daily crying time (370 minutes for the probiotics group vs 300 minutes for the placebo group).

 

 

 

By Day 21, the number of infants with crying times >3 hours was significantly lower in the treatment group compared with the placebo group (4 vs 12, respectively; P=.009). At all stages in the study, crying time for those in the treatment group was less than in the placebo group; median crying times for the intervention group were 95, 60, and 35 minutes per day, vs 185, 150, and 90 minutes for the controls, at 7, 14, and 21 days, respectively.

The number of infants with a 50% reduction in crying time was significantly greater in the treatment group than in the placebo group on Days 7, 14, and 21 (TABLE), although both groups saw an increase in the number of children whose average crying time had dropped by 50% as time went by. The number needed to treat to reduce crying time by 50% on Day 21 was 4.

There were no differences between the groups in growth, weight gain, frequency of stools, or incidence of regurgitation or constipation. No adverse events related to the treatment were reported.

TABLE
Babies respond* to probiotics

Day of studyL reuteri n=25 (%)Placebo n=21 (%)P value
720 (80)8 (36).006
1424 (96)13 (62).007
2124 (96)15 (71).036
*An infant with a decrease in daily average crying time of 50% from baseline was defined as a responder.

WHAT’S NEW: We have an evidence-based remedy that’s safe and effective

This study represents the first randomized, double-blinded, placebo-controlled investigation of probiotics to reduce infant colic. The researchers’ focus on patient-oriented outcomes and their solid study design move the notion of probiotics’ efficacy from conjecture to evidence. Furthermore, the study documents the safety of the intervention in the treatment group. This study increases our evidence-based armamentarium for treating colic, and family physicians should consider prescribing probiotics for healthy breastfed infants with colic.

CAVEATS: Will it work for bottle-fed infants?

This study was conducted in exclusively breast-fed, healthy infants whose mothers avoided dietary cow’s milk, which limits its applicability to a more general infant population. The study was funded by the makers of the probiotic, but the rigorous study design with random allocation, double-blind design, and intention-to-treat analysis makes bias unlikely. Although no adverse effects were reported during this study, there is little available evidence about the long-term effects of probiotics in infants. As L reuteri are naturally occurring gut bacteria, however, it seems unlikely that it would be harmful in the long term.

CHALLENGES TO IMPLEMENTATION: Parents will need to purchase the probiotics

As with any non–FDA-regulated product, it will be important to guide patients toward reputable manufacturers to ensure homogeneity of dosing. A 29-day supply of BioGaia probiotic drops (100 million units once a day), which costs $37 according to the manufacturer’s Web site, http://www.biogaia.com/consumer/biogaia-probiotic-products/probiotic-drops, should be affordable for most parents. Otherwise, little stands in the way of using this therapy to reduce the crying and subsequent stress associated with infant colic.

Acknowledgement

The PURLs Surveillance System is supported in part by Grant Number UL1RR024999 from the National Center for Research Resources, a Clinical Translational Science Award to the University of Chicago. The content is solely the responsibility of the authors and does not necessarily represent the official views of the National Center for Research Resources or the National Institutes of Health.

Click here to view PURL METHODOLOGY

PRACTICE CHANGER

Suggest that parents of colicky breastfed infants try probiotics (Lactobacillus reuteri), which can significantly reduce daily crying time with no adverse effects.1

STRENGTH OF RECOMMENDATION:

A: A good-quality randomized controlled trial (RCT).

Savino F, Cordisco L, Tarasco V, et al. Lactobacillus reuteri DSM 17938 in infantile colic: a randomized, double-blind, placebo-controlled trial. Pediatrics. 2010;126:e526-e533.

 

ILLUSTRATIVE CASE

The parents of an otherwise healthy 10-week-old breastfed baby girl bring her in for the second time in 2 weeks because of her in-consolable crying. Physical examination and work-up remain normal, and you again diagnose colic. What can you suggest to help decrease the baby’s crying, other than the usual dietary advice?

Colic affects up to 28% of infants, causing considerable stress for parents and for their health care providers.2 Indeed, in the first 3 months of a baby’s life, crying is the No. 1 reason for pediatric visits.3 Parents often perceive—incorrectly—that the inconsolable crying is either a sign of serious illness or a result of poor parenting skills.4

A distressing problem, with few remedies
Despite the ubiquity and frustration that accompany colic, its exact etiology remains unclear and effective treatments remain elusive. With very little quality evidence to support interventions for colicky infants, we often have nothing more than grandmotherly advice to offer parents of babies with this vexing condition.

Current guidelines recommend only one strategy for breastfeeding mothers: a lowallergen diet.5 However, recent studies suggest that low counts of intestinal lactobacilli may play a role in colic and have documented improved symptoms after treatment with lactobacilli compared with treatment with simethicone.6-8 Infant formulas that contain probiotics are now available, as a result.

Although the results of the recent studies have been promising, they were not double-blinded or placebo-controlled. The study detailed here is the first to provide compelling evidence for a safe intervention for colicky breastfed infants.

STUDY SUMMARY: Lactobacilli cut crying time

In a randomized, double-blinded, placebo-controlled trial, Savino et al randomly assigned 50 exclusively breastfed colicky infants ages 2 to 16 weeks to receive either L reuteri DSM 17 938 (108 colony-forming units) or placebo daily for 21 days. Diagnosis of colic was made according to the modified Wessel’s criteria—fussy crying for ≥3 hours per day for ≥3 days per week in the week before enrollment. Their mothers were told to avoid ingesting cow’s milk during the course of the study.

Term infants adequate for gestational age were eligible for inclusion in the study. Exclusion criteria included evidence of chronic illness or gastrointestinal disorders, any intake of probiotics and/or antibiotics in the week preceding recruitment, and any formula feeding.

Parents and providers were blinded during the study, and they reported daily crying time, stool characteristics, adverse events, and growth patterns. An identical looking and tasting triglyceride oil without live bacteria was used in the placebo group. Each infant received 5 drops of L reuteri or placebo each morning 30 minutes before the morning feeding.

The primary outcome was a reduction of average crying time to <3 hours a day by Day 21. A secondary outcome was the number of infants in each group who experienced a 50% decrease in the daily average crying time from baseline on Days 7, 14, and 21 of the study.

Initially, the babies were divided equally between the control and intervention groups, but 4 participants in the control group were later excluded from analysis (1 had fever, 1 had reflux, and the parents of 2 infants did not complete the analysis).

At the time of enrollment, no significant differences were noted between the intervention and control groups regarding type of delivery, sex, age, family history of gastrointestinal disorders, growth parameters, and median daily crying time (370 minutes for the probiotics group vs 300 minutes for the placebo group).

 

 

 

By Day 21, the number of infants with crying times >3 hours was significantly lower in the treatment group compared with the placebo group (4 vs 12, respectively; P=.009). At all stages in the study, crying time for those in the treatment group was less than in the placebo group; median crying times for the intervention group were 95, 60, and 35 minutes per day, vs 185, 150, and 90 minutes for the controls, at 7, 14, and 21 days, respectively.

The number of infants with a 50% reduction in crying time was significantly greater in the treatment group than in the placebo group on Days 7, 14, and 21 (TABLE), although both groups saw an increase in the number of children whose average crying time had dropped by 50% as time went by. The number needed to treat to reduce crying time by 50% on Day 21 was 4.

There were no differences between the groups in growth, weight gain, frequency of stools, or incidence of regurgitation or constipation. No adverse events related to the treatment were reported.

TABLE
Babies respond* to probiotics

Day of studyL reuteri n=25 (%)Placebo n=21 (%)P value
720 (80)8 (36).006
1424 (96)13 (62).007
2124 (96)15 (71).036
*An infant with a decrease in daily average crying time of 50% from baseline was defined as a responder.

WHAT’S NEW: We have an evidence-based remedy that’s safe and effective

This study represents the first randomized, double-blinded, placebo-controlled investigation of probiotics to reduce infant colic. The researchers’ focus on patient-oriented outcomes and their solid study design move the notion of probiotics’ efficacy from conjecture to evidence. Furthermore, the study documents the safety of the intervention in the treatment group. This study increases our evidence-based armamentarium for treating colic, and family physicians should consider prescribing probiotics for healthy breastfed infants with colic.

CAVEATS: Will it work for bottle-fed infants?

This study was conducted in exclusively breast-fed, healthy infants whose mothers avoided dietary cow’s milk, which limits its applicability to a more general infant population. The study was funded by the makers of the probiotic, but the rigorous study design with random allocation, double-blind design, and intention-to-treat analysis makes bias unlikely. Although no adverse effects were reported during this study, there is little available evidence about the long-term effects of probiotics in infants. As L reuteri are naturally occurring gut bacteria, however, it seems unlikely that it would be harmful in the long term.

CHALLENGES TO IMPLEMENTATION: Parents will need to purchase the probiotics

As with any non–FDA-regulated product, it will be important to guide patients toward reputable manufacturers to ensure homogeneity of dosing. A 29-day supply of BioGaia probiotic drops (100 million units once a day), which costs $37 according to the manufacturer’s Web site, http://www.biogaia.com/consumer/biogaia-probiotic-products/probiotic-drops, should be affordable for most parents. Otherwise, little stands in the way of using this therapy to reduce the crying and subsequent stress associated with infant colic.

Acknowledgement

The PURLs Surveillance System is supported in part by Grant Number UL1RR024999 from the National Center for Research Resources, a Clinical Translational Science Award to the University of Chicago. The content is solely the responsibility of the authors and does not necessarily represent the official views of the National Center for Research Resources or the National Institutes of Health.

Click here to view PURL METHODOLOGY

References

1. Savino F, Cordisco L, Tarasco V, et al. Lactobacillus reuteri DSM 17938 in infantile colic: a randomized, double-blind, placebo-controlled trial. Pediatrics. 2010;126:e526-e533.

2. Keefe MR, Kajrlsen KA, Lobo ML, et al. Reducing parenting stress in families with irritable infants. Nurs Res. 2006;55:198-205

3. Forsyth BWC, McCarthy PL, Leventhal JM. Problems of early infancy, formula changes, and mothers’ beliefs about their infants. J Pediatr. 1985;106:1012-1017.

4. Lehtonen LA, Rautava PT. Infantile colic: natural history and treatment. Curr Probl Pediatr. 1996;26:79.-

5. Cincinnati Children’s Hospital Medical Center. Best evidence statement (BESt). Maternal dietary antigen avoidance in lactation. Available at: http://www.cincinnatichildrens.org/svc/alpha/h/health-policy/best.htm. Accessed December 14, 2010.

6. Savino F, Cresi F, Pautasso S, et al. Intestinal microflora in breast-fed colicky and non-colicky infants. Acta Paediatr. 2004;93:825-829.

7. Savino F, Bailo E, Oggero R, et al. Bacterial counts of intestinal Lactobacillus species in infants with colic. Pediatr Allergy Immunol. 2005;16:72-75.

8. Savino F, Pelle E, Palumeri E, Oggero R, et al. Lactobacillus reuteri (American type culture collection strain 55 730) versus simethicone in the treatment of infantile colic: a prospective randomized study. Pediatrics. 2007;119:e124-e130.

References

1. Savino F, Cordisco L, Tarasco V, et al. Lactobacillus reuteri DSM 17938 in infantile colic: a randomized, double-blind, placebo-controlled trial. Pediatrics. 2010;126:e526-e533.

2. Keefe MR, Kajrlsen KA, Lobo ML, et al. Reducing parenting stress in families with irritable infants. Nurs Res. 2006;55:198-205

3. Forsyth BWC, McCarthy PL, Leventhal JM. Problems of early infancy, formula changes, and mothers’ beliefs about their infants. J Pediatr. 1985;106:1012-1017.

4. Lehtonen LA, Rautava PT. Infantile colic: natural history and treatment. Curr Probl Pediatr. 1996;26:79.-

5. Cincinnati Children’s Hospital Medical Center. Best evidence statement (BESt). Maternal dietary antigen avoidance in lactation. Available at: http://www.cincinnatichildrens.org/svc/alpha/h/health-policy/best.htm. Accessed December 14, 2010.

6. Savino F, Cresi F, Pautasso S, et al. Intestinal microflora in breast-fed colicky and non-colicky infants. Acta Paediatr. 2004;93:825-829.

7. Savino F, Bailo E, Oggero R, et al. Bacterial counts of intestinal Lactobacillus species in infants with colic. Pediatr Allergy Immunol. 2005;16:72-75.

8. Savino F, Pelle E, Palumeri E, Oggero R, et al. Lactobacillus reuteri (American type culture collection strain 55 730) versus simethicone in the treatment of infantile colic: a prospective randomized study. Pediatrics. 2007;119:e124-e130.

Issue
The Journal of Family Practice - 60(1)
Issue
The Journal of Family Practice - 60(1)
Page Number
34-36
Page Number
34-36
Publications
Publications
Topics
Article Type
Display Headline
Colicky baby? Here’s a surprising remedy
Display Headline
Colicky baby? Here’s a surprising remedy
Legacy Keywords
Thomas Koonce; PURLs; colicky baby; probiotics; breastfed infant; lactobacilli
Legacy Keywords
Thomas Koonce; PURLs; colicky baby; probiotics; breastfed infant; lactobacilli
Sections
PURLs Copyright

Copyright © 2011 The Family Physicians Inquiries Network.
All rights reserved.

Disallow All Ads
Alternative CME
Article PDF Media

Palliative care: Earlier is better

Article Type
Changed
Mon, 01/14/2019 - 11:38
Display Headline
Palliative care: Earlier is better

PRACTICE CHANGER

Recommend a palliative care consultation at the time of diagnosis. Early palliative care can improve quality of life, decrease depressive symptoms, and prolong life in patients with metastatic cancer.1

STRENGTH OF RECOMMENDATION

B: Based on a single well-done randomized controlled trial (RCT).

Temel JS, Greer JA, Muzikansky A, et al. Early palliative care for patients with metastatic non–small-cell lung cancer. N Engl J Med. 2010;363:733-742.

ILLUSTRATIVE CASE

A 73-year-old patient you’ve known for your entire career comes in for follow-up after a recent hospitalization, during which he was diagnosed with metastatic non–small-cell lung cancer. “I know things don’t look good,” he says. “I don’t want to die a miserable, painful death. But I’m not going to just roll over and die without fighting this.” What can you do to improve his quality of life while he undergoes cancer treatment?

Palliative care focuses on the prevention and treatment of pain and other debilitating effects of serious illness, with a goal of improving quality of life for patients and their families. Unlike hospice care, which requires a prognosis of less than 6 months of life to qualify for Medicare reimbursement,2 eligibility for palliative care is not dependent on prognosis. Indeed, palliative care can occur at the same time as curative or life-prolonging treatment. Palliative care programs include psychosocial and spiritual care for patient and family; management of symptoms such as pain, fatigue, shortness of breath, depression, constipation, and nausea; support for complex decisions, such as discussions of goals, do not resuscitate (DNR) orders, and requests for treatment; and coordination of care across various health care settings.3

Palliative care lowers health care spending
One study found that palliative care consultation was associated with an average savings of $1700 per admission for patients who were discharged, and $4900, on average, for every patient who died in the hospital.4 Another study demonstrated an association between states with a higher percentage of hospitals with palliative care services and fewer Medicare hospital deaths; fewer admissions to, and days in, intensive care units in the last 6 months of life; and lower total Medicare spending per enrollee.5

A 2008 systematic review of the effectiveness of palliative care revealed that there were methodological limitations in all the existing studies of palliative care, and called for higher quality studies.6 The RCT detailed here is a first step toward filling the gap in palliative care research.

STUDY SUMMARY: Intervention group lived longer and felt better

Temel et al enrolled 151 ambulatory patients with biopsy-proven non–small-cell lung cancer. The average age of the enrollees was 64 years, and slightly more than half (51.6%) were female. All had been diagnosed with metastatic cancer within 8 weeks of enrollment in the study.

The patients were randomized to receive either an early referral to the palliative care team along with standard oncology care or standard oncology care alone. Race, marital status, smoking history, presence of brain metastases, and initial cancer therapy—radiation, chemotherapy, or a combination—were similar for both groups.

The study ran for 12 weeks. Those in the intervention group had an initial meeting with a member of the palliative care team, which consisted of board-certified palliative care physicians and advanced practice nurses. Follow-up meetings with the team were scheduled at least monthly, and more frequently if requested by the patient or recommended by either the palliative care team or the oncology team—with an average of 4 meetings over the course of the study. Palliative care team members worked with patients to assess physical and emotional symptoms, coordinate care, and determine and document goals of treatment.

The primary outcome was the change in quality of life (QOL) from baseline to 12 weeks after the initial meeting with the palliative care team. QOL was measured with the Functional Assessment of Cancer Therapy-Lung (FACT-L) tool; scores range from 0 to 136, with higher scores indicating a higher QOL. The researchers used 3 subscales of the FACT-L—physical well-being, functional well-being, and a lung-cancer subscale (LCS) based on questions about 7 symptoms—to create a Trial Outcome Index (TOI), the main outcome measure. The TOI, which is the sum of the subscales, has a range of 0 to 84, with higher scores indicating higher QOL.

Secondary outcome measures were mood, use of health care services, and survival. The researchers assessed mood with 2 tools: the Patient Health Questionnaire-9 (PHQ-9) and the Hospital Anxiety and Depression Scale (HADS). The PHQ-9 is a 9-question survey that uses criteria from the Diagnostic and Statistical Manual of Psychiatric Disorders, 4th edition (DSM-IV) to diagnose depression. HADS is a 14-question survey with subscales for depression (HADS-D) and anxiety (HADS-A).

 

 

Intervention group had better scores. At study’s end, the control group had average scores of 91.5, 19.3, and 53.0 on the FACT-L, LCS, and TOI, respectively, vs 98.0, 21.0, and 59.0 for the intervention group. The palliative care group had an average increase on the TOI of 2.3 points, while the average for the control group decreased by 2.3 points (P=.04). A comparison of the mean change in scores between the 2 groups indicated statistically significant improvements in the FACT-L and TOI results for the intervention group. The improvement in LCS was not statistically significant.

The palliative care group also had a lower prevalence of depression compared with the controls (4% vs 17% on the PHQ-9 [P=.04]; 16% vs 38% on the HADS-D [P=.01]). For every 8 patients who received early palliative care, 1 less patient was diagnosed with depression. The prevalence of anxiety was not significantly different between groups.

Among patients who died during the study period, those in the palliative care group were less likely to have received aggressive end-of-life interventions compared with the controls (33% vs 54%, respectively, P=.05). Aggressive care was defined as chemotherapy within 14 days of death or little or no hospice care. Those in the early palliative care group also lived significantly longer; median survival was 11.6 months, vs 8.9 months for the control group (P=.02).

WHAT’S NEW: This study highlights the need for early referral

This is the first high-quality RCT to demonstrate improved patient outcomes when palliative care is begun close to the time of cancer diagnosis. Previous studies of late palliative care referrals did not demonstrate improved QOL or more appropriate use of health care services. This study established that patients with lung cancer are less depressed and live longer when they receive palliative care services soon after diagnosis. It also showed a link between palliative care and a reduction in aggressive, possibly inappropriate, end-of-life treatment of metastatic cancer.

Several recent practice guidelines, including that of the Institute for Clinical Systems Improvement (ICSI), recommend that palliative care referrals be made early in the course of a progressive, debilitating illness, regardless of the patient’s life expectancy.7 Other organizations, including the Institute of Medicine and the World Health Organization, recommend palliative care as an essential component of comprehensive cancer care.8 This study supports both of these recommendations.

CAVEATS: Would extra attention from any clinician work equally well?

No attempt was made to control for the extra attention (an average of 4 visits) that the palliative care team provided to those in the intervention group. Thus, it is possible that the study results could be replicated by having patients meet with their primary care physician or another health professional instead of a palliative care team.

The reduction in depression and increase in survival are clinically significant outcomes. But the improvement in QOL (an average of 7 points better on the 136-point FACT-L scale, or 6 points on the 84-point TOI scale) may not be.

It is important to note, too, that the survival benefits the researchers found may not be generalizable to other kinds of cancers. In addition, most patients (97%) in this study were white, so the findings may be less generalizable to patients of other races. Nonetheless, we think it’s likely that the improvements in QOL and mood revealed in this study would be realized by most patients with terminal cancer who received early palliative care.

CHALLENGES TO IMPLEMENTATION: Palliative care must be explained—and available

Physicians must be able to explain to their patients the difference between palliative care and hospice—most notably, that patients can continue to receive anticancer treatment while receiving palliative care. The recommendation to seek palliative care should not be considered “giving up” on the patient.

In order to refer patients to palliative care early in the course of cancer care, physicians must have access to a palliative care team, which may not be available in all cases. In 2006, only 53% of hospitals with more than 50 beds reported having a palliative care program.5 If there is no such program available, physicians can refer to the ICSI guideline on palliative care for more information on how to implement elements of palliative care for their patients with advanced cancer.7

References

1. Temel JS, Greer JA, Muzikansky A, et al. Early palliative care for patients with metastatic non–small-cell lung cancer. N Engl J Med. 2010;363:733-742.

2. Centers for Medicare and Medicaid Services. Medicare hospice benefits. Available at: http://www.medicare.gov/publications/pubs/pdf/02154.pdf. Accessed September 30, 2010.

3. Center to Advance Palliative Care. The case for hospital palliative care: improving quality, reducing cost. Available at: http://www.capc.org/building-a-hospital-based-palliative-care-program/case/support-from-capc/capc_publications/making-the-case.pdf. Accessed September 30, 2010.

4. Morrison RS, Penrod JD, Cassel JB, et al. Cost savings associated with US hospital palliative care consultation programs. Arch Intern Med. 2008;168:1783-1790.

5. Goldsmith B, Dietrich J, Du Q, et al. Variability in access to hospital palliative care in the United States. J Palliat Med. 2008;11:1094-1102.

6. Zimmermann C, Riechelmann R. Effectiveness of specialized palliative care: a systematic review. JAMA. 2008;299:1698-1709.

7. Institute for Clinical Systems Improvement (ICSI) 2009 palliative care guideline. Available at: http://www.icsi.org/guidelines_and_more/gl_os_prot/other_health_care_conditions/palliative_care/palliative_care_11875.html. Accessed September 30, 2010.

8. Ferris FD, Bruera E, Cherny N, et al. Palliative cancer care a decade later: accomplishments, the need, next steps—from the American Society of Clinical Oncology. J Clin Oncol. 2009;27:3052-3058.

Article PDF
Author and Disclosure Information

Kate Rowland, MD
Sarah-Anne Schumann, MD
Department of Family Medicine, University of Chicago

PURLs EDITOR
John Hickner, MD, MSc
Cleveland Clinic

Issue
The Journal of Family Practice - 59(12)
Publications
Topics
Page Number
695-698
Legacy Keywords
Kate Rowland; palliative care; PURLs; metastatic cancer; early palliative care; palliative care consultation; quality of life
Sections
Author and Disclosure Information

Kate Rowland, MD
Sarah-Anne Schumann, MD
Department of Family Medicine, University of Chicago

PURLs EDITOR
John Hickner, MD, MSc
Cleveland Clinic

Author and Disclosure Information

Kate Rowland, MD
Sarah-Anne Schumann, MD
Department of Family Medicine, University of Chicago

PURLs EDITOR
John Hickner, MD, MSc
Cleveland Clinic

Article PDF
Article PDF

PRACTICE CHANGER

Recommend a palliative care consultation at the time of diagnosis. Early palliative care can improve quality of life, decrease depressive symptoms, and prolong life in patients with metastatic cancer.1

STRENGTH OF RECOMMENDATION

B: Based on a single well-done randomized controlled trial (RCT).

Temel JS, Greer JA, Muzikansky A, et al. Early palliative care for patients with metastatic non–small-cell lung cancer. N Engl J Med. 2010;363:733-742.

ILLUSTRATIVE CASE

A 73-year-old patient you’ve known for your entire career comes in for follow-up after a recent hospitalization, during which he was diagnosed with metastatic non–small-cell lung cancer. “I know things don’t look good,” he says. “I don’t want to die a miserable, painful death. But I’m not going to just roll over and die without fighting this.” What can you do to improve his quality of life while he undergoes cancer treatment?

Palliative care focuses on the prevention and treatment of pain and other debilitating effects of serious illness, with a goal of improving quality of life for patients and their families. Unlike hospice care, which requires a prognosis of less than 6 months of life to qualify for Medicare reimbursement,2 eligibility for palliative care is not dependent on prognosis. Indeed, palliative care can occur at the same time as curative or life-prolonging treatment. Palliative care programs include psychosocial and spiritual care for patient and family; management of symptoms such as pain, fatigue, shortness of breath, depression, constipation, and nausea; support for complex decisions, such as discussions of goals, do not resuscitate (DNR) orders, and requests for treatment; and coordination of care across various health care settings.3

Palliative care lowers health care spending
One study found that palliative care consultation was associated with an average savings of $1700 per admission for patients who were discharged, and $4900, on average, for every patient who died in the hospital.4 Another study demonstrated an association between states with a higher percentage of hospitals with palliative care services and fewer Medicare hospital deaths; fewer admissions to, and days in, intensive care units in the last 6 months of life; and lower total Medicare spending per enrollee.5

A 2008 systematic review of the effectiveness of palliative care revealed that there were methodological limitations in all the existing studies of palliative care, and called for higher quality studies.6 The RCT detailed here is a first step toward filling the gap in palliative care research.

STUDY SUMMARY: Intervention group lived longer and felt better

Temel et al enrolled 151 ambulatory patients with biopsy-proven non–small-cell lung cancer. The average age of the enrollees was 64 years, and slightly more than half (51.6%) were female. All had been diagnosed with metastatic cancer within 8 weeks of enrollment in the study.

The patients were randomized to receive either an early referral to the palliative care team along with standard oncology care or standard oncology care alone. Race, marital status, smoking history, presence of brain metastases, and initial cancer therapy—radiation, chemotherapy, or a combination—were similar for both groups.

The study ran for 12 weeks. Those in the intervention group had an initial meeting with a member of the palliative care team, which consisted of board-certified palliative care physicians and advanced practice nurses. Follow-up meetings with the team were scheduled at least monthly, and more frequently if requested by the patient or recommended by either the palliative care team or the oncology team—with an average of 4 meetings over the course of the study. Palliative care team members worked with patients to assess physical and emotional symptoms, coordinate care, and determine and document goals of treatment.

The primary outcome was the change in quality of life (QOL) from baseline to 12 weeks after the initial meeting with the palliative care team. QOL was measured with the Functional Assessment of Cancer Therapy-Lung (FACT-L) tool; scores range from 0 to 136, with higher scores indicating a higher QOL. The researchers used 3 subscales of the FACT-L—physical well-being, functional well-being, and a lung-cancer subscale (LCS) based on questions about 7 symptoms—to create a Trial Outcome Index (TOI), the main outcome measure. The TOI, which is the sum of the subscales, has a range of 0 to 84, with higher scores indicating higher QOL.

Secondary outcome measures were mood, use of health care services, and survival. The researchers assessed mood with 2 tools: the Patient Health Questionnaire-9 (PHQ-9) and the Hospital Anxiety and Depression Scale (HADS). The PHQ-9 is a 9-question survey that uses criteria from the Diagnostic and Statistical Manual of Psychiatric Disorders, 4th edition (DSM-IV) to diagnose depression. HADS is a 14-question survey with subscales for depression (HADS-D) and anxiety (HADS-A).

 

 

Intervention group had better scores. At study’s end, the control group had average scores of 91.5, 19.3, and 53.0 on the FACT-L, LCS, and TOI, respectively, vs 98.0, 21.0, and 59.0 for the intervention group. The palliative care group had an average increase on the TOI of 2.3 points, while the average for the control group decreased by 2.3 points (P=.04). A comparison of the mean change in scores between the 2 groups indicated statistically significant improvements in the FACT-L and TOI results for the intervention group. The improvement in LCS was not statistically significant.

The palliative care group also had a lower prevalence of depression compared with the controls (4% vs 17% on the PHQ-9 [P=.04]; 16% vs 38% on the HADS-D [P=.01]). For every 8 patients who received early palliative care, 1 less patient was diagnosed with depression. The prevalence of anxiety was not significantly different between groups.

Among patients who died during the study period, those in the palliative care group were less likely to have received aggressive end-of-life interventions compared with the controls (33% vs 54%, respectively, P=.05). Aggressive care was defined as chemotherapy within 14 days of death or little or no hospice care. Those in the early palliative care group also lived significantly longer; median survival was 11.6 months, vs 8.9 months for the control group (P=.02).

WHAT’S NEW: This study highlights the need for early referral

This is the first high-quality RCT to demonstrate improved patient outcomes when palliative care is begun close to the time of cancer diagnosis. Previous studies of late palliative care referrals did not demonstrate improved QOL or more appropriate use of health care services. This study established that patients with lung cancer are less depressed and live longer when they receive palliative care services soon after diagnosis. It also showed a link between palliative care and a reduction in aggressive, possibly inappropriate, end-of-life treatment of metastatic cancer.

Several recent practice guidelines, including that of the Institute for Clinical Systems Improvement (ICSI), recommend that palliative care referrals be made early in the course of a progressive, debilitating illness, regardless of the patient’s life expectancy.7 Other organizations, including the Institute of Medicine and the World Health Organization, recommend palliative care as an essential component of comprehensive cancer care.8 This study supports both of these recommendations.

CAVEATS: Would extra attention from any clinician work equally well?

No attempt was made to control for the extra attention (an average of 4 visits) that the palliative care team provided to those in the intervention group. Thus, it is possible that the study results could be replicated by having patients meet with their primary care physician or another health professional instead of a palliative care team.

The reduction in depression and increase in survival are clinically significant outcomes. But the improvement in QOL (an average of 7 points better on the 136-point FACT-L scale, or 6 points on the 84-point TOI scale) may not be.

It is important to note, too, that the survival benefits the researchers found may not be generalizable to other kinds of cancers. In addition, most patients (97%) in this study were white, so the findings may be less generalizable to patients of other races. Nonetheless, we think it’s likely that the improvements in QOL and mood revealed in this study would be realized by most patients with terminal cancer who received early palliative care.

CHALLENGES TO IMPLEMENTATION: Palliative care must be explained—and available

Physicians must be able to explain to their patients the difference between palliative care and hospice—most notably, that patients can continue to receive anticancer treatment while receiving palliative care. The recommendation to seek palliative care should not be considered “giving up” on the patient.

In order to refer patients to palliative care early in the course of cancer care, physicians must have access to a palliative care team, which may not be available in all cases. In 2006, only 53% of hospitals with more than 50 beds reported having a palliative care program.5 If there is no such program available, physicians can refer to the ICSI guideline on palliative care for more information on how to implement elements of palliative care for their patients with advanced cancer.7

PRACTICE CHANGER

Recommend a palliative care consultation at the time of diagnosis. Early palliative care can improve quality of life, decrease depressive symptoms, and prolong life in patients with metastatic cancer.1

STRENGTH OF RECOMMENDATION

B: Based on a single well-done randomized controlled trial (RCT).

Temel JS, Greer JA, Muzikansky A, et al. Early palliative care for patients with metastatic non–small-cell lung cancer. N Engl J Med. 2010;363:733-742.

ILLUSTRATIVE CASE

A 73-year-old patient you’ve known for your entire career comes in for follow-up after a recent hospitalization, during which he was diagnosed with metastatic non–small-cell lung cancer. “I know things don’t look good,” he says. “I don’t want to die a miserable, painful death. But I’m not going to just roll over and die without fighting this.” What can you do to improve his quality of life while he undergoes cancer treatment?

Palliative care focuses on the prevention and treatment of pain and other debilitating effects of serious illness, with a goal of improving quality of life for patients and their families. Unlike hospice care, which requires a prognosis of less than 6 months of life to qualify for Medicare reimbursement,2 eligibility for palliative care is not dependent on prognosis. Indeed, palliative care can occur at the same time as curative or life-prolonging treatment. Palliative care programs include psychosocial and spiritual care for patient and family; management of symptoms such as pain, fatigue, shortness of breath, depression, constipation, and nausea; support for complex decisions, such as discussions of goals, do not resuscitate (DNR) orders, and requests for treatment; and coordination of care across various health care settings.3

Palliative care lowers health care spending
One study found that palliative care consultation was associated with an average savings of $1700 per admission for patients who were discharged, and $4900, on average, for every patient who died in the hospital.4 Another study demonstrated an association between states with a higher percentage of hospitals with palliative care services and fewer Medicare hospital deaths; fewer admissions to, and days in, intensive care units in the last 6 months of life; and lower total Medicare spending per enrollee.5

A 2008 systematic review of the effectiveness of palliative care revealed that there were methodological limitations in all the existing studies of palliative care, and called for higher quality studies.6 The RCT detailed here is a first step toward filling the gap in palliative care research.

STUDY SUMMARY: Intervention group lived longer and felt better

Temel et al enrolled 151 ambulatory patients with biopsy-proven non–small-cell lung cancer. The average age of the enrollees was 64 years, and slightly more than half (51.6%) were female. All had been diagnosed with metastatic cancer within 8 weeks of enrollment in the study.

The patients were randomized to receive either an early referral to the palliative care team along with standard oncology care or standard oncology care alone. Race, marital status, smoking history, presence of brain metastases, and initial cancer therapy—radiation, chemotherapy, or a combination—were similar for both groups.

The study ran for 12 weeks. Those in the intervention group had an initial meeting with a member of the palliative care team, which consisted of board-certified palliative care physicians and advanced practice nurses. Follow-up meetings with the team were scheduled at least monthly, and more frequently if requested by the patient or recommended by either the palliative care team or the oncology team—with an average of 4 meetings over the course of the study. Palliative care team members worked with patients to assess physical and emotional symptoms, coordinate care, and determine and document goals of treatment.

The primary outcome was the change in quality of life (QOL) from baseline to 12 weeks after the initial meeting with the palliative care team. QOL was measured with the Functional Assessment of Cancer Therapy-Lung (FACT-L) tool; scores range from 0 to 136, with higher scores indicating a higher QOL. The researchers used 3 subscales of the FACT-L—physical well-being, functional well-being, and a lung-cancer subscale (LCS) based on questions about 7 symptoms—to create a Trial Outcome Index (TOI), the main outcome measure. The TOI, which is the sum of the subscales, has a range of 0 to 84, with higher scores indicating higher QOL.

Secondary outcome measures were mood, use of health care services, and survival. The researchers assessed mood with 2 tools: the Patient Health Questionnaire-9 (PHQ-9) and the Hospital Anxiety and Depression Scale (HADS). The PHQ-9 is a 9-question survey that uses criteria from the Diagnostic and Statistical Manual of Psychiatric Disorders, 4th edition (DSM-IV) to diagnose depression. HADS is a 14-question survey with subscales for depression (HADS-D) and anxiety (HADS-A).

 

 

Intervention group had better scores. At study’s end, the control group had average scores of 91.5, 19.3, and 53.0 on the FACT-L, LCS, and TOI, respectively, vs 98.0, 21.0, and 59.0 for the intervention group. The palliative care group had an average increase on the TOI of 2.3 points, while the average for the control group decreased by 2.3 points (P=.04). A comparison of the mean change in scores between the 2 groups indicated statistically significant improvements in the FACT-L and TOI results for the intervention group. The improvement in LCS was not statistically significant.

The palliative care group also had a lower prevalence of depression compared with the controls (4% vs 17% on the PHQ-9 [P=.04]; 16% vs 38% on the HADS-D [P=.01]). For every 8 patients who received early palliative care, 1 less patient was diagnosed with depression. The prevalence of anxiety was not significantly different between groups.

Among patients who died during the study period, those in the palliative care group were less likely to have received aggressive end-of-life interventions compared with the controls (33% vs 54%, respectively, P=.05). Aggressive care was defined as chemotherapy within 14 days of death or little or no hospice care. Those in the early palliative care group also lived significantly longer; median survival was 11.6 months, vs 8.9 months for the control group (P=.02).

WHAT’S NEW: This study highlights the need for early referral

This is the first high-quality RCT to demonstrate improved patient outcomes when palliative care is begun close to the time of cancer diagnosis. Previous studies of late palliative care referrals did not demonstrate improved QOL or more appropriate use of health care services. This study established that patients with lung cancer are less depressed and live longer when they receive palliative care services soon after diagnosis. It also showed a link between palliative care and a reduction in aggressive, possibly inappropriate, end-of-life treatment of metastatic cancer.

Several recent practice guidelines, including that of the Institute for Clinical Systems Improvement (ICSI), recommend that palliative care referrals be made early in the course of a progressive, debilitating illness, regardless of the patient’s life expectancy.7 Other organizations, including the Institute of Medicine and the World Health Organization, recommend palliative care as an essential component of comprehensive cancer care.8 This study supports both of these recommendations.

CAVEATS: Would extra attention from any clinician work equally well?

No attempt was made to control for the extra attention (an average of 4 visits) that the palliative care team provided to those in the intervention group. Thus, it is possible that the study results could be replicated by having patients meet with their primary care physician or another health professional instead of a palliative care team.

The reduction in depression and increase in survival are clinically significant outcomes. But the improvement in QOL (an average of 7 points better on the 136-point FACT-L scale, or 6 points on the 84-point TOI scale) may not be.

It is important to note, too, that the survival benefits the researchers found may not be generalizable to other kinds of cancers. In addition, most patients (97%) in this study were white, so the findings may be less generalizable to patients of other races. Nonetheless, we think it’s likely that the improvements in QOL and mood revealed in this study would be realized by most patients with terminal cancer who received early palliative care.

CHALLENGES TO IMPLEMENTATION: Palliative care must be explained—and available

Physicians must be able to explain to their patients the difference between palliative care and hospice—most notably, that patients can continue to receive anticancer treatment while receiving palliative care. The recommendation to seek palliative care should not be considered “giving up” on the patient.

In order to refer patients to palliative care early in the course of cancer care, physicians must have access to a palliative care team, which may not be available in all cases. In 2006, only 53% of hospitals with more than 50 beds reported having a palliative care program.5 If there is no such program available, physicians can refer to the ICSI guideline on palliative care for more information on how to implement elements of palliative care for their patients with advanced cancer.7

References

1. Temel JS, Greer JA, Muzikansky A, et al. Early palliative care for patients with metastatic non–small-cell lung cancer. N Engl J Med. 2010;363:733-742.

2. Centers for Medicare and Medicaid Services. Medicare hospice benefits. Available at: http://www.medicare.gov/publications/pubs/pdf/02154.pdf. Accessed September 30, 2010.

3. Center to Advance Palliative Care. The case for hospital palliative care: improving quality, reducing cost. Available at: http://www.capc.org/building-a-hospital-based-palliative-care-program/case/support-from-capc/capc_publications/making-the-case.pdf. Accessed September 30, 2010.

4. Morrison RS, Penrod JD, Cassel JB, et al. Cost savings associated with US hospital palliative care consultation programs. Arch Intern Med. 2008;168:1783-1790.

5. Goldsmith B, Dietrich J, Du Q, et al. Variability in access to hospital palliative care in the United States. J Palliat Med. 2008;11:1094-1102.

6. Zimmermann C, Riechelmann R. Effectiveness of specialized palliative care: a systematic review. JAMA. 2008;299:1698-1709.

7. Institute for Clinical Systems Improvement (ICSI) 2009 palliative care guideline. Available at: http://www.icsi.org/guidelines_and_more/gl_os_prot/other_health_care_conditions/palliative_care/palliative_care_11875.html. Accessed September 30, 2010.

8. Ferris FD, Bruera E, Cherny N, et al. Palliative cancer care a decade later: accomplishments, the need, next steps—from the American Society of Clinical Oncology. J Clin Oncol. 2009;27:3052-3058.

References

1. Temel JS, Greer JA, Muzikansky A, et al. Early palliative care for patients with metastatic non–small-cell lung cancer. N Engl J Med. 2010;363:733-742.

2. Centers for Medicare and Medicaid Services. Medicare hospice benefits. Available at: http://www.medicare.gov/publications/pubs/pdf/02154.pdf. Accessed September 30, 2010.

3. Center to Advance Palliative Care. The case for hospital palliative care: improving quality, reducing cost. Available at: http://www.capc.org/building-a-hospital-based-palliative-care-program/case/support-from-capc/capc_publications/making-the-case.pdf. Accessed September 30, 2010.

4. Morrison RS, Penrod JD, Cassel JB, et al. Cost savings associated with US hospital palliative care consultation programs. Arch Intern Med. 2008;168:1783-1790.

5. Goldsmith B, Dietrich J, Du Q, et al. Variability in access to hospital palliative care in the United States. J Palliat Med. 2008;11:1094-1102.

6. Zimmermann C, Riechelmann R. Effectiveness of specialized palliative care: a systematic review. JAMA. 2008;299:1698-1709.

7. Institute for Clinical Systems Improvement (ICSI) 2009 palliative care guideline. Available at: http://www.icsi.org/guidelines_and_more/gl_os_prot/other_health_care_conditions/palliative_care/palliative_care_11875.html. Accessed September 30, 2010.

8. Ferris FD, Bruera E, Cherny N, et al. Palliative cancer care a decade later: accomplishments, the need, next steps—from the American Society of Clinical Oncology. J Clin Oncol. 2009;27:3052-3058.

Issue
The Journal of Family Practice - 59(12)
Issue
The Journal of Family Practice - 59(12)
Page Number
695-698
Page Number
695-698
Publications
Publications
Topics
Article Type
Display Headline
Palliative care: Earlier is better
Display Headline
Palliative care: Earlier is better
Legacy Keywords
Kate Rowland; palliative care; PURLs; metastatic cancer; early palliative care; palliative care consultation; quality of life
Legacy Keywords
Kate Rowland; palliative care; PURLs; metastatic cancer; early palliative care; palliative care consultation; quality of life
Sections
Article Source

PURLs Copyright

Copyright © 2010 The Family Physicians Inquiries Network.
All rights reserved.

Inside the Article

Article PDF Media

Add a fibrate to a statin?

Article Type
Changed
Mon, 01/14/2019 - 11:38
Display Headline
Add a fibrate to a statin?
PRACTICE CHANGER

Do not routinely add a fibrate to a statin for patients with type 2 diabetes who are at high risk for cardiovascular events.1

STRENGTH OF RECOMMENDATION

B: Based on a good-quality randomized controlled trial.

The ACCORD Study Group. Effects of combination lipid therapy in type 2 diabetes mellitus. N Engl J Med. 2010;362:1563-1574.

 

ILLUSTRATIVE CASE

A 60-year-old man with cardiovascular disease and diabetes comes to your clinic for a routine check of his dyslipidemia, for which he is on statin therapy. His fasting lipid panel shows a low-density lipoprotein (LDL) of 70 mg/dL, triglycerides of 200 mg/dL, and a high-density lipoprotein (HDL) of 30 mg/dL. Should you recommend adding fenofibrate?

Patients with type 2 diabetes are at increased risk for cardiovascular events. National Cholesterol Education Program Adult Treatment Panel (NCEP ATP) III guidelines recommend treatment of dyslipidemia for all patients with high risk of cardiovascular events to an LDL goal of <100 mg/dL (optional <70 mg/dL), HDL goal of >40 mg/dL, and triglyceride goal of <150 mg/dL. These recommendations include the use of combination therapy with a statin and fenofibrate for patients who have elevated triglycerides and low HDL cholesterol despite being on statin therapy alone.2

Survival benefit of combo therapy remains unproven
We know that fibrate therapy alone in patients with type 2 diabetes reduces major cardiovascular events.3,4 We also know that adding a fibrate to statin therapy can help patients reach their HDL and triglyceride targets. However, the survival benefit of the fibrate-statin combination over that of a statin alone has not been proven. In addition, there have been concerns about the increased risk of adverse effects with the combination. In fact, the overall benefits (and risks) of combining fibrates and statins for patients with diabetes and dyslipidemia were not addressed in a large randomized trial until the study we report on here.

STUDY SUMMARY: Statin + fibrate = minimal benefit for most patients

The Action to Control Cardiovascular Risk in Diabetes (ACCORD) study is among the largest trials conducted in adults with type 2 diabetes at high risk of cardiovascular events.5 The study examined 3 approaches to lowering the risk of major cardiovascular events: intensive lowering of blood sugar levels compared with standard blood sugar treatment; intensive lowering of blood pressure (BP) compared with standard BP treatment; and treatment of lipids with 2 drugs—a fibrate plus a statin—compared with a statin alone. This summary focuses on the lipid arm of the ACCORD study.1

All patients in the study had type 2 diabetes and a hemoglobin A1c ≥7.5%. The study included patients ages 40 to 79 years with clinical evidence of cardiovascular disease and patients ages 55 to 79 years with either subclinical cardiovascular disease or ≥2 cardiovascular risk factors in addition to diabetes.

The lipid arm enrolled patients who had an LDL cholesterol of 60 to 180 mg/dL, an HDL cholesterol <55 mg/dL for women and blacks and <50 mg/dL for all other groups, and a triglyceride level <750 mg/dL for those not receiving lipid therapy and <400 mg/dL for those on lipid therapy. Enrollees (N=5518) were started on open-label simvastatin 20 mg, titrated up as needed to reach the LDL goal, then randomized to receive either fenofibrate or placebo 1 month later. The mean duration of follow-up was 4.7 years.

The primary outcome was the first occurrence of a major cardiovascular event— nonfatal myocardial infarction (MI), nonfatal stroke, or death from a cardiovascular cause. The annual rate of the primary outcome was 2.2% (n=291) in the fenofibrate-statin group and 2.4% (n=310) in the placebo group, a nonsignificant difference (P=.32).

The results were reported by sex. The primary outcome rate for men during the 4.7-year follow-up was 11.2% in the fenofibrate-statin group vs 13.3% in the placebo group; for women, the outcome rates were 9.1% in the treatment group and 6.6% in the placebo group (P =.01). These rates suggest a small benefit for men, and harm for women.

Subgroup analysis showed additional benefit from fenofibrate in patients with a combination of a high baseline triglyceride level (≥204 mg/dL) and very low baseline HDL cholesterol (≤34 mg/dL), representing about 16% of the study participants. The primary outcome rate for patients in this subgroup was 12.4% in the fenofibrate-statin group and 17.3% in the placebo group (P=.057); number needed to treat (NNT)=20 patients for 4.7 years to prevent 1 major cardiovascular event.

Harm was similar in both groups. A small number of patients had elevations of alanine aminotransferase of >3 times the upper limit of normal (1.9% in the fenofibrate-statin group and 1.5% in the statin group). The study drug was discontinued in 66 patients (2.4%) in the fenofibrate-statin group (and the placebo was discontinued in 30 patients [1.1%] in the statin group). The fenofibrate or statin dose was reduced in 440 patients (15.9%) in the fenofibrate-statin group and in 194 patients (7%) in the statin group due to a decrease in estimated glomerular filtration rate. There was no significant difference in the incidence of hemodialysis and end-stage renal disease (75 patients in the fenofibrate-statin group vs 77 patients in the statin group).

 

 

 

WHAT’S NEW: We have evidence that combo therapy doesn’t further reduce risk

This study examined a previously unaddressed question, the role of combination fibrate-statin therapy in high-risk patients with type 2 diabetes. The findings do not support the routine use of combination therapy compared with a statin alone for most patients with diabetes. Overall, combination therapy with simvastatin and fenofibrate did not lower the risk of MI, stroke, or death from cardiovascular disease more than simvastatin alone.

This trial showed that women with diabetes and hyperlipidemia should not be treated with both a statin and a fibrate. Men appeared to have a very small benefit from combination therapy (NNT=50). Patients with a baseline HDL ≤34 mg/dL and baseline triglyceride ≥204 mg/dL appeared to benefit from the combination, but this group constituted only 16% of the patients in this trial and the difference had borderline statistical significance. Nonetheless, it may be reasonable to treat such patients with combination therapy until a definitive study is done.

CAVEATS: Statin dose did not match standard practice

This study used a low dose of statin. The average daily simvastatin dose was 22.3 mg in the fenofibrate-statin group and 22.4 mg in the placebo group. This constitutes low-dose therapy compared with doses routinely used in practice (ie, 40 or 80 mg). A higher dose of simvastatin may have negated any outcome differences.

CHALLENGES TO IMPLEMENTATION: This “practice changer” conflicts with NCEP guidelines

The current NCEP ATP III guidelines recommend combination fibrate-statin therapy for all patients when statin therapy alone is not adequate to achieve lipid goals. This is a major challenge to our recommendation against using this combination for most patients with diabetes. Some physicians may choose to follow the ATP III guidelines rather than the new evidence because they feel more confident adhering to national guidelines.

Clinical inertia is another challenge, as clinicians may be hesitant to stop therapy in patients already on a fibrate-statin combination. Finally, specialists may continue to use fibrate-statin combinations in all patients with diabetes who do not achieve lipid goals on a statin, and family physicians may hesitate to contradict their recommendations.

Acknowledgement
The PURLs Surveillance System is supported in part by Grant Number UL1RR024999 from the National Center for Research Resources; the grant is a Clinical Translational Science Award to the University of Chicago. The content is solely the responsibility of the authors and does not necessarily represent the official views of the National Center for Research Resources or the National Institutes of Health.

Click here to view PURL METHODOLOGY

References

1. The ACCORD Study Group. Effects of combination lipid therapy in type 2 diabetes mellitus. N Engl J Med. 2010;362:1563-1574.

2. National Cholesterol Education Program (NCEP) Expert Panel on Detection, Evaluation, and Treatment of High Blood Cholesterol in Adults (Adult Treatment Panel III). Third Report of the National Cholesterol Education Program (NCEP) Expert Panel on Detection, Evaluation, and Treatment of High Blood Cholesterol in Adults (Adult Treatment Panel III) final report. Circulation. 2002;106:3143-3421.

3. Rubins HB, Robins SJ, Collins D, et al. Diabetes, plasma insulin, and cardiovascular disease: subgroup analysis from the Department of Veterans Affairs high-density lipoprotein intervention trial (VA-HIT). Arch Intern Med. 2002;162:2597-2604.

4. Scott R, O’Brien R, Fulcher G, et al. Fenofibrate intervention and event lowering in diabetes (FIELD) study investigators. Effects of fenofibrate treatment on cardiovascular disease risk in 9,795 individuals with type 2 diabetes and various components of the metabolic syndrome: the Fenofibrate Intervention and Event Lowering in Diabetes (FIELD) study. Diabetes Care. 2009;32:493-498.

5. Buse JB, Bigger JT, et al. ACCORD Study Group Action to Control Cardiovascular Risk in Diabetes (ACCORD) trial: design and methods. Am J Cardiol. 2007;99:21i-33i.

Article PDF
Author and Disclosure Information

Jody Lounsbery, PharmD, BCPS
North Memorial Family Medicine Residency, University of Minnesota, Minneapolis

Shailendra Prasad, MD, MBBS, MPH
North Memorial Family Medicine Residency, University of Minnesota, Minneapolis

Kate Rowland, MD
Department of Family Medicine, University of Chicago

PURLs EDITOR
John Hickner, MD, MSc
Cleveland Clinic

Issue
The Journal of Family Practice - 59(10)
Publications
Topics
Page Number
582-584
Legacy Keywords
Jody Lounsbery; PURLs; add a fibrate; statin; diabetes and dyslipidemia; cardiovascular outcomes
Sections
Author and Disclosure Information

Jody Lounsbery, PharmD, BCPS
North Memorial Family Medicine Residency, University of Minnesota, Minneapolis

Shailendra Prasad, MD, MBBS, MPH
North Memorial Family Medicine Residency, University of Minnesota, Minneapolis

Kate Rowland, MD
Department of Family Medicine, University of Chicago

PURLs EDITOR
John Hickner, MD, MSc
Cleveland Clinic

Author and Disclosure Information

Jody Lounsbery, PharmD, BCPS
North Memorial Family Medicine Residency, University of Minnesota, Minneapolis

Shailendra Prasad, MD, MBBS, MPH
North Memorial Family Medicine Residency, University of Minnesota, Minneapolis

Kate Rowland, MD
Department of Family Medicine, University of Chicago

PURLs EDITOR
John Hickner, MD, MSc
Cleveland Clinic

Article PDF
Article PDF
PRACTICE CHANGER

Do not routinely add a fibrate to a statin for patients with type 2 diabetes who are at high risk for cardiovascular events.1

STRENGTH OF RECOMMENDATION

B: Based on a good-quality randomized controlled trial.

The ACCORD Study Group. Effects of combination lipid therapy in type 2 diabetes mellitus. N Engl J Med. 2010;362:1563-1574.

 

ILLUSTRATIVE CASE

A 60-year-old man with cardiovascular disease and diabetes comes to your clinic for a routine check of his dyslipidemia, for which he is on statin therapy. His fasting lipid panel shows a low-density lipoprotein (LDL) of 70 mg/dL, triglycerides of 200 mg/dL, and a high-density lipoprotein (HDL) of 30 mg/dL. Should you recommend adding fenofibrate?

Patients with type 2 diabetes are at increased risk for cardiovascular events. National Cholesterol Education Program Adult Treatment Panel (NCEP ATP) III guidelines recommend treatment of dyslipidemia for all patients with high risk of cardiovascular events to an LDL goal of <100 mg/dL (optional <70 mg/dL), HDL goal of >40 mg/dL, and triglyceride goal of <150 mg/dL. These recommendations include the use of combination therapy with a statin and fenofibrate for patients who have elevated triglycerides and low HDL cholesterol despite being on statin therapy alone.2

Survival benefit of combo therapy remains unproven
We know that fibrate therapy alone in patients with type 2 diabetes reduces major cardiovascular events.3,4 We also know that adding a fibrate to statin therapy can help patients reach their HDL and triglyceride targets. However, the survival benefit of the fibrate-statin combination over that of a statin alone has not been proven. In addition, there have been concerns about the increased risk of adverse effects with the combination. In fact, the overall benefits (and risks) of combining fibrates and statins for patients with diabetes and dyslipidemia were not addressed in a large randomized trial until the study we report on here.

STUDY SUMMARY: Statin + fibrate = minimal benefit for most patients

The Action to Control Cardiovascular Risk in Diabetes (ACCORD) study is among the largest trials conducted in adults with type 2 diabetes at high risk of cardiovascular events.5 The study examined 3 approaches to lowering the risk of major cardiovascular events: intensive lowering of blood sugar levels compared with standard blood sugar treatment; intensive lowering of blood pressure (BP) compared with standard BP treatment; and treatment of lipids with 2 drugs—a fibrate plus a statin—compared with a statin alone. This summary focuses on the lipid arm of the ACCORD study.1

All patients in the study had type 2 diabetes and a hemoglobin A1c ≥7.5%. The study included patients ages 40 to 79 years with clinical evidence of cardiovascular disease and patients ages 55 to 79 years with either subclinical cardiovascular disease or ≥2 cardiovascular risk factors in addition to diabetes.

The lipid arm enrolled patients who had an LDL cholesterol of 60 to 180 mg/dL, an HDL cholesterol <55 mg/dL for women and blacks and <50 mg/dL for all other groups, and a triglyceride level <750 mg/dL for those not receiving lipid therapy and <400 mg/dL for those on lipid therapy. Enrollees (N=5518) were started on open-label simvastatin 20 mg, titrated up as needed to reach the LDL goal, then randomized to receive either fenofibrate or placebo 1 month later. The mean duration of follow-up was 4.7 years.

The primary outcome was the first occurrence of a major cardiovascular event— nonfatal myocardial infarction (MI), nonfatal stroke, or death from a cardiovascular cause. The annual rate of the primary outcome was 2.2% (n=291) in the fenofibrate-statin group and 2.4% (n=310) in the placebo group, a nonsignificant difference (P=.32).

The results were reported by sex. The primary outcome rate for men during the 4.7-year follow-up was 11.2% in the fenofibrate-statin group vs 13.3% in the placebo group; for women, the outcome rates were 9.1% in the treatment group and 6.6% in the placebo group (P =.01). These rates suggest a small benefit for men, and harm for women.

Subgroup analysis showed additional benefit from fenofibrate in patients with a combination of a high baseline triglyceride level (≥204 mg/dL) and very low baseline HDL cholesterol (≤34 mg/dL), representing about 16% of the study participants. The primary outcome rate for patients in this subgroup was 12.4% in the fenofibrate-statin group and 17.3% in the placebo group (P=.057); number needed to treat (NNT)=20 patients for 4.7 years to prevent 1 major cardiovascular event.

Harm was similar in both groups. A small number of patients had elevations of alanine aminotransferase of >3 times the upper limit of normal (1.9% in the fenofibrate-statin group and 1.5% in the statin group). The study drug was discontinued in 66 patients (2.4%) in the fenofibrate-statin group (and the placebo was discontinued in 30 patients [1.1%] in the statin group). The fenofibrate or statin dose was reduced in 440 patients (15.9%) in the fenofibrate-statin group and in 194 patients (7%) in the statin group due to a decrease in estimated glomerular filtration rate. There was no significant difference in the incidence of hemodialysis and end-stage renal disease (75 patients in the fenofibrate-statin group vs 77 patients in the statin group).

 

 

 

WHAT’S NEW: We have evidence that combo therapy doesn’t further reduce risk

This study examined a previously unaddressed question, the role of combination fibrate-statin therapy in high-risk patients with type 2 diabetes. The findings do not support the routine use of combination therapy compared with a statin alone for most patients with diabetes. Overall, combination therapy with simvastatin and fenofibrate did not lower the risk of MI, stroke, or death from cardiovascular disease more than simvastatin alone.

This trial showed that women with diabetes and hyperlipidemia should not be treated with both a statin and a fibrate. Men appeared to have a very small benefit from combination therapy (NNT=50). Patients with a baseline HDL ≤34 mg/dL and baseline triglyceride ≥204 mg/dL appeared to benefit from the combination, but this group constituted only 16% of the patients in this trial and the difference had borderline statistical significance. Nonetheless, it may be reasonable to treat such patients with combination therapy until a definitive study is done.

CAVEATS: Statin dose did not match standard practice

This study used a low dose of statin. The average daily simvastatin dose was 22.3 mg in the fenofibrate-statin group and 22.4 mg in the placebo group. This constitutes low-dose therapy compared with doses routinely used in practice (ie, 40 or 80 mg). A higher dose of simvastatin may have negated any outcome differences.

CHALLENGES TO IMPLEMENTATION: This “practice changer” conflicts with NCEP guidelines

The current NCEP ATP III guidelines recommend combination fibrate-statin therapy for all patients when statin therapy alone is not adequate to achieve lipid goals. This is a major challenge to our recommendation against using this combination for most patients with diabetes. Some physicians may choose to follow the ATP III guidelines rather than the new evidence because they feel more confident adhering to national guidelines.

Clinical inertia is another challenge, as clinicians may be hesitant to stop therapy in patients already on a fibrate-statin combination. Finally, specialists may continue to use fibrate-statin combinations in all patients with diabetes who do not achieve lipid goals on a statin, and family physicians may hesitate to contradict their recommendations.

Acknowledgement
The PURLs Surveillance System is supported in part by Grant Number UL1RR024999 from the National Center for Research Resources; the grant is a Clinical Translational Science Award to the University of Chicago. The content is solely the responsibility of the authors and does not necessarily represent the official views of the National Center for Research Resources or the National Institutes of Health.

Click here to view PURL METHODOLOGY

PRACTICE CHANGER

Do not routinely add a fibrate to a statin for patients with type 2 diabetes who are at high risk for cardiovascular events.1

STRENGTH OF RECOMMENDATION

B: Based on a good-quality randomized controlled trial.

The ACCORD Study Group. Effects of combination lipid therapy in type 2 diabetes mellitus. N Engl J Med. 2010;362:1563-1574.

 

ILLUSTRATIVE CASE

A 60-year-old man with cardiovascular disease and diabetes comes to your clinic for a routine check of his dyslipidemia, for which he is on statin therapy. His fasting lipid panel shows a low-density lipoprotein (LDL) of 70 mg/dL, triglycerides of 200 mg/dL, and a high-density lipoprotein (HDL) of 30 mg/dL. Should you recommend adding fenofibrate?

Patients with type 2 diabetes are at increased risk for cardiovascular events. National Cholesterol Education Program Adult Treatment Panel (NCEP ATP) III guidelines recommend treatment of dyslipidemia for all patients with high risk of cardiovascular events to an LDL goal of <100 mg/dL (optional <70 mg/dL), HDL goal of >40 mg/dL, and triglyceride goal of <150 mg/dL. These recommendations include the use of combination therapy with a statin and fenofibrate for patients who have elevated triglycerides and low HDL cholesterol despite being on statin therapy alone.2

Survival benefit of combo therapy remains unproven
We know that fibrate therapy alone in patients with type 2 diabetes reduces major cardiovascular events.3,4 We also know that adding a fibrate to statin therapy can help patients reach their HDL and triglyceride targets. However, the survival benefit of the fibrate-statin combination over that of a statin alone has not been proven. In addition, there have been concerns about the increased risk of adverse effects with the combination. In fact, the overall benefits (and risks) of combining fibrates and statins for patients with diabetes and dyslipidemia were not addressed in a large randomized trial until the study we report on here.

STUDY SUMMARY: Statin + fibrate = minimal benefit for most patients

The Action to Control Cardiovascular Risk in Diabetes (ACCORD) study is among the largest trials conducted in adults with type 2 diabetes at high risk of cardiovascular events.5 The study examined 3 approaches to lowering the risk of major cardiovascular events: intensive lowering of blood sugar levels compared with standard blood sugar treatment; intensive lowering of blood pressure (BP) compared with standard BP treatment; and treatment of lipids with 2 drugs—a fibrate plus a statin—compared with a statin alone. This summary focuses on the lipid arm of the ACCORD study.1

All patients in the study had type 2 diabetes and a hemoglobin A1c ≥7.5%. The study included patients ages 40 to 79 years with clinical evidence of cardiovascular disease and patients ages 55 to 79 years with either subclinical cardiovascular disease or ≥2 cardiovascular risk factors in addition to diabetes.

The lipid arm enrolled patients who had an LDL cholesterol of 60 to 180 mg/dL, an HDL cholesterol <55 mg/dL for women and blacks and <50 mg/dL for all other groups, and a triglyceride level <750 mg/dL for those not receiving lipid therapy and <400 mg/dL for those on lipid therapy. Enrollees (N=5518) were started on open-label simvastatin 20 mg, titrated up as needed to reach the LDL goal, then randomized to receive either fenofibrate or placebo 1 month later. The mean duration of follow-up was 4.7 years.

The primary outcome was the first occurrence of a major cardiovascular event— nonfatal myocardial infarction (MI), nonfatal stroke, or death from a cardiovascular cause. The annual rate of the primary outcome was 2.2% (n=291) in the fenofibrate-statin group and 2.4% (n=310) in the placebo group, a nonsignificant difference (P=.32).

The results were reported by sex. The primary outcome rate for men during the 4.7-year follow-up was 11.2% in the fenofibrate-statin group vs 13.3% in the placebo group; for women, the outcome rates were 9.1% in the treatment group and 6.6% in the placebo group (P =.01). These rates suggest a small benefit for men, and harm for women.

Subgroup analysis showed additional benefit from fenofibrate in patients with a combination of a high baseline triglyceride level (≥204 mg/dL) and very low baseline HDL cholesterol (≤34 mg/dL), representing about 16% of the study participants. The primary outcome rate for patients in this subgroup was 12.4% in the fenofibrate-statin group and 17.3% in the placebo group (P=.057); number needed to treat (NNT)=20 patients for 4.7 years to prevent 1 major cardiovascular event.

Harm was similar in both groups. A small number of patients had elevations of alanine aminotransferase of >3 times the upper limit of normal (1.9% in the fenofibrate-statin group and 1.5% in the statin group). The study drug was discontinued in 66 patients (2.4%) in the fenofibrate-statin group (and the placebo was discontinued in 30 patients [1.1%] in the statin group). The fenofibrate or statin dose was reduced in 440 patients (15.9%) in the fenofibrate-statin group and in 194 patients (7%) in the statin group due to a decrease in estimated glomerular filtration rate. There was no significant difference in the incidence of hemodialysis and end-stage renal disease (75 patients in the fenofibrate-statin group vs 77 patients in the statin group).

 

 

 

WHAT’S NEW: We have evidence that combo therapy doesn’t further reduce risk

This study examined a previously unaddressed question, the role of combination fibrate-statin therapy in high-risk patients with type 2 diabetes. The findings do not support the routine use of combination therapy compared with a statin alone for most patients with diabetes. Overall, combination therapy with simvastatin and fenofibrate did not lower the risk of MI, stroke, or death from cardiovascular disease more than simvastatin alone.

This trial showed that women with diabetes and hyperlipidemia should not be treated with both a statin and a fibrate. Men appeared to have a very small benefit from combination therapy (NNT=50). Patients with a baseline HDL ≤34 mg/dL and baseline triglyceride ≥204 mg/dL appeared to benefit from the combination, but this group constituted only 16% of the patients in this trial and the difference had borderline statistical significance. Nonetheless, it may be reasonable to treat such patients with combination therapy until a definitive study is done.

CAVEATS: Statin dose did not match standard practice

This study used a low dose of statin. The average daily simvastatin dose was 22.3 mg in the fenofibrate-statin group and 22.4 mg in the placebo group. This constitutes low-dose therapy compared with doses routinely used in practice (ie, 40 or 80 mg). A higher dose of simvastatin may have negated any outcome differences.

CHALLENGES TO IMPLEMENTATION: This “practice changer” conflicts with NCEP guidelines

The current NCEP ATP III guidelines recommend combination fibrate-statin therapy for all patients when statin therapy alone is not adequate to achieve lipid goals. This is a major challenge to our recommendation against using this combination for most patients with diabetes. Some physicians may choose to follow the ATP III guidelines rather than the new evidence because they feel more confident adhering to national guidelines.

Clinical inertia is another challenge, as clinicians may be hesitant to stop therapy in patients already on a fibrate-statin combination. Finally, specialists may continue to use fibrate-statin combinations in all patients with diabetes who do not achieve lipid goals on a statin, and family physicians may hesitate to contradict their recommendations.

Acknowledgement
The PURLs Surveillance System is supported in part by Grant Number UL1RR024999 from the National Center for Research Resources; the grant is a Clinical Translational Science Award to the University of Chicago. The content is solely the responsibility of the authors and does not necessarily represent the official views of the National Center for Research Resources or the National Institutes of Health.

Click here to view PURL METHODOLOGY

References

1. The ACCORD Study Group. Effects of combination lipid therapy in type 2 diabetes mellitus. N Engl J Med. 2010;362:1563-1574.

2. National Cholesterol Education Program (NCEP) Expert Panel on Detection, Evaluation, and Treatment of High Blood Cholesterol in Adults (Adult Treatment Panel III). Third Report of the National Cholesterol Education Program (NCEP) Expert Panel on Detection, Evaluation, and Treatment of High Blood Cholesterol in Adults (Adult Treatment Panel III) final report. Circulation. 2002;106:3143-3421.

3. Rubins HB, Robins SJ, Collins D, et al. Diabetes, plasma insulin, and cardiovascular disease: subgroup analysis from the Department of Veterans Affairs high-density lipoprotein intervention trial (VA-HIT). Arch Intern Med. 2002;162:2597-2604.

4. Scott R, O’Brien R, Fulcher G, et al. Fenofibrate intervention and event lowering in diabetes (FIELD) study investigators. Effects of fenofibrate treatment on cardiovascular disease risk in 9,795 individuals with type 2 diabetes and various components of the metabolic syndrome: the Fenofibrate Intervention and Event Lowering in Diabetes (FIELD) study. Diabetes Care. 2009;32:493-498.

5. Buse JB, Bigger JT, et al. ACCORD Study Group Action to Control Cardiovascular Risk in Diabetes (ACCORD) trial: design and methods. Am J Cardiol. 2007;99:21i-33i.

References

1. The ACCORD Study Group. Effects of combination lipid therapy in type 2 diabetes mellitus. N Engl J Med. 2010;362:1563-1574.

2. National Cholesterol Education Program (NCEP) Expert Panel on Detection, Evaluation, and Treatment of High Blood Cholesterol in Adults (Adult Treatment Panel III). Third Report of the National Cholesterol Education Program (NCEP) Expert Panel on Detection, Evaluation, and Treatment of High Blood Cholesterol in Adults (Adult Treatment Panel III) final report. Circulation. 2002;106:3143-3421.

3. Rubins HB, Robins SJ, Collins D, et al. Diabetes, plasma insulin, and cardiovascular disease: subgroup analysis from the Department of Veterans Affairs high-density lipoprotein intervention trial (VA-HIT). Arch Intern Med. 2002;162:2597-2604.

4. Scott R, O’Brien R, Fulcher G, et al. Fenofibrate intervention and event lowering in diabetes (FIELD) study investigators. Effects of fenofibrate treatment on cardiovascular disease risk in 9,795 individuals with type 2 diabetes and various components of the metabolic syndrome: the Fenofibrate Intervention and Event Lowering in Diabetes (FIELD) study. Diabetes Care. 2009;32:493-498.

5. Buse JB, Bigger JT, et al. ACCORD Study Group Action to Control Cardiovascular Risk in Diabetes (ACCORD) trial: design and methods. Am J Cardiol. 2007;99:21i-33i.

Issue
The Journal of Family Practice - 59(10)
Issue
The Journal of Family Practice - 59(10)
Page Number
582-584
Page Number
582-584
Publications
Publications
Topics
Article Type
Display Headline
Add a fibrate to a statin?
Display Headline
Add a fibrate to a statin?
Legacy Keywords
Jody Lounsbery; PURLs; add a fibrate; statin; diabetes and dyslipidemia; cardiovascular outcomes
Legacy Keywords
Jody Lounsbery; PURLs; add a fibrate; statin; diabetes and dyslipidemia; cardiovascular outcomes
Sections
PURLs Copyright

Copyright © 2010 The Family Physicians Inquiries Network.
All rights reserved.

Disallow All Ads
Alternative CME
Article PDF Media

PT or cervical collar for cervical radiculopathy?

Article Type
Changed
Mon, 01/14/2019 - 11:28
Display Headline
PT or cervical collar for cervical radiculopathy?
PRACTICE CHANGER

To shorten recovery time for adults with acute cervical radiculopathy, recommend either physical therapy (PT) and a home exercise plan or a cervical collar and rest.1 Both are more effective than a wait-and-see strategy.1

STRENGTH OF RECOMMENDATION

B: Based on a single well-done randomized controlled trial (RCT).

Kuijper B, Tans JT, Beelen A, et al. Cervical collar or physiotherapy versus wait and see policy for recent onset cervical radiculopathy: randomized trial. BMJ. 2009;339:b3883.

 

Illustrative case

James M, a 43-year-old self-employed mechanic, came to see you 2 weeks ago because of neck pain radiating to his right shoulder, arm, forearm, and dorsum of his hand. You diagnosed acute right-sided cervical radiculopathy and prescribed a nonsteroidal anti-inflammatory drug.

Today he’s back in your office, reporting that he has experienced only minimal transient relief. You reassure him that the pain will subside within a few months, but james wants to know if you can give him something to speed up his recovery and enable him to return to work.

Each year in the United States, approximately 85 out of every 100,000 adults develop cervical radiculopathy2—a neurologic condition characterized by dysfunction of a cervical spinal nerve, the roots of the nerve, or both. In addition to pain in the neck and the arm on the affected side, patients often develop sensory loss, loss of motor function, and/or reflex changes in the affected nerve-root distribution.

Most patients respond to conservative measures
A nonsurgical approach is the preferred first-line treatment strategy for cervical radiculopathy.3 The Bone and Joint Decade 2000-2010 Task Force on Neck Pain and Its Associated Disorders—an international network of experts in a number of specialties—found no evidence that surgery provides better long-term outcomes than more conservative treatment.3 Approximately 80% to 90% of patients respond to a conservative approach, with improvements in pain, function, and mood in 3 to 6 months.4,5

There are numerous conservative therapies for cervical radiculopathy, including oral analgesics, rest, cervical traction, short-term immobilization with a cervical collar, PT, a short course of oral corticosteroids, and perineural steroid injections.4-6 These therapies may be used singly or in combination. Until now, however, no high-quality RCTs compared the efficacy of various nonsurgical treatment modalities for acute cervical radiculopathy—and their effectiveness is still subject to debate.

STUDY SUMMARY: Initially, both Tx modes beat wait-and-see

The study by Kuijper et al1 is the first RCT to compare the effectiveness of PT, cervical collars, and a wait-and-see strategy in alleviating symptoms of cervical radiculopathy. Enrollees (N=205) were men and women ages 18 to 75 years who were referred by general practitioners in 3 Dutch hospitals. All the participants had a diagnosis of cervical radiculopathy confirmed by a neurologist. In addition, all the cases were of recent onset, with symptoms of <1 month’s duration at the time of enrollment. Patients with clinical signs of cord compression and those who had previously been treated with either PT or a cervical collar for this episode were excluded.

The researchers randomized the participants into 3 groups: PT, cervical collar, or control. All the groups were comparable at baseline.

Those in the PT group received twice weekly therapy for 6 weeks, with a focus on mobilizing and stabilizing the cervical spine. They were also taught to perform home exercises and advised to do the exercises daily.

Patients in the cervical collar group were given a semi-hard, snugly fitted collar and instructed to wear it during the day for 3 weeks—and to rest as much as possible. They were weaned from the collar over the course of another 3 weeks.

Participants in the control group were simply told to follow their normal daily routine as much as possible. All 3 groups were permitted to take oral pain medication as needed.

The primary outcome measures were changes over time in neck and arm pain scores, using 2 validated measurement tools: a 100-mm visual analog scale (VAS) and a 100-point neck disability index (NDI). Both tools were used at 3 weeks, 6 weeks, and 6 months. Secondary outcomes were treatment satisfaction (as measured on a 5-point scale), use of opiates, and working status.

 

 

 

By 6 months, differences virtually disappeared
Both the active and passive interventions reduced arm and neck pain faster than the wait-and-see strategy. At 6 weeks, participants in both the PT and cervical collar groups reported a 31-mm reduction in arm pain (P=.007 and .006, respectively), compared with a 19-mm reduction for those in the control group (P=.006). This is a clinically meaningful difference.

The rate of reduction in neck pain over the first 6 weeks was: PT group, 2.4 mm/week, P=.002; cervical collar group, 2.8 mm/week, P=<.001; and control group, 0.9 mm/week. The rate of reduction in the NDI was 1.4 points per week for the control group vs 2.3 points per week for the cervical collar group (P= .024). The PT group fared no better on the NDI measure than the control group. This may reflect the fact that the index predominantly measured disability caused by neck pain, whereas arm pain scores,—which were highest initially—showed the greatest improvement, the authors note.

At 6 months, pain and disability had almost resolved for all the patients, regardless of their treatment group, and secondary outcomes—treatment satisfaction, analgesic use, and working status—were similar for all 3 groups.

WHAT'S NEW: High-quality RCT supports PT and cervical collar

Some investigators have advocated the short-term use of immobilization with either a cervical collar or a cervical pillow during sleep. Until now, however, there was no conclusive evidence about the benefits of this approach.

One earlier RCT (N=493) compared 5 treatment modalities—traction, positioning, collar, placebo tablets, and heat treatment—and found no significant difference in pain and ability to work.7 That trial was done nearly 15 years ago, however, and the investigators did not use validated outcome scales. Therefore, the trial would not meet current RCT standards.

The study we report on here leaves little doubt that the 2 treatments reviewed—PT and cervical collar—provide more rapid relief than a wait-and-see approach.

CAVEATS: Pain meds still needed, unanswered questions remain

Although the cervical collar and PT groups had less pain at 3 and 6 weeks compared with the controls—and all 3 groups showed equal improvement at study’s end—the researchers found little difference in use of analgesics. Data on adherence to treatment was recorded by patients, so treatment adherence may not be completely accurate.8

Patients without severe arm pain or signs of muscle weakness were not included in this study, so we don’t know whether individuals with less severe cervical radiculopathy would benefit from these treatments. What’s more, this study focused only on new cases of acute cervical radiculopathy, and the findings may not apply to patients with chronic, recurrent, or persistent symptoms.

The apparent contradiction in the finding that both immobilization and PT are beneficial does not have a clear scientific explanation. The researchers hypothesize that immobilizing the neck with a collar reduces foraminal root compression and inflammation; this could explain the larger reduction in arm pain compared with neck pain and neck disability found in this study. The mechanism of pain reduction with PT is unclear, although it is probably related to the restoration of the neck musculature’s strength and range of motion.

Cost is another issue. A cervical collar and rest is at least as effective as PT for recent onset cervical radiculopathy, but the collar costs only about $20—far less than the cost of 12 sessions of therapy.

One final caveat: Any patient with persistent or worsening symptoms should undergo additional evaluation, including imaging.

CHALLENGES TO IMPLEMENTATION: Rest is contrary to usual approach

Some physicians may not agree with the recommendation to encourage rest. Indeed, rest and immobilization are contrary to the usual recommendation for musculoskeletal injuries—to resume activity as soon as possible.

Patients might not like wearing a collar for a variety of personal reasons, such as cosmetic appearance or limitations of motion. On the other hand, some patients may feel that their pain is too severe to be able to participate in PT—which may also be too expensive for, or not readily available to, some patients.

Acknowledgement
The PURLs Surveillance System is supported in part by Grant number UL1RR024999 from the National Center for Research Resources; the grant was a Clinical Translational Science Award to the University of Chicago. The content is solely the responsibility of the authors and does not necessarily represent the official views of the National Center for Research Resources or the National Institutes of Health.

Click here to view PURL METHODOLOGY

References

1. Kuijper B, Tans JT, Beelen A, et al. Cervical collar or physiotherapy versus wait and see policy for recent onset cervical radiculopathy: randomized trial. BMJ 2009;339:b3883.-

2. Radhakrishan K, Litchy WJ, O’Fallon WM, et al. Epidemiology of cervical radiculopathy: a population-based study from Rochester, Minnesota, 1976 through 1990 Brain 1994;117:325-335.

3. Nordin M, Carragee EJ, Hogg-Johnson S, et al. Assessment of neck pain and its associated disorders: results of the Bone and Joint Decade 2000-2010 Task Force on Neck Pain and Its Associated Disorders. Spine 2008;33(suppl 4):S101-S122.

4. Saal JS, Saal JA, Yurth EF. Nonoperative management of herniated cervical intervertebral disc with radiculopathy. Spine 1996;21:1877-1883.

5. Persson LC, Carlsson CA, Carlsson JY. Long-lasting cervical radicular pain managed with surgery, physiotherapy, or a cervical collar: a prospective, randomized study. Spine 1997;22:751-758.

6. Wolff MW, Levine LA. Cervical radiculopathies: conservative approaches to management. Phys Med Rehabil Clin N Am 2002;13:589-608.

7. Levine MJ, Albert TJ, Smith MD. Cervical radiculopathy: diagnosis and nonoperative management. J Am Acad Orthop Surg. 1996;4:305-316.

8. Wainner RS, Fritz JM, Irrgang JJ, et al. Reliability and diagnostic accuracy of the clinical examination and patient self-report measures for cervical radiculopathy. Spine 2003;28:52-62.

Article PDF
Author and Disclosure Information

Mariya Dmytriv, MD
Kate Rowland, MD
Thomas Gavagan, MD, MPH
David Holub, MD, FAAFP
Department of Family Medicine, The University of Chicago

PURLs EDITOR
John Hickner, MD, MSc
Department of Family Medicine, Cleveland Clinic

Issue
The Journal of Family Practice - 59(5)
Publications
Topics
Page Number
269-272
Sections
Author and Disclosure Information

Mariya Dmytriv, MD
Kate Rowland, MD
Thomas Gavagan, MD, MPH
David Holub, MD, FAAFP
Department of Family Medicine, The University of Chicago

PURLs EDITOR
John Hickner, MD, MSc
Department of Family Medicine, Cleveland Clinic

Author and Disclosure Information

Mariya Dmytriv, MD
Kate Rowland, MD
Thomas Gavagan, MD, MPH
David Holub, MD, FAAFP
Department of Family Medicine, The University of Chicago

PURLs EDITOR
John Hickner, MD, MSc
Department of Family Medicine, Cleveland Clinic

Article PDF
Article PDF
PRACTICE CHANGER

To shorten recovery time for adults with acute cervical radiculopathy, recommend either physical therapy (PT) and a home exercise plan or a cervical collar and rest.1 Both are more effective than a wait-and-see strategy.1

STRENGTH OF RECOMMENDATION

B: Based on a single well-done randomized controlled trial (RCT).

Kuijper B, Tans JT, Beelen A, et al. Cervical collar or physiotherapy versus wait and see policy for recent onset cervical radiculopathy: randomized trial. BMJ. 2009;339:b3883.

 

Illustrative case

James M, a 43-year-old self-employed mechanic, came to see you 2 weeks ago because of neck pain radiating to his right shoulder, arm, forearm, and dorsum of his hand. You diagnosed acute right-sided cervical radiculopathy and prescribed a nonsteroidal anti-inflammatory drug.

Today he’s back in your office, reporting that he has experienced only minimal transient relief. You reassure him that the pain will subside within a few months, but james wants to know if you can give him something to speed up his recovery and enable him to return to work.

Each year in the United States, approximately 85 out of every 100,000 adults develop cervical radiculopathy2—a neurologic condition characterized by dysfunction of a cervical spinal nerve, the roots of the nerve, or both. In addition to pain in the neck and the arm on the affected side, patients often develop sensory loss, loss of motor function, and/or reflex changes in the affected nerve-root distribution.

Most patients respond to conservative measures
A nonsurgical approach is the preferred first-line treatment strategy for cervical radiculopathy.3 The Bone and Joint Decade 2000-2010 Task Force on Neck Pain and Its Associated Disorders—an international network of experts in a number of specialties—found no evidence that surgery provides better long-term outcomes than more conservative treatment.3 Approximately 80% to 90% of patients respond to a conservative approach, with improvements in pain, function, and mood in 3 to 6 months.4,5

There are numerous conservative therapies for cervical radiculopathy, including oral analgesics, rest, cervical traction, short-term immobilization with a cervical collar, PT, a short course of oral corticosteroids, and perineural steroid injections.4-6 These therapies may be used singly or in combination. Until now, however, no high-quality RCTs compared the efficacy of various nonsurgical treatment modalities for acute cervical radiculopathy—and their effectiveness is still subject to debate.

STUDY SUMMARY: Initially, both Tx modes beat wait-and-see

The study by Kuijper et al1 is the first RCT to compare the effectiveness of PT, cervical collars, and a wait-and-see strategy in alleviating symptoms of cervical radiculopathy. Enrollees (N=205) were men and women ages 18 to 75 years who were referred by general practitioners in 3 Dutch hospitals. All the participants had a diagnosis of cervical radiculopathy confirmed by a neurologist. In addition, all the cases were of recent onset, with symptoms of <1 month’s duration at the time of enrollment. Patients with clinical signs of cord compression and those who had previously been treated with either PT or a cervical collar for this episode were excluded.

The researchers randomized the participants into 3 groups: PT, cervical collar, or control. All the groups were comparable at baseline.

Those in the PT group received twice weekly therapy for 6 weeks, with a focus on mobilizing and stabilizing the cervical spine. They were also taught to perform home exercises and advised to do the exercises daily.

Patients in the cervical collar group were given a semi-hard, snugly fitted collar and instructed to wear it during the day for 3 weeks—and to rest as much as possible. They were weaned from the collar over the course of another 3 weeks.

Participants in the control group were simply told to follow their normal daily routine as much as possible. All 3 groups were permitted to take oral pain medication as needed.

The primary outcome measures were changes over time in neck and arm pain scores, using 2 validated measurement tools: a 100-mm visual analog scale (VAS) and a 100-point neck disability index (NDI). Both tools were used at 3 weeks, 6 weeks, and 6 months. Secondary outcomes were treatment satisfaction (as measured on a 5-point scale), use of opiates, and working status.

 

 

 

By 6 months, differences virtually disappeared
Both the active and passive interventions reduced arm and neck pain faster than the wait-and-see strategy. At 6 weeks, participants in both the PT and cervical collar groups reported a 31-mm reduction in arm pain (P=.007 and .006, respectively), compared with a 19-mm reduction for those in the control group (P=.006). This is a clinically meaningful difference.

The rate of reduction in neck pain over the first 6 weeks was: PT group, 2.4 mm/week, P=.002; cervical collar group, 2.8 mm/week, P=<.001; and control group, 0.9 mm/week. The rate of reduction in the NDI was 1.4 points per week for the control group vs 2.3 points per week for the cervical collar group (P= .024). The PT group fared no better on the NDI measure than the control group. This may reflect the fact that the index predominantly measured disability caused by neck pain, whereas arm pain scores,—which were highest initially—showed the greatest improvement, the authors note.

At 6 months, pain and disability had almost resolved for all the patients, regardless of their treatment group, and secondary outcomes—treatment satisfaction, analgesic use, and working status—were similar for all 3 groups.

WHAT'S NEW: High-quality RCT supports PT and cervical collar

Some investigators have advocated the short-term use of immobilization with either a cervical collar or a cervical pillow during sleep. Until now, however, there was no conclusive evidence about the benefits of this approach.

One earlier RCT (N=493) compared 5 treatment modalities—traction, positioning, collar, placebo tablets, and heat treatment—and found no significant difference in pain and ability to work.7 That trial was done nearly 15 years ago, however, and the investigators did not use validated outcome scales. Therefore, the trial would not meet current RCT standards.

The study we report on here leaves little doubt that the 2 treatments reviewed—PT and cervical collar—provide more rapid relief than a wait-and-see approach.

CAVEATS: Pain meds still needed, unanswered questions remain

Although the cervical collar and PT groups had less pain at 3 and 6 weeks compared with the controls—and all 3 groups showed equal improvement at study’s end—the researchers found little difference in use of analgesics. Data on adherence to treatment was recorded by patients, so treatment adherence may not be completely accurate.8

Patients without severe arm pain or signs of muscle weakness were not included in this study, so we don’t know whether individuals with less severe cervical radiculopathy would benefit from these treatments. What’s more, this study focused only on new cases of acute cervical radiculopathy, and the findings may not apply to patients with chronic, recurrent, or persistent symptoms.

The apparent contradiction in the finding that both immobilization and PT are beneficial does not have a clear scientific explanation. The researchers hypothesize that immobilizing the neck with a collar reduces foraminal root compression and inflammation; this could explain the larger reduction in arm pain compared with neck pain and neck disability found in this study. The mechanism of pain reduction with PT is unclear, although it is probably related to the restoration of the neck musculature’s strength and range of motion.

Cost is another issue. A cervical collar and rest is at least as effective as PT for recent onset cervical radiculopathy, but the collar costs only about $20—far less than the cost of 12 sessions of therapy.

One final caveat: Any patient with persistent or worsening symptoms should undergo additional evaluation, including imaging.

CHALLENGES TO IMPLEMENTATION: Rest is contrary to usual approach

Some physicians may not agree with the recommendation to encourage rest. Indeed, rest and immobilization are contrary to the usual recommendation for musculoskeletal injuries—to resume activity as soon as possible.

Patients might not like wearing a collar for a variety of personal reasons, such as cosmetic appearance or limitations of motion. On the other hand, some patients may feel that their pain is too severe to be able to participate in PT—which may also be too expensive for, or not readily available to, some patients.

Acknowledgement
The PURLs Surveillance System is supported in part by Grant number UL1RR024999 from the National Center for Research Resources; the grant was a Clinical Translational Science Award to the University of Chicago. The content is solely the responsibility of the authors and does not necessarily represent the official views of the National Center for Research Resources or the National Institutes of Health.

Click here to view PURL METHODOLOGY

PRACTICE CHANGER

To shorten recovery time for adults with acute cervical radiculopathy, recommend either physical therapy (PT) and a home exercise plan or a cervical collar and rest.1 Both are more effective than a wait-and-see strategy.1

STRENGTH OF RECOMMENDATION

B: Based on a single well-done randomized controlled trial (RCT).

Kuijper B, Tans JT, Beelen A, et al. Cervical collar or physiotherapy versus wait and see policy for recent onset cervical radiculopathy: randomized trial. BMJ. 2009;339:b3883.

 

Illustrative case

James M, a 43-year-old self-employed mechanic, came to see you 2 weeks ago because of neck pain radiating to his right shoulder, arm, forearm, and dorsum of his hand. You diagnosed acute right-sided cervical radiculopathy and prescribed a nonsteroidal anti-inflammatory drug.

Today he’s back in your office, reporting that he has experienced only minimal transient relief. You reassure him that the pain will subside within a few months, but james wants to know if you can give him something to speed up his recovery and enable him to return to work.

Each year in the United States, approximately 85 out of every 100,000 adults develop cervical radiculopathy2—a neurologic condition characterized by dysfunction of a cervical spinal nerve, the roots of the nerve, or both. In addition to pain in the neck and the arm on the affected side, patients often develop sensory loss, loss of motor function, and/or reflex changes in the affected nerve-root distribution.

Most patients respond to conservative measures
A nonsurgical approach is the preferred first-line treatment strategy for cervical radiculopathy.3 The Bone and Joint Decade 2000-2010 Task Force on Neck Pain and Its Associated Disorders—an international network of experts in a number of specialties—found no evidence that surgery provides better long-term outcomes than more conservative treatment.3 Approximately 80% to 90% of patients respond to a conservative approach, with improvements in pain, function, and mood in 3 to 6 months.4,5

There are numerous conservative therapies for cervical radiculopathy, including oral analgesics, rest, cervical traction, short-term immobilization with a cervical collar, PT, a short course of oral corticosteroids, and perineural steroid injections.4-6 These therapies may be used singly or in combination. Until now, however, no high-quality RCTs compared the efficacy of various nonsurgical treatment modalities for acute cervical radiculopathy—and their effectiveness is still subject to debate.

STUDY SUMMARY: Initially, both Tx modes beat wait-and-see

The study by Kuijper et al1 is the first RCT to compare the effectiveness of PT, cervical collars, and a wait-and-see strategy in alleviating symptoms of cervical radiculopathy. Enrollees (N=205) were men and women ages 18 to 75 years who were referred by general practitioners in 3 Dutch hospitals. All the participants had a diagnosis of cervical radiculopathy confirmed by a neurologist. In addition, all the cases were of recent onset, with symptoms of <1 month’s duration at the time of enrollment. Patients with clinical signs of cord compression and those who had previously been treated with either PT or a cervical collar for this episode were excluded.

The researchers randomized the participants into 3 groups: PT, cervical collar, or control. All the groups were comparable at baseline.

Those in the PT group received twice weekly therapy for 6 weeks, with a focus on mobilizing and stabilizing the cervical spine. They were also taught to perform home exercises and advised to do the exercises daily.

Patients in the cervical collar group were given a semi-hard, snugly fitted collar and instructed to wear it during the day for 3 weeks—and to rest as much as possible. They were weaned from the collar over the course of another 3 weeks.

Participants in the control group were simply told to follow their normal daily routine as much as possible. All 3 groups were permitted to take oral pain medication as needed.

The primary outcome measures were changes over time in neck and arm pain scores, using 2 validated measurement tools: a 100-mm visual analog scale (VAS) and a 100-point neck disability index (NDI). Both tools were used at 3 weeks, 6 weeks, and 6 months. Secondary outcomes were treatment satisfaction (as measured on a 5-point scale), use of opiates, and working status.

 

 

 

By 6 months, differences virtually disappeared
Both the active and passive interventions reduced arm and neck pain faster than the wait-and-see strategy. At 6 weeks, participants in both the PT and cervical collar groups reported a 31-mm reduction in arm pain (P=.007 and .006, respectively), compared with a 19-mm reduction for those in the control group (P=.006). This is a clinically meaningful difference.

The rate of reduction in neck pain over the first 6 weeks was: PT group, 2.4 mm/week, P=.002; cervical collar group, 2.8 mm/week, P=<.001; and control group, 0.9 mm/week. The rate of reduction in the NDI was 1.4 points per week for the control group vs 2.3 points per week for the cervical collar group (P= .024). The PT group fared no better on the NDI measure than the control group. This may reflect the fact that the index predominantly measured disability caused by neck pain, whereas arm pain scores,—which were highest initially—showed the greatest improvement, the authors note.

At 6 months, pain and disability had almost resolved for all the patients, regardless of their treatment group, and secondary outcomes—treatment satisfaction, analgesic use, and working status—were similar for all 3 groups.

WHAT'S NEW: High-quality RCT supports PT and cervical collar

Some investigators have advocated the short-term use of immobilization with either a cervical collar or a cervical pillow during sleep. Until now, however, there was no conclusive evidence about the benefits of this approach.

One earlier RCT (N=493) compared 5 treatment modalities—traction, positioning, collar, placebo tablets, and heat treatment—and found no significant difference in pain and ability to work.7 That trial was done nearly 15 years ago, however, and the investigators did not use validated outcome scales. Therefore, the trial would not meet current RCT standards.

The study we report on here leaves little doubt that the 2 treatments reviewed—PT and cervical collar—provide more rapid relief than a wait-and-see approach.

CAVEATS: Pain meds still needed, unanswered questions remain

Although the cervical collar and PT groups had less pain at 3 and 6 weeks compared with the controls—and all 3 groups showed equal improvement at study’s end—the researchers found little difference in use of analgesics. Data on adherence to treatment was recorded by patients, so treatment adherence may not be completely accurate.8

Patients without severe arm pain or signs of muscle weakness were not included in this study, so we don’t know whether individuals with less severe cervical radiculopathy would benefit from these treatments. What’s more, this study focused only on new cases of acute cervical radiculopathy, and the findings may not apply to patients with chronic, recurrent, or persistent symptoms.

The apparent contradiction in the finding that both immobilization and PT are beneficial does not have a clear scientific explanation. The researchers hypothesize that immobilizing the neck with a collar reduces foraminal root compression and inflammation; this could explain the larger reduction in arm pain compared with neck pain and neck disability found in this study. The mechanism of pain reduction with PT is unclear, although it is probably related to the restoration of the neck musculature’s strength and range of motion.

Cost is another issue. A cervical collar and rest is at least as effective as PT for recent onset cervical radiculopathy, but the collar costs only about $20—far less than the cost of 12 sessions of therapy.

One final caveat: Any patient with persistent or worsening symptoms should undergo additional evaluation, including imaging.

CHALLENGES TO IMPLEMENTATION: Rest is contrary to usual approach

Some physicians may not agree with the recommendation to encourage rest. Indeed, rest and immobilization are contrary to the usual recommendation for musculoskeletal injuries—to resume activity as soon as possible.

Patients might not like wearing a collar for a variety of personal reasons, such as cosmetic appearance or limitations of motion. On the other hand, some patients may feel that their pain is too severe to be able to participate in PT—which may also be too expensive for, or not readily available to, some patients.

Acknowledgement
The PURLs Surveillance System is supported in part by Grant number UL1RR024999 from the National Center for Research Resources; the grant was a Clinical Translational Science Award to the University of Chicago. The content is solely the responsibility of the authors and does not necessarily represent the official views of the National Center for Research Resources or the National Institutes of Health.

Click here to view PURL METHODOLOGY

References

1. Kuijper B, Tans JT, Beelen A, et al. Cervical collar or physiotherapy versus wait and see policy for recent onset cervical radiculopathy: randomized trial. BMJ 2009;339:b3883.-

2. Radhakrishan K, Litchy WJ, O’Fallon WM, et al. Epidemiology of cervical radiculopathy: a population-based study from Rochester, Minnesota, 1976 through 1990 Brain 1994;117:325-335.

3. Nordin M, Carragee EJ, Hogg-Johnson S, et al. Assessment of neck pain and its associated disorders: results of the Bone and Joint Decade 2000-2010 Task Force on Neck Pain and Its Associated Disorders. Spine 2008;33(suppl 4):S101-S122.

4. Saal JS, Saal JA, Yurth EF. Nonoperative management of herniated cervical intervertebral disc with radiculopathy. Spine 1996;21:1877-1883.

5. Persson LC, Carlsson CA, Carlsson JY. Long-lasting cervical radicular pain managed with surgery, physiotherapy, or a cervical collar: a prospective, randomized study. Spine 1997;22:751-758.

6. Wolff MW, Levine LA. Cervical radiculopathies: conservative approaches to management. Phys Med Rehabil Clin N Am 2002;13:589-608.

7. Levine MJ, Albert TJ, Smith MD. Cervical radiculopathy: diagnosis and nonoperative management. J Am Acad Orthop Surg. 1996;4:305-316.

8. Wainner RS, Fritz JM, Irrgang JJ, et al. Reliability and diagnostic accuracy of the clinical examination and patient self-report measures for cervical radiculopathy. Spine 2003;28:52-62.

References

1. Kuijper B, Tans JT, Beelen A, et al. Cervical collar or physiotherapy versus wait and see policy for recent onset cervical radiculopathy: randomized trial. BMJ 2009;339:b3883.-

2. Radhakrishan K, Litchy WJ, O’Fallon WM, et al. Epidemiology of cervical radiculopathy: a population-based study from Rochester, Minnesota, 1976 through 1990 Brain 1994;117:325-335.

3. Nordin M, Carragee EJ, Hogg-Johnson S, et al. Assessment of neck pain and its associated disorders: results of the Bone and Joint Decade 2000-2010 Task Force on Neck Pain and Its Associated Disorders. Spine 2008;33(suppl 4):S101-S122.

4. Saal JS, Saal JA, Yurth EF. Nonoperative management of herniated cervical intervertebral disc with radiculopathy. Spine 1996;21:1877-1883.

5. Persson LC, Carlsson CA, Carlsson JY. Long-lasting cervical radicular pain managed with surgery, physiotherapy, or a cervical collar: a prospective, randomized study. Spine 1997;22:751-758.

6. Wolff MW, Levine LA. Cervical radiculopathies: conservative approaches to management. Phys Med Rehabil Clin N Am 2002;13:589-608.

7. Levine MJ, Albert TJ, Smith MD. Cervical radiculopathy: diagnosis and nonoperative management. J Am Acad Orthop Surg. 1996;4:305-316.

8. Wainner RS, Fritz JM, Irrgang JJ, et al. Reliability and diagnostic accuracy of the clinical examination and patient self-report measures for cervical radiculopathy. Spine 2003;28:52-62.

Issue
The Journal of Family Practice - 59(5)
Issue
The Journal of Family Practice - 59(5)
Page Number
269-272
Page Number
269-272
Publications
Publications
Topics
Article Type
Display Headline
PT or cervical collar for cervical radiculopathy?
Display Headline
PT or cervical collar for cervical radiculopathy?
Sections
PURLs Copyright

Copyright © 2010 The Family Physicians Inquiries Network.
All rights reserved.

Disallow All Ads
Alternative CME
Article PDF Media

Start a statin prior to vascular surgery

Article Type
Changed
Mon, 01/14/2019 - 11:38
Display Headline
Start a statin prior to vascular surgery
PRACTICE CHANGER

HMG-CoA reductase inhibitors (statins), initiated 30 days before noncardiac vascular surgery, reduce the incidence of postoperative cardiac complications, including fatal myocardial infarction.1,2

STRENGTH OF RECOMMENDATION

A: 1 new randomized controlled trial (RCT), and 1 smaller, older RCT.

Schouten O, Boersma E, Hoeks S, et al. Fluvastatin and perioperative events in patients undergoing vascular surgery. N Engl J Med. 2009;361:980-989.

Durazzo AE, Machado FS, Ikeoka DT, et al. Reduction in cardiovascular events after vascular surgery with atorvastatin: a randomized trial. J Vasc Surg. 2004;39:967-975.

 

ILLUSTRATIVE CASE

A 67-year-old man with recurrent transient ischemic attacks comes in for a preoperative evaluation for carotid endarterectomy. The patient’s total cholesterol is 207 mg/dL and his low-density lipoprotein cholesterol (LDL-C) is 109 mg/dL. He takes metoprolol and lisinopril for hypertension.

Should you start him on a statin before surgery?

Nearly 25% of patients with peripheral vascular disease suffer from a cardiac event within 72 hours of elective, noncardiac vascular surgery.3 While most of these “complications” have minimal clinical impact and are detected by biochemical markers alone, some patients experience serious cardiac complications—including fatal myocardial infarction (MI).

That’s not surprising, given that most patients who require noncardiac vascular surgery suffer from severe coronary vascular disease.4 What is surprising is that most candidates for noncardiac vascular surgery are not put on statins prior to undergoing surgery.1,2,5

Statins were thought to increase—not prevent—complications
Until recently, taking statins during the perioperative period was believed to increase complications, including statin-associated myopathy. Indeed, guidelines from the American Heart Association (AHA), American College of Cardiology (ACC), and National Heart, Lung and Blood Institute (NHLBI) suggest that it is prudent to withhold statins during hospitalization for major surgery.6

1 small study hinted at value of perioperative statins
A small Brazilian trial conducted in 2004 called the AHA/ACC/NHLBI guidelines into question. the researchers studied 100 patients slated for noncardiac vascular surgery who were randomized to receive either 20 mg atorvastatin (Lipitor) or placebo preoperatively —and monitored them for cardiac events 6 months postoperatively. They found that the incidence of cardiac events (cardiac death, nonfatal MI, stroke, or unstable angina) was more than 3 times higher in the placebo group compared with patients receiving atorvastatin (26% vs 8%, number needed to treat [NNT]=5.6; P=.031).2

The results of this small single study, although suggestive, were not sufficiently convincing to change recommendations about the preoperative use of statins, however. A more comprehensive study was needed to alter standard practice, and the Schouten study that we report on below fits the bill.1

STUDY SUMMARY: Preoperative statin use cuts risk in half

Schouten et al followed 500 patients, who were randomized to receive either 80 mg extended-release fluvastatin (Lescol XL) or placebo for a median of 37 days prior to surgery.1 All enrollees were older than 40 years of age and were scheduled for noncardiac vascular surgery. the reasons for the surgery were abdominal aortic aneurysm repair (47.5%), lower limb arterial reconstruction (38.6%), or carotid artery endarterectomy (13.9%). Patients who were taking long-term beta-blocker therapy were continued on it; otherwise, bisoprolol 2.5 mg was initiated at the screening visit. Patients who were already taking statins (<50% of potential subjects) were excluded. Other exclusions were a contraindication to statin therapy; emergent surgery; and a repeat procedure within the last 29 days. Patients with unstable coronary artery disease or extensive stress-induced ischemia consistent with left main artery disease (or its equivalent) were also excluded.

The primary study outcome was myocardial ischemia, determined by continuous electrocardiogram (EKG) monitoring in the first 48 hours postsurgery and by 12-lead EKG recordings on days 3, 7, and 30. Troponin T levels were measured on postoperative days 1, 3, 7, and 30, as well. the principal secondary end point was either death from cardiovascular causes or nonfatal MI. MI was diagnosed by characteristic ischemic symptoms, with EKG evidence of ischemia or positive troponin T with characteristic rising and falling values.

To gauge fluvastatin’s effect on biomarkers, lipids, high-sensitivity C-reactive protein, and interleukin-6 were measured upon initiation of the medication and on the day of admission for surgery. Serum creatine kinase, alanine aminotransferase (ALT) levels, clinical myopathy, and rhabdomyolysis were monitored as safety measures, with levels measured prior to randomization, on the day of admission, and on postoperative days 1, 3, 7, and 30.

Both groups were similar in age (mean of 66 years), total serum cholesterol levels, risk factors for cardiac events, and medication use. About 75% of the enrollees were men. At baseline, 51% of the participants had a total cholesterol <213 mg/dL, and 39% had an LDL-C <116 mg/dL. Within 30 days after surgery, 27 (10.8%) of those in the fluvastatin group and 47 (19%) of patients in the placebo group had evidence of myocardial ischemia (hazard ratio=0.55; 95% confidence interval [CI], 0.34-0.88; P=.01). the NNT to prevent 1 patient from experiencing myocardial ischemia was 12.

 

 

 

Statin users had fewer MIs. A total of 6 patients receiving fluvastatin died, with 4 deaths attributed to cardiovascular causes. In the placebo group, 12 patients died, 8 of which were ascribed to cardiovascular causes. Eight patients in the fluvastatin group experienced nonfatal MIs, compared with 17 patients in the placebo group (NNT=19 to prevent 1 nonfatal MI or cardiac death (hazard ratio= 0.47; 95% CI, 0.24-0.94; P=.03).

Effects of statins were evident preoperatively. At the time of surgery, patients in the fluvastatin group had, on average, a 20% reduction in their total cholesterol and a 24% reduction in LDL-C; in the placebo group, total cholesterol had fallen by 4% and LDL-C, by 3%.

Patients receiving fluvastatin had an average 21% decrease in C-reactive protein, compared with a 3% increase for the placebo group. Interleukin-6 levels also were reduced far more in the fluvastatin group (33% vs a 4% reduction in the placebo group [P<.001]).

The medication was well tolerated. Overall, 6.8% of participants discontinued the study because of side effects, including 16 (6.4%) patients in the fluvastatin group and 18 (7.3%) in the placebo group. (After surgery, 115 [23.1%] of patients in the statin group temporarily discontinued the drug because of an inability to take oral medications for a median of 2 days.)

Rates of increase in creatine kinase of >10× the upper limit of normal (ULN) were similar between the fluvastatin and placebo groups (4% vs 3.2%, respectively). Increases in ALT to >3× ULN were more frequent in the placebo group compared with the fluvastatin group (5.3%, placebo; 3.2%, fluvastatin). No cases of myopathy or rhabdomyolysis were observed in either group.

WHAT’S NEW: Preop statins can be a lifesaver

The initiation of fluvastatin prior to vascular surgery reduced the incidence of cardiovascular events by 50%—a remarkable result. While patients at the highest risk were excluded from the study, those with lower cardiac risk nonetheless benefi ted from statin therapy. Experts have not typically recommended statins in the perioperative period for this patient population. the results of this study make it clear that they should.

CAVEATS: Extended-release formulation may have affected outcome

The statin used in this study was a longacting formulation, which may have protected patients who were unable to take oral medicines postoperatively. While we don’t know if the extended-release formulation made a difference in this study, we do know that atorvastatin was effective in the Brazilian study discussed earlier.

CHALLENGES TO IMPLEMENTATION: Preop statins may be overlooked

Not all patients see a primary care physician prior to undergoing vascular surgery. This means that it will sometimes be left to surgeons or other specialists to initiate statin therapy prior to surgery, and they may or may not do so.

Optimal timing is unknown. It is not clear how little time a patient scheduled for vascular surgery could spend on a statin and still reap these benefits. Nor do we know if the benefits would extend to patients undergoing other types of surgery; in a large study of patients undergoing all kinds of major noncardiac surgery, no benefits of perioperative statins were found.7

Adherence to the medication regimen presents another challenge, at least for some patients. In this case, however, we think the prospect of preventing major cardiac events postoperatively simply by taking statins for a month should be compelling enough to convince patients to take their medicine.

Acknowledgement
The PURLs Surveillance System is supported in part by Grant Number UL1RR024999 from the National Center for Research Resources; the grant is a Clinical Translational Science Award to the University of Chicago. The content is solely the responsibility of the authors and does not necessarily represent the ofcial views of either the National Center for Research Resources or the National Institutes of Health.

Click here to view PURL METHODOLOGY

References

1. Schouten O, Boersma E, Hoeks SE, et al. Fluvastatin and perioperative events in patients undergoing vascular surgery. N Engl J Med. 2009;361:980-989.

2. Durazzo AE, Machado FS, Ikeoka DT, et al. Reduction in cardiovascular events after vascular surgery with atorvastatin: a randomized trial. J Vasc Surg. 2004;39:967-975.

3. Pasternak RC, Smith SC, Jr, Bairey-Merz CN, et al. ACC/AHA/ NHLBI Clinical advisory on the use and safety of statins. Circulation. 2002;106:1024-1028.

4. Landesberg G, Shatz V, Akopnik I, et al. Association of cardiac troponin, CK-MB, and postoperative myocardial ischemia with long-term survival after major vascular surgery. J Am Coll Cardiol. 2003;42:1547-1554.

5. Hertzer NR, Beven EG, Young JR, et al. Coronary artery disease in peripheral vascular patients. A classification of 1000 coronary angiograms and results of surgical management. Ann Surg. 1984;199:223-233.

6. Brady AR, Gibbs JS, Greenhalgh RM, et al. Perioperative betablockade (POBBLE) for patients undergoing infrarenal vascular surgery: results of a randomized double-blind controlled trial. J Vasc Surg. 2005;41:602-609.

7. Dunkelgrun M, Boersma E, Schouten O, et al. Bisoprolol and fluvastatin for the reduction of perioperative cardiac mortality and myocardial infarction in intermediate-risk patients undergoing noncardiovascular surgery: a randomized controlled trial (DECREASE-IV). Ann Surg. 2009;249:921-926.

Article PDF
Author and Disclosure Information

Susan L. Pereira, MD
James J. Stevermer, MD, MSPH
Department of Family and Community Medicine, University of Missouri-Columbia

Kate Rowland, MD
Department of Family Medicine, University of Chicago

PURLs EDITOR
Bernard Ewigman, MD, MSPH
University of Chicago, Pritzker School of Medicine

Issue
The Journal of Family Practice - 59(2)
Publications
Topics
Page Number
108-110
Sections
Author and Disclosure Information

Susan L. Pereira, MD
James J. Stevermer, MD, MSPH
Department of Family and Community Medicine, University of Missouri-Columbia

Kate Rowland, MD
Department of Family Medicine, University of Chicago

PURLs EDITOR
Bernard Ewigman, MD, MSPH
University of Chicago, Pritzker School of Medicine

Author and Disclosure Information

Susan L. Pereira, MD
James J. Stevermer, MD, MSPH
Department of Family and Community Medicine, University of Missouri-Columbia

Kate Rowland, MD
Department of Family Medicine, University of Chicago

PURLs EDITOR
Bernard Ewigman, MD, MSPH
University of Chicago, Pritzker School of Medicine

Article PDF
Article PDF
PRACTICE CHANGER

HMG-CoA reductase inhibitors (statins), initiated 30 days before noncardiac vascular surgery, reduce the incidence of postoperative cardiac complications, including fatal myocardial infarction.1,2

STRENGTH OF RECOMMENDATION

A: 1 new randomized controlled trial (RCT), and 1 smaller, older RCT.

Schouten O, Boersma E, Hoeks S, et al. Fluvastatin and perioperative events in patients undergoing vascular surgery. N Engl J Med. 2009;361:980-989.

Durazzo AE, Machado FS, Ikeoka DT, et al. Reduction in cardiovascular events after vascular surgery with atorvastatin: a randomized trial. J Vasc Surg. 2004;39:967-975.

 

ILLUSTRATIVE CASE

A 67-year-old man with recurrent transient ischemic attacks comes in for a preoperative evaluation for carotid endarterectomy. The patient’s total cholesterol is 207 mg/dL and his low-density lipoprotein cholesterol (LDL-C) is 109 mg/dL. He takes metoprolol and lisinopril for hypertension.

Should you start him on a statin before surgery?

Nearly 25% of patients with peripheral vascular disease suffer from a cardiac event within 72 hours of elective, noncardiac vascular surgery.3 While most of these “complications” have minimal clinical impact and are detected by biochemical markers alone, some patients experience serious cardiac complications—including fatal myocardial infarction (MI).

That’s not surprising, given that most patients who require noncardiac vascular surgery suffer from severe coronary vascular disease.4 What is surprising is that most candidates for noncardiac vascular surgery are not put on statins prior to undergoing surgery.1,2,5

Statins were thought to increase—not prevent—complications
Until recently, taking statins during the perioperative period was believed to increase complications, including statin-associated myopathy. Indeed, guidelines from the American Heart Association (AHA), American College of Cardiology (ACC), and National Heart, Lung and Blood Institute (NHLBI) suggest that it is prudent to withhold statins during hospitalization for major surgery.6

1 small study hinted at value of perioperative statins
A small Brazilian trial conducted in 2004 called the AHA/ACC/NHLBI guidelines into question. the researchers studied 100 patients slated for noncardiac vascular surgery who were randomized to receive either 20 mg atorvastatin (Lipitor) or placebo preoperatively —and monitored them for cardiac events 6 months postoperatively. They found that the incidence of cardiac events (cardiac death, nonfatal MI, stroke, or unstable angina) was more than 3 times higher in the placebo group compared with patients receiving atorvastatin (26% vs 8%, number needed to treat [NNT]=5.6; P=.031).2

The results of this small single study, although suggestive, were not sufficiently convincing to change recommendations about the preoperative use of statins, however. A more comprehensive study was needed to alter standard practice, and the Schouten study that we report on below fits the bill.1

STUDY SUMMARY: Preoperative statin use cuts risk in half

Schouten et al followed 500 patients, who were randomized to receive either 80 mg extended-release fluvastatin (Lescol XL) or placebo for a median of 37 days prior to surgery.1 All enrollees were older than 40 years of age and were scheduled for noncardiac vascular surgery. the reasons for the surgery were abdominal aortic aneurysm repair (47.5%), lower limb arterial reconstruction (38.6%), or carotid artery endarterectomy (13.9%). Patients who were taking long-term beta-blocker therapy were continued on it; otherwise, bisoprolol 2.5 mg was initiated at the screening visit. Patients who were already taking statins (<50% of potential subjects) were excluded. Other exclusions were a contraindication to statin therapy; emergent surgery; and a repeat procedure within the last 29 days. Patients with unstable coronary artery disease or extensive stress-induced ischemia consistent with left main artery disease (or its equivalent) were also excluded.

The primary study outcome was myocardial ischemia, determined by continuous electrocardiogram (EKG) monitoring in the first 48 hours postsurgery and by 12-lead EKG recordings on days 3, 7, and 30. Troponin T levels were measured on postoperative days 1, 3, 7, and 30, as well. the principal secondary end point was either death from cardiovascular causes or nonfatal MI. MI was diagnosed by characteristic ischemic symptoms, with EKG evidence of ischemia or positive troponin T with characteristic rising and falling values.

To gauge fluvastatin’s effect on biomarkers, lipids, high-sensitivity C-reactive protein, and interleukin-6 were measured upon initiation of the medication and on the day of admission for surgery. Serum creatine kinase, alanine aminotransferase (ALT) levels, clinical myopathy, and rhabdomyolysis were monitored as safety measures, with levels measured prior to randomization, on the day of admission, and on postoperative days 1, 3, 7, and 30.

Both groups were similar in age (mean of 66 years), total serum cholesterol levels, risk factors for cardiac events, and medication use. About 75% of the enrollees were men. At baseline, 51% of the participants had a total cholesterol <213 mg/dL, and 39% had an LDL-C <116 mg/dL. Within 30 days after surgery, 27 (10.8%) of those in the fluvastatin group and 47 (19%) of patients in the placebo group had evidence of myocardial ischemia (hazard ratio=0.55; 95% confidence interval [CI], 0.34-0.88; P=.01). the NNT to prevent 1 patient from experiencing myocardial ischemia was 12.

 

 

 

Statin users had fewer MIs. A total of 6 patients receiving fluvastatin died, with 4 deaths attributed to cardiovascular causes. In the placebo group, 12 patients died, 8 of which were ascribed to cardiovascular causes. Eight patients in the fluvastatin group experienced nonfatal MIs, compared with 17 patients in the placebo group (NNT=19 to prevent 1 nonfatal MI or cardiac death (hazard ratio= 0.47; 95% CI, 0.24-0.94; P=.03).

Effects of statins were evident preoperatively. At the time of surgery, patients in the fluvastatin group had, on average, a 20% reduction in their total cholesterol and a 24% reduction in LDL-C; in the placebo group, total cholesterol had fallen by 4% and LDL-C, by 3%.

Patients receiving fluvastatin had an average 21% decrease in C-reactive protein, compared with a 3% increase for the placebo group. Interleukin-6 levels also were reduced far more in the fluvastatin group (33% vs a 4% reduction in the placebo group [P<.001]).

The medication was well tolerated. Overall, 6.8% of participants discontinued the study because of side effects, including 16 (6.4%) patients in the fluvastatin group and 18 (7.3%) in the placebo group. (After surgery, 115 [23.1%] of patients in the statin group temporarily discontinued the drug because of an inability to take oral medications for a median of 2 days.)

Rates of increase in creatine kinase of >10× the upper limit of normal (ULN) were similar between the fluvastatin and placebo groups (4% vs 3.2%, respectively). Increases in ALT to >3× ULN were more frequent in the placebo group compared with the fluvastatin group (5.3%, placebo; 3.2%, fluvastatin). No cases of myopathy or rhabdomyolysis were observed in either group.

WHAT’S NEW: Preop statins can be a lifesaver

The initiation of fluvastatin prior to vascular surgery reduced the incidence of cardiovascular events by 50%—a remarkable result. While patients at the highest risk were excluded from the study, those with lower cardiac risk nonetheless benefi ted from statin therapy. Experts have not typically recommended statins in the perioperative period for this patient population. the results of this study make it clear that they should.

CAVEATS: Extended-release formulation may have affected outcome

The statin used in this study was a longacting formulation, which may have protected patients who were unable to take oral medicines postoperatively. While we don’t know if the extended-release formulation made a difference in this study, we do know that atorvastatin was effective in the Brazilian study discussed earlier.

CHALLENGES TO IMPLEMENTATION: Preop statins may be overlooked

Not all patients see a primary care physician prior to undergoing vascular surgery. This means that it will sometimes be left to surgeons or other specialists to initiate statin therapy prior to surgery, and they may or may not do so.

Optimal timing is unknown. It is not clear how little time a patient scheduled for vascular surgery could spend on a statin and still reap these benefits. Nor do we know if the benefits would extend to patients undergoing other types of surgery; in a large study of patients undergoing all kinds of major noncardiac surgery, no benefits of perioperative statins were found.7

Adherence to the medication regimen presents another challenge, at least for some patients. In this case, however, we think the prospect of preventing major cardiac events postoperatively simply by taking statins for a month should be compelling enough to convince patients to take their medicine.

Acknowledgement
The PURLs Surveillance System is supported in part by Grant Number UL1RR024999 from the National Center for Research Resources; the grant is a Clinical Translational Science Award to the University of Chicago. The content is solely the responsibility of the authors and does not necessarily represent the ofcial views of either the National Center for Research Resources or the National Institutes of Health.

Click here to view PURL METHODOLOGY

PRACTICE CHANGER

HMG-CoA reductase inhibitors (statins), initiated 30 days before noncardiac vascular surgery, reduce the incidence of postoperative cardiac complications, including fatal myocardial infarction.1,2

STRENGTH OF RECOMMENDATION

A: 1 new randomized controlled trial (RCT), and 1 smaller, older RCT.

Schouten O, Boersma E, Hoeks S, et al. Fluvastatin and perioperative events in patients undergoing vascular surgery. N Engl J Med. 2009;361:980-989.

Durazzo AE, Machado FS, Ikeoka DT, et al. Reduction in cardiovascular events after vascular surgery with atorvastatin: a randomized trial. J Vasc Surg. 2004;39:967-975.

 

ILLUSTRATIVE CASE

A 67-year-old man with recurrent transient ischemic attacks comes in for a preoperative evaluation for carotid endarterectomy. The patient’s total cholesterol is 207 mg/dL and his low-density lipoprotein cholesterol (LDL-C) is 109 mg/dL. He takes metoprolol and lisinopril for hypertension.

Should you start him on a statin before surgery?

Nearly 25% of patients with peripheral vascular disease suffer from a cardiac event within 72 hours of elective, noncardiac vascular surgery.3 While most of these “complications” have minimal clinical impact and are detected by biochemical markers alone, some patients experience serious cardiac complications—including fatal myocardial infarction (MI).

That’s not surprising, given that most patients who require noncardiac vascular surgery suffer from severe coronary vascular disease.4 What is surprising is that most candidates for noncardiac vascular surgery are not put on statins prior to undergoing surgery.1,2,5

Statins were thought to increase—not prevent—complications
Until recently, taking statins during the perioperative period was believed to increase complications, including statin-associated myopathy. Indeed, guidelines from the American Heart Association (AHA), American College of Cardiology (ACC), and National Heart, Lung and Blood Institute (NHLBI) suggest that it is prudent to withhold statins during hospitalization for major surgery.6

1 small study hinted at value of perioperative statins
A small Brazilian trial conducted in 2004 called the AHA/ACC/NHLBI guidelines into question. the researchers studied 100 patients slated for noncardiac vascular surgery who were randomized to receive either 20 mg atorvastatin (Lipitor) or placebo preoperatively —and monitored them for cardiac events 6 months postoperatively. They found that the incidence of cardiac events (cardiac death, nonfatal MI, stroke, or unstable angina) was more than 3 times higher in the placebo group compared with patients receiving atorvastatin (26% vs 8%, number needed to treat [NNT]=5.6; P=.031).2

The results of this small single study, although suggestive, were not sufficiently convincing to change recommendations about the preoperative use of statins, however. A more comprehensive study was needed to alter standard practice, and the Schouten study that we report on below fits the bill.1

STUDY SUMMARY: Preoperative statin use cuts risk in half

Schouten et al followed 500 patients, who were randomized to receive either 80 mg extended-release fluvastatin (Lescol XL) or placebo for a median of 37 days prior to surgery.1 All enrollees were older than 40 years of age and were scheduled for noncardiac vascular surgery. the reasons for the surgery were abdominal aortic aneurysm repair (47.5%), lower limb arterial reconstruction (38.6%), or carotid artery endarterectomy (13.9%). Patients who were taking long-term beta-blocker therapy were continued on it; otherwise, bisoprolol 2.5 mg was initiated at the screening visit. Patients who were already taking statins (<50% of potential subjects) were excluded. Other exclusions were a contraindication to statin therapy; emergent surgery; and a repeat procedure within the last 29 days. Patients with unstable coronary artery disease or extensive stress-induced ischemia consistent with left main artery disease (or its equivalent) were also excluded.

The primary study outcome was myocardial ischemia, determined by continuous electrocardiogram (EKG) monitoring in the first 48 hours postsurgery and by 12-lead EKG recordings on days 3, 7, and 30. Troponin T levels were measured on postoperative days 1, 3, 7, and 30, as well. the principal secondary end point was either death from cardiovascular causes or nonfatal MI. MI was diagnosed by characteristic ischemic symptoms, with EKG evidence of ischemia or positive troponin T with characteristic rising and falling values.

To gauge fluvastatin’s effect on biomarkers, lipids, high-sensitivity C-reactive protein, and interleukin-6 were measured upon initiation of the medication and on the day of admission for surgery. Serum creatine kinase, alanine aminotransferase (ALT) levels, clinical myopathy, and rhabdomyolysis were monitored as safety measures, with levels measured prior to randomization, on the day of admission, and on postoperative days 1, 3, 7, and 30.

Both groups were similar in age (mean of 66 years), total serum cholesterol levels, risk factors for cardiac events, and medication use. About 75% of the enrollees were men. At baseline, 51% of the participants had a total cholesterol <213 mg/dL, and 39% had an LDL-C <116 mg/dL. Within 30 days after surgery, 27 (10.8%) of those in the fluvastatin group and 47 (19%) of patients in the placebo group had evidence of myocardial ischemia (hazard ratio=0.55; 95% confidence interval [CI], 0.34-0.88; P=.01). the NNT to prevent 1 patient from experiencing myocardial ischemia was 12.

 

 

 

Statin users had fewer MIs. A total of 6 patients receiving fluvastatin died, with 4 deaths attributed to cardiovascular causes. In the placebo group, 12 patients died, 8 of which were ascribed to cardiovascular causes. Eight patients in the fluvastatin group experienced nonfatal MIs, compared with 17 patients in the placebo group (NNT=19 to prevent 1 nonfatal MI or cardiac death (hazard ratio= 0.47; 95% CI, 0.24-0.94; P=.03).

Effects of statins were evident preoperatively. At the time of surgery, patients in the fluvastatin group had, on average, a 20% reduction in their total cholesterol and a 24% reduction in LDL-C; in the placebo group, total cholesterol had fallen by 4% and LDL-C, by 3%.

Patients receiving fluvastatin had an average 21% decrease in C-reactive protein, compared with a 3% increase for the placebo group. Interleukin-6 levels also were reduced far more in the fluvastatin group (33% vs a 4% reduction in the placebo group [P<.001]).

The medication was well tolerated. Overall, 6.8% of participants discontinued the study because of side effects, including 16 (6.4%) patients in the fluvastatin group and 18 (7.3%) in the placebo group. (After surgery, 115 [23.1%] of patients in the statin group temporarily discontinued the drug because of an inability to take oral medications for a median of 2 days.)

Rates of increase in creatine kinase of >10× the upper limit of normal (ULN) were similar between the fluvastatin and placebo groups (4% vs 3.2%, respectively). Increases in ALT to >3× ULN were more frequent in the placebo group compared with the fluvastatin group (5.3%, placebo; 3.2%, fluvastatin). No cases of myopathy or rhabdomyolysis were observed in either group.

WHAT’S NEW: Preop statins can be a lifesaver

The initiation of fluvastatin prior to vascular surgery reduced the incidence of cardiovascular events by 50%—a remarkable result. While patients at the highest risk were excluded from the study, those with lower cardiac risk nonetheless benefi ted from statin therapy. Experts have not typically recommended statins in the perioperative period for this patient population. the results of this study make it clear that they should.

CAVEATS: Extended-release formulation may have affected outcome

The statin used in this study was a longacting formulation, which may have protected patients who were unable to take oral medicines postoperatively. While we don’t know if the extended-release formulation made a difference in this study, we do know that atorvastatin was effective in the Brazilian study discussed earlier.

CHALLENGES TO IMPLEMENTATION: Preop statins may be overlooked

Not all patients see a primary care physician prior to undergoing vascular surgery. This means that it will sometimes be left to surgeons or other specialists to initiate statin therapy prior to surgery, and they may or may not do so.

Optimal timing is unknown. It is not clear how little time a patient scheduled for vascular surgery could spend on a statin and still reap these benefits. Nor do we know if the benefits would extend to patients undergoing other types of surgery; in a large study of patients undergoing all kinds of major noncardiac surgery, no benefits of perioperative statins were found.7

Adherence to the medication regimen presents another challenge, at least for some patients. In this case, however, we think the prospect of preventing major cardiac events postoperatively simply by taking statins for a month should be compelling enough to convince patients to take their medicine.

Acknowledgement
The PURLs Surveillance System is supported in part by Grant Number UL1RR024999 from the National Center for Research Resources; the grant is a Clinical Translational Science Award to the University of Chicago. The content is solely the responsibility of the authors and does not necessarily represent the ofcial views of either the National Center for Research Resources or the National Institutes of Health.

Click here to view PURL METHODOLOGY

References

1. Schouten O, Boersma E, Hoeks SE, et al. Fluvastatin and perioperative events in patients undergoing vascular surgery. N Engl J Med. 2009;361:980-989.

2. Durazzo AE, Machado FS, Ikeoka DT, et al. Reduction in cardiovascular events after vascular surgery with atorvastatin: a randomized trial. J Vasc Surg. 2004;39:967-975.

3. Pasternak RC, Smith SC, Jr, Bairey-Merz CN, et al. ACC/AHA/ NHLBI Clinical advisory on the use and safety of statins. Circulation. 2002;106:1024-1028.

4. Landesberg G, Shatz V, Akopnik I, et al. Association of cardiac troponin, CK-MB, and postoperative myocardial ischemia with long-term survival after major vascular surgery. J Am Coll Cardiol. 2003;42:1547-1554.

5. Hertzer NR, Beven EG, Young JR, et al. Coronary artery disease in peripheral vascular patients. A classification of 1000 coronary angiograms and results of surgical management. Ann Surg. 1984;199:223-233.

6. Brady AR, Gibbs JS, Greenhalgh RM, et al. Perioperative betablockade (POBBLE) for patients undergoing infrarenal vascular surgery: results of a randomized double-blind controlled trial. J Vasc Surg. 2005;41:602-609.

7. Dunkelgrun M, Boersma E, Schouten O, et al. Bisoprolol and fluvastatin for the reduction of perioperative cardiac mortality and myocardial infarction in intermediate-risk patients undergoing noncardiovascular surgery: a randomized controlled trial (DECREASE-IV). Ann Surg. 2009;249:921-926.

References

1. Schouten O, Boersma E, Hoeks SE, et al. Fluvastatin and perioperative events in patients undergoing vascular surgery. N Engl J Med. 2009;361:980-989.

2. Durazzo AE, Machado FS, Ikeoka DT, et al. Reduction in cardiovascular events after vascular surgery with atorvastatin: a randomized trial. J Vasc Surg. 2004;39:967-975.

3. Pasternak RC, Smith SC, Jr, Bairey-Merz CN, et al. ACC/AHA/ NHLBI Clinical advisory on the use and safety of statins. Circulation. 2002;106:1024-1028.

4. Landesberg G, Shatz V, Akopnik I, et al. Association of cardiac troponin, CK-MB, and postoperative myocardial ischemia with long-term survival after major vascular surgery. J Am Coll Cardiol. 2003;42:1547-1554.

5. Hertzer NR, Beven EG, Young JR, et al. Coronary artery disease in peripheral vascular patients. A classification of 1000 coronary angiograms and results of surgical management. Ann Surg. 1984;199:223-233.

6. Brady AR, Gibbs JS, Greenhalgh RM, et al. Perioperative betablockade (POBBLE) for patients undergoing infrarenal vascular surgery: results of a randomized double-blind controlled trial. J Vasc Surg. 2005;41:602-609.

7. Dunkelgrun M, Boersma E, Schouten O, et al. Bisoprolol and fluvastatin for the reduction of perioperative cardiac mortality and myocardial infarction in intermediate-risk patients undergoing noncardiovascular surgery: a randomized controlled trial (DECREASE-IV). Ann Surg. 2009;249:921-926.

Issue
The Journal of Family Practice - 59(2)
Issue
The Journal of Family Practice - 59(2)
Page Number
108-110
Page Number
108-110
Publications
Publications
Topics
Article Type
Display Headline
Start a statin prior to vascular surgery
Display Headline
Start a statin prior to vascular surgery
Sections
PURLs Copyright

Copyright © 2010 The Family Physicians Inquiries Network.
All rights reserved.

Disallow All Ads
Alternative CME
Article PDF Media